Download as pdf or txt
Download as pdf or txt
You are on page 1of 137

MPH-003

ELECTROMAGNETIC THEORY
Indira Gandhi National Open University
School of Sciences

Electrostatics 1
MPH-003
ELECTROMAGNETIC
Indira Gandhi National
Open University
THEORY
School of Sciences

Block

1
ELECTROSTATICS
UNIT 1
Electrostatics in Free Space 9
UNIT 2
Laplace’s and Poisson’s Equations 49
UNIT 3
Special Techniques 71
UNIT 4
Dielectric in Electric Field 89
UNIT 5
Dielectric Properties 119
Programme Design Committee
Prof. V.B. Bhatia, Retd. Prof. Enakshi Sharma Prof. G. Pushpa Chakrapani Prof. Suresh Garg, Retd.
University of Delhi, Delhi University of Delhi, South BRAOU School of Sciences,
Prof. Abhai Mansingh, Retd. Campus, Delhi Prof. Y.K. Vijay IGNOU, New Delhi
University of Delhi, Delhi Prof. H.S. Mani, Retd. University of Rajasthan, Prof. Vijayshri
Prof. Feroz Ahmed, Retd. IIT Kanpur Rajasthan School of Sciences,
University of Delhi, Delhi Prof. S. Annapoorni Prof. J. Nag, Retd. IGNOU, New Delhi
Prof Yashwant Singh, Retd. University of Delhi, Delhi Jadavpur University Prof. S.R. Jha
Banaras Hindu University, Prof. D. Choudhury Prof. Zulfequar, School of Sciences,
Varanasi University of Delhi, Delhi Jamia Milia Islamia, New Delhi IGNOU, New Delhi
Prof. Deepak Kumar Prof. T.R. Seshadri Dr. Om Pal Singh Prof. Shubha Gokhale
J.N.U., New Delhi University of Delhi, Delhi IGCAR, Kalpakkam, School of Sciences,
Tamil Nadu IGNOU, New Delhi
Prof. Vipin Srivastava Prof. S. Ghosh
Central University of J.N.U., New Delhi Prof. Prabhat Munshi Prof. Sanjay Gupta
Hyderabad, Hyderabad IIT Kanpur School of Sciences,
Prof. Neeraj Khare IGNOU, New Delhi
Prof. G.S. Singh IIT Delhi, Delhi Prof. R.M. Mehra, Retd.
IIT Roorkee, Roorkee Dept. of Electronics, South Dr. Subhalakshmi Lamba
Prof. V.K. Tripathi
Campus, University of Delhi, School of Sciences,
Prof. A.K. Rastogi. IIT Delhi, Delhi
Delhi IGNOU, New Delhi
J.N.U., New Delhi Prof. Pankaj Sharan, Retd.
Prof. S.K. Kulkarni Dr. M.B. Newmai
Prof. A.K. Ghatak Jamia Milia Islamia,
Pune University/ School of Sciences,
IIT Delhi, Delhi New Delhi
IISER Pune, Pune IGNOU, New Delhi
Prof. Rupamajari Ghosh Prof. Kirti Ranjan
J.N.U., New Delhi University of Delhi, Delhi

Course Design Committee


Prof. Suresh Garg, Retd. Prof. Roopmanjari Ghosh Prof. Vijayshri
SOS, IGNOU, New Delhi School of Physics Sciences, SOS, IGNOU, New Delhi
Jawaharlal Nehru University, Prof. S.R. Jha
Prof. Geeta Kaicker, Retd. Delhi
SOS, IGNOU, New Delhi SOS, IGNOU, New Delhi
Prof. A.K. Gangal Prof. Shubha Gokhale
Prof. B.P. Asthana IISER, Pune, Maharashtra SOS, IGNOU, New Delhi
Department of Physics,
Banaras Hindu University, Prof. P.K. Suresh Prof. Sanjay Gupta
Varanasi School of Physics, SOS, IGNOU, New Delhi
University of Hyderabad, Dr. Subhalakshmi Lamba
Prof. V.K. Tripathi Hyderabad SOS, IGNOU, New Delhi
Department of Physics,
IIT Delhi, Delhi Dr. M.B. Newmai
SOS, IGNOU, New Delhi

Block Preparation Team


Prof. Vijayshri (Units 1 to 3) Prof. S.R. Jha (Units 4-5)
School of Sciences, IGNOU, New Delhi School of Sciences, IGNOU, New Delhi
Course Coordinators: Prof. S.R. Jha, Prof. S. Gokhale
Block Production Team
Sh. Rajiv Girdhar
AR (P), IGNOU
Acknowledgement: Shri Gopal Krishan Arora, EDP, SOS for CRC preparation.
July, 2023
© Indira Gandhi National Open University, 2023
ISBN:
Disclaimer: Any materials adapted from web-based resources in this module are being used for educational purposes
only and not for commercial purposes.
All rights reserved. No part of this work may be reproduced in any form, by mimeograph or any other means, without
permission in writing from the Copyright holder.
Further information on the Indira Gandhi National Open University courses may be obtained from the University’s office at
Maidan Garhi, New Delhi-110 068 or the official website of IGNOU at www.ignou.ac.in.
Printed and published on behalf of Indira Gandhi National Open University, New Delhi by Prof. Meenal Mishra, Director,
SOS, IGNOU.
Printed at
CONTENTS
Block and Unit Titles 1
Credit page 2
Contents 3

ELECTROMAGNETIC THEORY: COURSE INTRODUCTION 5


BLOCK 1: Electrostatics 7

Unit 1 Electrostatics in Free Space 9


1.1 Introduction 9
1.2 Electric Field 10
1.2.1 Electrostatic Force Field 10
1.2.2 Electric Field due to Discrete Charges 12
1.2.3 Electric Field due to Continuous Charge Distributions 15
1.3 Electric Potential 17
1.3.1 Electric Potential due to Discrete Charges 18
1.3.2 Electric Potential due to Continuous Charge Distributions 20
1.4 Gauss’s Law 20
1.5 Applications of Gauss’s Law 24
1.5.1 Spherical Symmetry 25
1.5.2 Cylindrical Symmetry 27
1.5.3 Planar Symmetry 31
1.6 Summary 34
1.7 Terminal Questions 35
1.8 Solutions and Answers 35

Unit 2 Laplace’s and Poisson’s Equations 49


2.1 Introduction 49
2.2 Laplace’s Equation 50
2.3 Poisson’s Equation 55
2.4 Boundary Value Problems in Electrostatics 56
2.4.1 Laplace’s Equation 57
2.4.2 Poisson’s Equation 62
2.5 Summary 63
2.6 Terminal Questions 63
2.7 Solutions and Answers 64

Unit 3 Special Techniques 71


3.1 Introduction 71
3.2 The Method of Images 71
3.3 Applications 75
3.4 Multipole Expansion 82
3.5 Summary 84
3.6 Terminal Questions 84
3.7 Solutions and Answers 85

3
Unit 4 Dielectric in Electric Field 89
4.1 Introduction 89
4.2 Dielectrics 90
4.3 Dielectric Material in Electric Field: Polarisation 92
4.3.1 Induced Dipoles in Neutral Atoms and Non-polar Molecules 93
4.3.2 Alignment of Polar Molecules in Electric Fields 95

4.3.3 Polarisation of Dielectrics and Polarisation Vector P 96
4.4 Electric Field of a Polarised Object 99
4.4.1 Physical Interpretation of Bound Charge Density  101
4.5 Electrostatic Equation in Dielectrics: Displacement Vector D
and Gauss’s Law   103
4.6 Boundary Conditions on E and D 108
4.7 Summary 110
4.8 Terminal Questions 112
4.9 Solutions and Answers 113

Unit 5 Dielectric Properties 119


5.1 Introduction 119
5.2 Electric Fields in a Dielectric 120
5.2.1 Molecular or Local Field and Average Macroscopic Field
in a Dielectric 120
5.2.2 Determination of Molecular (Local) Field in a Spherical
Cavity in the Dielectric 122
5.3 Relation between Polarisability and Dielectric Constant:
Clausius-Mossotti Equation 124
5.3.1 Atomic/Molecular Polarisability: A Simple Model 126
5.4 Electrostatic Energy in a Dielectric 128
5.5 Summary 130
5.6 Terminal Questions 131
5.7 Solutions and Answers 131

4
ELECTROMAGNETIC THEORY: COURSE
INTRODUCTION
The word electromagnetism comes from a combination of electricity and magnetism. Electric
and magnetic phenomena have been observed in nature since ancient times and they were
considered as two entirely separate phenomena. The discoveries of Oersted and Faraday
regarding the magnetic effects of current and electromagnetic induction changed things
dramatically. These developments and discoveries indicating some kind of relation between
electric and magnetic phenomena culminated in the work of Maxwell, who clearly established
that electricity and magnetism are two aspects of the same phenomenon. He developed a
complete and consistent theory of electromagnetism by putting together the four fundamental
laws, namely, Gauss’ law for electric fields, Gauss’ law for magnetic fields, Ampere’s law and
Faraday’s law that govern electric and magnetic phenomena.

Electromagnetic theory lies at the core of modern physics and plays a vital role in countless
technological advancement. Hardly ever has a scientific theory had such profound and
far-reaching consequences. We are all familiar with innumerable electrical appliances and
machinery which we use all the time in our daily lives. The harnessing of electrical power and
the development of electrical/electronic communication has changed our life. From the design
of intricate circuitry to the development of wireless communication systems and the
understanding of the behaviour of celestial bodies, this course will equip you with the
theoretical framework and analytical tools to understand and analyse the behaviour of electric
and magnetic fields in a wide range of applications.

Maxwell’s contribution in the form of electromagnetic theory led to a revolutionary change in


human understanding of the way nature works. It paved the way for future path breaking
advances in physics. In fact, on the centenary of Maxwell's birthday, Einstein described
Maxwell's work as the "most profound and the most fruitful that physics has experienced
since the time of Newton.” Maxwell’s theory of electromagnetism has evolved into a vast body
of knowledge due to its numerous applications. This has necessitated presenting the subject
matter based on the Maxwell’s theory as two separate courses namely, Electromagnetic
Theory and Classical Electrodynamics in this Master’s programme in Physics.

In this course on Electromagnetic Theory, we will investigate the realm of electrostatics and
magnetostatics. While electrostatics is the study of electric fields produced by stationary
charges, magnetostatics deals with the stationary magnetic fields and their interactions. In the
last unit, we shall see how electrostatics and magnetostatics are two aspects of the same
phenomenon, namely, electromagnetism.

This is a 2-credit core course for students of M Sc (Physics) programme. The course is
presented in two blocks:

1. Electrostatics
2. Magnetostatics

In Block 1 entitled Electrostatics, we will begin by revisiting the principles and concepts of
electrostatics, such as the electrostatic force, electric field and electric potential. You have
learnt these principles and concepts in undergraduate physics. However, these concepts are
fundamental for understanding the physics of electric phenomena. You will learn to cast
electrostatic problems in the form of Laplace equation and Poisson equation and solve them.
You will learn another technique called method of images to solve electrostatic problems. In
5
real life, we mostly have situations in which electric phenomena take place in material
medium. Different kinds of materials such as conductors and insulators behave differently in
the electric field. You are familiar with the behaviour of the conductors in electric field. Thus,
in this block, we have discussed electrostatics in the insulators (also called dielectrics) as one
of the major themes.

In Block 2 entitled Magnetostatics, you will discover that moving charges produce magnetic
field. We also discuss Biot-Savart law which allows us to determine the magnetic field due to
various current distributions. You will see how a magnetic field is defined in terms of the
forces on moving charges and on current-carrying wires. In addition, the expression for the
torque on a current-carrying loop in a magnetic field is derived, which is the basis of classical
understanding of the magnetic properties of materials. We also investigate how various
materials behave in a magnetic field. Some major experimental observations related to the
response of magnetic materials to the magnetic field have been discussed on the basis of the
concepts of classical physics and some elementary quantum ideas. You will also learn the
concept of magnetisation and magnetic parameters such as magnetic susceptibility and
permeability which allow us to classify magnetic materials in to three broad categories,
namely, diamagnetic, paramagnetic and ferromagnetic. The block culminates with an
explanation of how the laws of electrostatics and magnetostatics were unified by Maxwell,
who developed the electromagnetic theory.

Throughout this course, we will adopt a rigorous mathematical approach to electromagnetic


theory, emphasizing the importance of vector calculus and mathematical techniques in
solving related problems. You will develop the skills to apply these mathematical tools to
analyse and solve a wide range of electromagnetic problems, enabling you to gain a profound
understanding of electrostatics and magnetostatics, and of electromagnetism.

We hope that you enjoy studying the course.

Our best wishes are with you!

6
BLOCK 1: ELECTROSTATICS
We observe electrostatic phenomena all around us. You must have observed that when you
run a comb through your dry hair, your hair gets attracted to the comb. Similarly, you may
have noted that when you removed a plastic wrap from a packet, plastic was attracted to your
hands. In fact, it has been known since ancient times that rubbing of some materials gives
them the ability to attract small, light objects. This is electrostatics at work. Lightning is the
most striking visual display of electrostatics in nature. Electrostatics has numerous
applications in modern life. These include photocopiers (Xerox machines), laser printers,
defibrillators, paint spraying machines, electrostatic precipitators, etc. Electronic components
such as capacitors and computer peripherals such as laser printers are based on principles of
electrostatics.

In this block entitled Electrostatics, you will revise the concepts of electrostatic forces
between static electric charges in free space, electric fields and electric potentials due to
them. You have studied these concepts in your school and UG courses in physics. The
principles and concepts of electrostatics, such as the electrostatic force, electric field and
electric potential were developed by scientists Coulomb, Gauss and many others. These
concepts are fundamental for understanding the physics of electric phenomena including
electric charges at rest, and their interactions. Therefore, we begin the discussion in this block
with a revision of these concepts. The block contains five units.

In Unit 1, entitled Electrostatics in Free Space, we revise the basic concepts of electrostatic
force, electric field and electric potential due to a free stationary charge or a system of static
discrete charges and charge distributions. We also revisit Gauss’s law that helps us
determine electric fields due to symmetric charge distributions along with its applications to
charged systems having spherical, cylindrical and planar symmetry.

Unit 2 is entitled Laplace’s and Poisson’s Equations. As the unit title indicates, we discuss
these two equations which are two of the most important equations of electrostatics. In this
unit, you will learn the methods of solving these equations and then apply them to solve a few
boundary value problems in electrostatics.

In Unit 3, entitled Special Techniques, we discuss the method of images and the technique
of multipole expansion to solve problems in electrostatics.

In Unit 4, entitled Dielectric in Electric Field, we describe the macroscopic behaviour of a


dielectric in an electric field. As you know, in a dielectric (insulator), there are no free charges
that can move through the material under the influence of an electric field. The only possible
motion in an insulator, in the presence of an electric field, is a small displacement of the
positive and negative charges in opposite directions. We explain the behaviour of such
materials in external electric fields in terms of polarisation in a dielectric. We then introduce
some macroscopic properties of a dielectric such as its dielectric constant and electrical
susceptibility, which can be measured experimentally. We also discuss the modification in
Gauss’s law in free space that you have studied in Unit 1 and obtain Gauss’s law for a
dielectric medium.

In Unit 5, entitled Dielectric Properties, we explore what is happening at the atomic or


molecular level in a dielectric when it is polarised. We first define a parameter called local or
molecular electric field responsible for polarising a molecule of the dielectric and then 7
establish its relation with the macroscopic electric field inside the dielectric. This enables us to
obtain the Clausius-Mossotti equation which relates the macroscopic property and
microscopic property of a dielectric. We also discuss a simple physical model of a non-polar
dielectric which is consistent with the predictions of Clausius-Mossotti equation. We end the
discussion on electrostatics with deriving an expression for the electrostatic energy in a
dielectric.

We hope that you enjoy learning these concepts of electrostatics in free space and material
media discussed in this block and wish you success!

8
Unit 1 Electrostatics in Free Space

UNIT 1
ELECTROSTATICS IN
FREE SPACE
Structure

1.1 Introduction 1.4 Gauss’s Law


Expected Learning Outcomes 1.5 Applications of Gauss’s Law
1.2 Electric Field Spherical Symmetry
Electrostatic Force Field Cylindrical Symmetry
Electric Field due to Discrete Charges Planar Symmetry
Electric Field due to Continuous 1.6 Summary
Charge Distributions 1.7 Terminal Questions
1.3 Electric Potential 1.8 Solutions and Answers
Electric Potential due to Discrete Charges
Electric Potential due to Continuous Charge
Distributions

1.1 INTRODUCTION
In your UG physics courses on Electricity and Magnetism, you have studied
about electrostatic force, Coulomb’s law, electric field, electric potential and
Gauss’s law with its applications. This unit is essentially a revision unit for all
these basic concepts of electromagnetic theory. Therefore, we will recapitulate
all these concepts in Secs. 1.2 to 1.4 without going into detailed explanations.
If you wish, you may like to refer to any undergraduate level physics text book
on electricity and magnetism. We also advise you to refresh your knowledge of
vector analysis so that you understand the mathematical treatment easily and
have better comprehension of the concepts of this unit.

In your UG courses, you have learnt how to determine the electrostatic force,
electric field and electric potential due to point charges at rest, and continuous
charge distributions in which charges are at rest. This is what electrostatics
is about: Calculating electric fields due to charges and electrostatic
forces on a charge or distribution of charges placed in an electric field,
and electric potentials due to them.

In this unit, we revise the tools that simplify the calculation of electric fields,
electric potentials and electrostatic forces. We also revisit Gauss’s law (along
with the concept of electric flux), which relates electric charge distributions
and electric fields and gives us a simple method to determine electric fields 9
Block 1 Electrostatics
associated with symmetric charge distributions. You know that Gauss’s law
relates the electric flux or electric field due to the charges/charge distributions
to the net charge. As you know, Gauss’s law has many applications in
systems having spherical, cylindrical, planar or any symmetry.

In the next unit, we discuss Poisson’s and Laplace’s equations, which you
have studied in Block 1 of the course MPH-001 with applications in
electrostatics.

Expected Learning Outcomes


After studying this unit, you should be able to:

 define electric field and electric potential due to multiple discrete charges
and continuous charge distributions;
 calculate the net electric field and electric potential due to a distribution of
multiple discrete charges and continuous charge distributions;
 define electric flux and calculate the electric flux due to an arbitrary
distribution of charges;
 state and apply Gauss’s law to calculate the electric field due to multiple
discrete charges and continuous symmetric charge distributions;

1.2 ELECTRIC FIELD


In this section, we will revise the concept of electric field and determine the
electric fields due to multiple discrete charges and continuous charge
distributions. We begin by revising the concept of electrostatic force.

1.2.1 Electrostatic Force Field


You know Coulomb’s law from your school and UG physics courses. It gives
the electrostatic force between like and unlike charged particles at rest. So,
let us briefly recapitulate it. Coulomb discovered experimentally that the
magnitude of the force (electric, Coulomb or electrostatic force as we know it
today) between two charged particles q1 and q 2 at rest is given by:

q1 q2
F  k (1.1)
r2
where r is the distance between the charged particles and k is the constant of
proportionality. The force is directed along the line joining the two particles.
The force on either particle is directed toward the other particle if the two have
opposite (unlike) charges and away if the two have similar (like) charges. So
we say that like charges repel and unlike charges attract each other. Since
force is a vector quantity, we state Coulomb’s law in vector form:
The electrostatic force on a particle carrying a charge q1 by a particle
carrying a charge q 2 situated at a distance r from it is given by
 qq
F21  k 1 2 rˆ21 (1.2)
2
r21
10
Unit 1 Electrostatics in Free Space
where rˆ21 is the unit vector along the line joining the particles and directed from

q 2 to q1 (see Fig. 1.1) and k is called the Coulomb constant. Note that F21
r21  r1  r2 and r21  r . Here r1 and r2 are the position vectors of q1 and
q 2 , respectively. Note also that the particles are at rest. In SI units, Coulomb’s
q1
law is written as
rˆ21 
r1
 1 q1q 2
F21  rˆ21 (1.3)
4 0 r21 2 q2

  r2
where the units of q1 and q 2 are coulomb, those of r21 and F21 are metre and
newton, respectively. The constant  0 is the permittivity of free space and
O
1
 8.99 109 N m2 C 2 .
4 0 Fig. 1.1: The electrostatic
force between two
Note that Eqs. (1.2 and 1.3) account for the attractive and repulsive nature of electric charges at rest.
the electrostatic force if q1 and q 2 include the sign of the charge. So, if the
charges
 are like, that is, both charges are either positive or negative, the

force F21 on q1 points away from q 2 , along r21 , i.e., it is repulsive. If the
charges are
 unlike, that is, one of them is positive and the other negative,
the force F21 on q1 is towards q 2 , in the direction opposite to r21 , i.e., it is
attractive. Actually, Eqs. (1.2 and 1.3) represent the electrostatic force field of
the charges. It is a vector field. (You should refresh the concept of vector fields
from your undergraduate physics course.)

The mathematical expressions of Coulomb’s law, given by Eqs. (1.2 and 1.3)
sum up four experimental observations:

Superposition principle
1. Unlike charges attract and like charges repel; follows from the notion
that electrostatic forces
2. The force between two charged particles is exerted along the line are two-body forces,
joining them; that is, the electrostatic
force between any pair
3. The force between any two charged particles is proportional to of charged objects does
the magnitude of charge on each particle; and not change if other
charged objects are
4. It is an inverse square force, i.e., it is inversely proportional to the present in their
square of the distance between the particles. surroundings. To
determine the net
electrostatic force on
Of course, you would have calculated electrostatic forces due to a system of
any given charged
charges by applying Coulomb’s law along with the superposition principle, particle in a system of
which we state here: charged particles, by
the other charged
Superposition Principle particles in the system,
we simply take the
According to the superposition principle (read the margin remark), in a many- vector sum of the

particle system of charged particles, the net electrostatic force Fi on the ith forces being exerted on
it by the other charged
charge q i due to all other charges q1, q2,...., q j ,... situated at distances r ji from
particles in the system.
it, is given by:
11
Block 1 Electrostatics
  qi q j
 F ji  4 0 
1
Fi  rˆji (1.4)
2
j i j  i r ji

Note that the summation in Eq. (1.4) does not include the ith charge. This is
indicated by putting j  i under the summation signs.

You may like to revise the calculations of electrostatic force. Solve SAQ 1.

y
SAQ 1

a) Determine the electrostatic force on q1 due to q2 for:


q3
i) q1  8.0  C, q2  8.0  C at a distance of 0.04 m.
0.4 m

ii) q1  15 m C, q 2   10 m C at a distance of 3.0 m.


q2 b) Three charges q1   2.0  C, q 2  9.0  C and q3  16.0  C are
q1 x situated at the corners of a right-angled triangle as shown in Fig. 1.2.
0.3 m
Calculate the electrostatic force exerted on q1 by q 2 and q 3 .
Fig. 1.2: Diagram for
SAQ 1b.

With this revision of the concept of electrostatic force between charged


At many places we will
particles/objects at rest, let us now quickly discuss the concept of electric field.
refer to point charges.
A point charge is a You may ask: How do we define electric field? We begin with the simplest
hypothetical charge case of a point charge and extend it to a system of discrete charges.
located at a single
point in space. In that 1.2.2 Electric Field due to Discrete Charges
sense, it has no size: it
is dimensionless. It is a The concept of electric field was developed by James Clerk Maxwell. You
purely abstract know that the concept of electric field is a very powerful concept in that it gives
mathematical concept us a simple tool for determining the electrostatic force on any charge due to
used in electrostatics.
another charge. We begin this section by explaining briefly, the concept of the
For many purposes,
electric field due to a point charge:
we consider the
electron to be a point
A point charge Q sets up an electric field in the region surrounding it. If another
charge. However, its charge, say q, is placed in this region, it experiences the electrostatic force in
size can be
characterized by a accordance with Coulomb’s law. The electric field generated by an electric
length scale known as charge or a group of charges is a vector field defined as follows:
the electron radius. We Suppose a positive charge q of an infinitesimal (negligibly small) magnitude,
often use the term
called a test charge, is placed at a position r relative to a point charge Q
point charge in
electrostatics when
(Fig. 1.3). According to Coulomb’s law, at that point, the test charge q will
we do not wish to take experience the electrostatic force
the size (dimensions)   1 Qq
F (r )  rˆ (1.5)
of the particle into 4 0 r 2
consideration. 
where rˆ is the unit vector along the vector r . Then the electric field of the point
charge Q at a point having position vector r is defined as the electrostatic force
on a test charge
  at that point divided by the magnitude of the test charge. It is
denoted by E (r ). Mathematically, it is given by

  F (r ) 1 Q
E (r )   rˆ (1.6a)
q 4 0 r 2
12
Unit 1 Electrostatics in Free Space
Its magnitude is given by
P
1 Q
E (1.6b)
4 0 r 2 r

The representations of the electric fields defined by Eqs. (1.6a and b) are rˆ
shown in Figs. 1.4a and b for positive and negative charges. Q

The electric field due to a positive point charge is directed away from the Fig. 1.3: Unit vector for
charge (Fig. 1.4a). For a negative point charge, it points towards the charge electric field at point P
(Fig. 1.4b). The arrows in both Figs. 1.4a and b indicate the direction of the due to a point charge
electric field. The continuous lines are called field lines (or the lines of force). Q.

 

(a) (b)
Fig. 1.4: Electric field lines around a) positive electric charge; b) negative
electric charge.

From Eq. (1.6a), you should also note that the electrostatic force on the
charge q when it is placed in the electric field of charge Q is given by
 
F  qE (1.7)

So, if you know the electric field in a region of space (could be due to a charge
or system of charges), you can determine the electrostatic force on any charge
placed in that electric field using Eq. (1.7).
Now let us define the electric field due to a system of charges.
Electric Field due to Multiple Discrete Charges
Consider a group of point charges q j , having position vectors r j . Let us place
a test charge q having position vector ri in the electric field of these charges.
From the principle of superposition for electrostatic forces, the net electrostatic
force on the test charge q due to this group of charges is given by

1  q q1 q q2 
 (1.8a)
F  F1  F2  ...    ...
40  r  r 2 r  r
2 
 i 1 i 2 

q  q1 q2 
 q 
qj
   ... rˆji
40  r  r 2
r  r
2  40 j  i
ri  r j
2
 i 1 i 2 
(1.8b)

The electric field due to the group of charges at the point with position vector

ri is defined as:

1 qj
E  E j , Ej  rˆji (1.9)
j 40 r  r 2
i j 13
Block 1 Electrostatics
Hence,
F 1 qj
E (ri )   
q 40 j  i r  r 2
rˆji (1.10)
i j

Eq. (1.10) defines the electric field at a point in space due to a system of
point charges. So, the total electric field due to a group of charges is the
vector sum of the individual electric fields of the charges. This is just the
principle of superposition at work. You may like to study Fig. 1.5 to get a
sense of the vectors involved in Eq. (1.10) before reading further.
P  
 ( ri  r j )
ri
qj

rj
O

r3 q3
 
r1 r2

q1 q2

Fig. 1.5: The vectors involved in defining the electric field due to a group of
  
charges. The vector r ji  ( ri  r j ) represents the vector joining q j to the
 
point P having position vector ri . The vector rˆji is the unit vector along r ji .
q C q r P
Notice that the electric field is a function of only the position ri at which we
d
r wish to determine the electric field because all other separation vectors
(a) depend on the position of this point. Also note that the test charge does not
appear in the expression of the electric field. Now, if we place a charge q in
the electric field given by Eq. (1.10), the electrostatic force exerted on it will be
given by Eq. (1.7). So, the concept of electric field makes the calculation of
P electrostatic force on a charge due to a group of charges much easier in
comparison to using Coulomb’s law.
Before studying further, you may like to calculate the electric field due to an
r
electric dipole for practice. Work out SAQ 2.
 
q q
d C SAQ 2
2d The equal and opposite point charges  q and  q in an electric dipole are
(b) separated by distance 2d.
a) Determine the net electric field due to the charges at the point P located on
Fig. 1.6: a) Electric field
the dipole axis (i.e., the line joining the charges) at a distance r from the
due to an electric
dipole at a point along midpoint C of the dipole axis (Fig. 1.6a);
the axis of the dipole. b) Determine the net electric field due to the electric dipole at a point P
The point P lies on the
situated on the perpendicular bisector at a distance r from the midpoint C
dipole axis at a
distance r from the of the dipole axis (Fig. 1.6b).
midpoint C; b) Diagram
for SAQ 2b. So far, we have defined the electric field and calculated its value for an
14 isolated point charge or a system of two or more point charges. We now define
Unit 1 Electrostatics in Free Space
the electric field of a continuous charge distribution, for example, charge
distribution on a wire, lamina, cylinder or sphere.

1.2.3 Electric Field due to Continuous Charge


Distributions
For a continuous distribution of charge (Fig. 1.7), the sum in Eq. (1.10) can be

written as an integral and the electric field at point P with position vector r due
to it is defined as:
 1 dq
E
4 0  r 2 rˆ (1.11)
P

The limits of the integral are defined so that the entire region over which r
charge is distributed is included. Remember that in Eq. (1.11), rˆ is the unit

vector from the charge dq to the point P (having position vector r ) at which dq
the electric field is being determined (see Fig. 1.7).

If the charge is distributed over a line, as in a wire (Fig. 1.8a), then we speak
of the line charge density, i.e., charge per unit length and usually denote it by Fig. 1.7: Electric field
. The SI unit of  is C m1. For continuous charge distribution over a surface at a point due to a
continuous charge
(Fig. 1.8b), we define the surface charge density  as the charge per unit
distribution.
area. Its SI unit is C m 2 . If the continuous charge distribution is spread over a
volume (Fig. 1.13c), then we use the volume charge density , which is the
charge per unit volume. Its SI unit is Cm3 .
P
P P

r r
r da d
dl 
Line charge density  Surface charge density  Volume charge density 
(a) (b) (c)

Fig. 1.8: Electric field due to a) line charge distribution; b) surface charge
distribution; c) volume charge distribution.

The element of charge dq for line, surface or volume charge distribution is


expressed in terms of the respective charge densities as follows:

dq  (r ) d (1.12a)

dq  (r ) dS (1.12b)

dq   (r ) dV  (1.12c)

Thus, the electric field for each type of charge distribution is given by:
 1 (r ) d
E
4 0  r2
rˆ Line charge (1.13a)
C

 1 (r ) dS 
E
4 0  r2
rˆ Surface charge (1.13b)
15
S
Block 1 Electrostatics
 1  (r ) dV 
E
4 0  r2
rˆ Volume charge (1.13c)
V

If the line, surface or volume charge is distributed uniformly, i.e., the respective
charge densities are constant, then we get the following results.

Electric field due to a uniformly distributed line charge (constant ):


  d
E
4 0  r 2 rˆ (1.14a)
C

Electric field due to a uniformly distributed surface charge (constant ):


  dS
E 
4 0  r 2 rˆ (1.14b)
S

Electric field due to a uniformly distributed volume charge (constant ):


  dV
E 
4 0  r 2 rˆ (1.14c)
V

You should study your undergraduate level mathematics to revise the


evaluation of line, surface and volume integrals. If we use Eqs. (1.14a to c) for
determining electric fields, the calculations can become quite tedious. You will
learn an elegant and simple method for such calculations albeit for symmetric
charge distributions in Sec. 1.4.

You may like to pause now and review what you have learnt in this section.
You have learnt the definition of the electric field and calculated it for a point
charge and systems of discrete point charges. To sum up, we would like to
emphasize: What exactly is an electric field?

You should think of the electric field as a real physical entity which exists in
the space in the neighbourhood of any charge, groups of charges or
continuous charge distributions, which set up the electric field. Any charge
kept in the electric field experiences the electrostatic force given by Eq. (1.7).
The concept of electric field is abstract and difficult to imagine concretely. But
you have learnt how to calculate the electric field and also the electrostatic
force experienced by a charge kept in the electric field.

Remember that the aim of all electrostatics is the determination of the


electrostatic forces and electric fields due to a given charge distribution.
However, the integrals involved in calculating electric fields can be quite
complicated even for simple charge distributions. Therefore, much of
electrostatics involves learning the methods that simplify these calculations so
that we have no need to solve such complicated integrals.

One such tool is provided by Gauss’s law for symmetric charge distributions
and another by the concept of electric potential.

We now revise the concept of electric potential, which you have studied in
your UG physics in quite some detail.
16
Unit 1 Electrostatics in Free Space

1.3 ELECTRIC POTENTIAL


You have learnt in your UG courses that the electrostatic force field is
conservative, which means that the work done by the electrostatic force field in
moving a particle around a closed path is zero. This also means that the
corresponding electric field is conservative since the force field is its scalar
multiple. Therefore, the curl of the electric field due to charges at rest will always
be zero:

E 0 (1.15a)

Then as you know from your UG courses, we can express the electric field as the
gradient of a scalar field:

E   V (1.15b)

Also, for a conservative field, the line integral of the field around any closed path
is zero:
 
E . dl  0 (1.15c)

Or the line integral of the electric field from any point A to any other point B is
independent of the path between the two points (that is, it is the same for all

paths between the points). Hence, we can define a scalar function V (r ) such
that:

r  


V (r )   E . dl (1.16a)
R

where R is some standard reference point. Usually, we take the reference point
 
at  in potential theory. The scalar function V (r ) that depends on the point r is
called the electric potential.

Then we define the potential difference between two points a and b as:
 
b   a   If we use Eq. (1.10)
 

V ( b )  V ( a )   E . d l  E . dl
 

for the electric field
and express the sum
R R
as an integral, then r
   in Eq. (1.13c) is
b   R   b    
  actually (r  r ),
or  
V (b )  V (a )   E . dl  E . dl   E . dl
 

(1.16b)
which is implied in
R a a
Eqs. (1.13a to
1.14c). You may like
Using a more general expression of Eq. (1.13c) for the electric field due to a to revisit the concept
volume charge distribution, we can establish a general expression for the electric of volume integral to
potential in terms of the volume charge density (read the margin remark): understand how this
sum is expressed as
 1  (r ) d 1  (r ) d 
 
the volume integral.
E rˆ  r
4 0 r2 4 0 r 3
V V

  
1 ( r  r )
or E 
4 0 
 (r )   d
r  r
3
(1.17a)
V 17
Block 1 Electrostatics
where we have used the symbol d for the volume element. Now, we can use
the following result from vector calculus to obtain an expression for the electric
potential:
   1 
( r  r )  
  3     r  r 
r  r  

Then we can write Eq. (1.17a) as:

   ( r ) 
1  d 
E 
4 0   
r  r
 
(1.17b)
 V 

Comparing Eq. (1.17b) with Eq. (1.15b), we get the expression for the electric
potential as:

1   ( r ) 
V    d  (1.17c)
4 0  r  r 
 V 

This is a scalar function and it is arbitrary to the extent of an integration constant.

In this manner, we have reduced a vector problem to a scalar problem! You will
agree that it is far easier to work with a scalar potential function and then obtain
the electric field by taking its gradient. Let us now determine the electric potential
due to discrete charges.

1.3.1 Electric Potential due to Discrete Charges


From school and UG physics, you know that the electric potential due to a
positive point charge Q at a point at distance r from it is given by:

Q
V (1.18)
4 0 r

Let us determine the electric potential due to an electric dipole, which is a well
known potential.

Example 1.1

Determine the potential due to an electric dipole and hence, its electric field.

Solution : We shall use polar coordinates for mathematical convenience. Refer


to Fig. 1.9, which shows a point P at a distance r from the midpoint C of the
dipole AB. The line joining P and C makes an angle  with the dipole axis.

Note that the polar coordinates of point P are r and  with respect to the origin at
C, the midpoint of dipole. The electric potential at P due to the two charges
 q and  q of the dipole is (read the margin remark on the next page):

q  1 1  2q a cos 
V  (r  a cos )  (r  a cos )  
4  0   4  0 (r 2  a 2 cos2 )

(1.19a)
18
Unit 1 Electrostatics in Free Space
 
Now, let us suppose that r is a vector from C to P and the unit vector along r is We use the result that
rˆ. Also, you know that the dipole moment, p  2q a. Since the electric potential
 
p.rˆ  2q a.rˆ  2qa cos , we can write Eq. (1.19a) for V as: due to a point charge
q is:

p . rˆ V 
q
V (1.19b) 4  0 r
2 2 2
4 0 (r  a cos )
The distances AP and
BP of point P from  q
and  q are:
BP  PC  CS
 r  a cos 

and
AP  TC  CP
 r  a cos 
The electric potential
at P due to charge q
is:
q 1
V q 
4  0 (r  a cos )
and that due to charge
 q is:
q 1
V q  
4  0 (r  a cos )
Fig. 1.9: An electric dipole AB of length 2a and point P at a distance r from the
mid-point C of the dipole. From superposition
principle, the resultant
When point P is far away from the dipole, r 2  a 2 cos2  and neglecting
potential is a sum of
a2 cos2  in the denominator, we write Eq. (1.19b) as: V q and V q .
p . rˆ p cos 
V  (1.19c)
2
4  0 r 4  0 r 2

Eq. (1.19c) gives the electric potential due to an electric dipole at a distance r
from its mid-point. Let us now determine the electric field of a dipole from its
electric potential using Eq. (1.15b) and polar coordinates. Thus, putting
 ˆ1 
  rˆ 
r r 
in Eq. (1.15b) and using Eq. (1.19e) for V, we get the electric field of the electric
dipole as:

   ˆ    p cos      p cos   ˆ   p cos  


E  rˆ     rˆ    
 r r    4 0r 
2  r  4 r 2  r   4 r 2 
 0   0 


1 p
4  0 r 3
 
rˆ(2 cos )  ˆ sin 

(1.20a)
For   0, the point P will lie along the axis of the dipole. Then Eq. (1.20a)
becomes:
 
1 2p 1 2p 
E rˆ  , where p  prˆ (1.20b)
4 0 r 3 4 0 r 3 19
Block 1 Electrostatics
This is the result you have obtained on solving SAQ 2a. Similarly, you can obtain
the result of SAQ 2b from Eq. (1.19).

You will agree that this is a much simpler way of determining the electric field of
an electric dipole than the one used in solving SAQs 2a and b. This is true for
most problems in electrostatics.

Let us now determine the electric potential due to continuous charge


distributions.

1.3.2 Electric Potential due to Continuous Charge


Distributions
In your UG physics, you have learnt how to determine the electric potential due
to various continuous charge distributions. Here we will just write the expressions
of electric potentials for a line charge and a uniformly charged non-conducting
sphere. Electric potential due to a line charge is:

 ln r
V (r )   (1.21a)
2 0

Potential due to a uniformly charged non-conducting sphere of radius R having


total charge Q at a point outside it is:

Q
V (r )  (1.21b)
4  0 r

and at a point inside it is:

Q (3R 2  r 2 )
V (r )  (1.21c)
8  0 R 3

(Solve Terminal Questions 3 and 4).

You should refresh the method of calculating the electric potential due to other
continuous charge distributions from your undergraduate physics course.

Let us now revise Gauss’s law, which you have studied in your UG physics. It
gives us another way to simplify the determination of electric fields for symmetric
charge distributions.

1.4 GAUSS’S LAW


In your UG Physics courses, you have studied about Gauss’s law which relates
electric flux associated with an electric field E through a surface of area S.

Gauss’s law states that the net electric flux through any imaginary closed
surface S (called the Gaussian surface) is directly proportional to the net charge
Q
(Qencl ) enclosed by the surface. In SI units, it is equal to encl . The net charge
0
is the algebraic sum (sum with sign of the charge included) of all charges
enclosed within the Gaussian surface.
20
Unit 1 Electrostatics in Free Space
Mathematically, we write Gauss’s law as:
  Q
E  
E . dS  encl
0
(1.22a)
S

where  E is the electric flux through a closed surfaces S. It is defined as: The word flux has its
  origins in the old
E  
E . dS (1.22b) French word ‘flus’ and
the Latin word ‘fluxus’
S
both meaning ‘flowing’
In your UG courses you have learnt how to determine electric flux through or ‘to flow’. When we
various surfaces. In general, electric flux can be determined for both closed say that something is in
and open surfaces. The electric flux through any surface S is defined as: the state of flux, we
  mean that it is
E   E . dS (1.22c) changing.
S

Here, we take a couple of examples to illustrate the calculation of electric flux.


You may like to revise the details of this concept from Sec. 6.2 of Unit 6, Vol.1,
BPHCT-133.

Example 1.2
y

Determine the electric flux due to Top face


Right
a) a point charge q through the surface of the sphere of radius R with centre face
at the charge;
b) acube of side 1.0 m is kept in an electric field (in units of N C1) given by
x
z
E  5.0 x iˆ  8.0 ˆj as shown in Fig. 1.10. Determine the electric flux
through the right and top faces of the cube. x  1.0 m x  2.0 m
Solution : a) We use Eq. (1.22a) to determine the electric flux through the
Fig. 1.10: Diagram for
surface of a sphere (of radius R) enclosing the charge q. Thus,
Example 1.2b.
 
E  E . dS
S
E
where S is the surface of a sphere of radius R enclosing the charge q,
which is kept at its centre. You know that the electric field of the charge q
dS
at a point on the surface of the sphere is given as:
ˆ
 1 q rˆ  r
E  rˆ q R
4 0 R 2
where rˆ is the unit vector along the radial direction. Now, for a sphere, the
 S
direction of the area vector dS is along the outward normal to its surface
at all points on the surface. From Fig. 1.11 (showing one such point), you Fig. 1.11: Calculation of
can see that it is along the vector rˆ. Thus, we have the electric flux through a
   spherical surface
dS  dS rˆ and E . dS  E dS rˆ .rˆ  E dS enclosing charge q.

The electric flux of the point charge through the sphere’s surface is then
  q  q 
dS     4 R 2  q
E   E . dS 
4 0R 2
  4 R 
2 0
S  0 
(1.23) 21
Block 1 Electrostatics
b) We use Eq. (1.22c) here and obtain the electric flux, i.e., by integrating the
 
scalar product E . dS over the right and top faces of the cube. Refer to
Fig. 1.12. For the choice of the coordinate axes, the area vector for the
 
right face is dS  dS iˆ and for the top face, it is dS  dS ĵ . So, the
electric flux through the right face of the cube is given as:
 
 
 E  E . dS  (5.0 x î  8.0 ĵ ) . dS î
S S

  (5.0 x ) î . î dS  (5.0)  ( x ) dS ( iˆ . iˆ  1, iˆ . ˆj  0)
S S

y Note that on the right face of the cube, x is constant and has the value
dS  dSjˆ x = 2.0 m. Therefore, for the right face of the cube, we get

 E  (5.0)  ( 2.0) dS  (10.0)  dS


dS  dSiˆ S S

z
x Also, the integral  dS is equal to the area of the right face of the cube,
S
2
which is just 1.0 m . Therefore, the electric flux through the right face of
x  1.0 m x  2.0 m
the cube is
Fig. 1.12: Diagram for  E  (10.0) NC 1 m2
Example 1.2b.
Now, we follow the same steps for the top face of the cube as we followed

for the right face of the cube. Since for the top face, dS  dS jˆ and
jˆ . jˆ  1, iˆ . ˆj  0, the electric flux through the top face of the cube is
given as:
 
E   E . dS   (5.0 x iˆ  8.0 jˆ) . dS ˆj
S S
  (8.0) dS  (8.0)  dS
S S
The integral  dS is equal to the area of the top face of the cube, which is
S
2
just 1.0 m . Therefore, the electric flux through the top face of the cube is
E  (8.0)NC1 m2 .

With this brief revision of the concept of electric flux, let us now return to
Gauss’s law.
Note that Gauss’s law applies to any arbitrary surface enclosing a charge
or charge distribution. However, it is extremely useful for calculating the
electric fields due to symmetric charge distributions as we will illustrate in
the next section.
You know that any imaginary surface enclosing a charge or a charge
distribution is called a Gaussian surface. The Gaussian surface for any
problem is chosen in a way that the calculations become easier.
Eq. (1.22a) is the integral form of Gauss’s law. We can write Gauss’s law in its
differential form, which is easier to use in many cases.
22
Unit 1 Electrostatics in Free Space
Gauss’s Law in Differential Form
We use the divergence theorem, which you have studied in your UG course.
We can write the charge enclosed by a surface in terms of the volume charge
density  and substitute it in Eq. (1.22a) as follows:
Qencl    dV (1.24a)
V
  1
and  E . dS 
0   dV (1.24b)
S V
Using the divergence theorem, we get:

 E . dS    . E dV (1.24c)
S V
 
We substitute the value of  E . dS from Eq. (1.24c) in the left hand side of
S
Eq. (1.24b) and write:
  1

 . E dV 
0   dV (1.24d)
V V

Since Eq. (1.24d) holds for any volume, the integrands must be equal and we
get:
1
 .E   (1.25)
0

Eq. (1.25) gives Gauss’s law in its differential form.


Note that we have expressed it only for volume charge density. Since the
integral form of Gauss’s law can be applied to point, line, surface and
volume charges, it has wider use.
In the next section, we consider some applications of Gauss’s law to
spherically symmetric systems. But before studying further, you may like to
understand the following aspects of Gauss’s law. Then you can solve an SAQ
to check your understanding.

 In Eq. (1.22a), Qencl is the net charge enclosed by the surface


taking into account the algebraic sign of the charges (in case of
many charges). So, if a surface encloses equal and opposite charges,
the net electric flux through it is zero.
 From the statement of Gauss’s law, it is clear that the charges lying
outside the closed surface are not included in Qencl .
 If the closed surface does not enclose any net charge, or if all charges
lie outside the closed surface, then the electric flux through the surface
is zero. This implies that the electric field through such a surface is
zero.
 We can calculate the net charge enclosed inside any closed surface
using this law if we know the net electric flux through the surface
enclosing the charges.
Contd…
23
Block 1 Electrostatics

 The form and location of the charges inside the closed surface do not
matter in the calculations. What matters is the total charge enclosed
by the closed surface and its sign. This very fact makes the
calculation of electric fields using the Gauss’s law far easier in
comparison with Coulomb’s law.
 Gauss’s law essentially follows from Coulomb’s law and the principle of
superposition. It contains no additional information that was not already
present in Coulomb’s law. The law follows from the inverse square
nature of the electrostatic force. Without that, the cancellation of r 2
would not take place. Then the total flux would also depend on the
surface chosen and not only upon the charge enclosed.

SAQ 3
a) The electric flux through a closed spherical Gaussian surface of radius
0.5 m surrounding a charged particle is equal to 500 N m2 C1. Determine
the value of the charge on the particle. If the radius of the surface were to
be halved, what would the value of the electric flux through it be?
b) Suppose that a Gaussian surface encloses zero net charge. i) Does Gauss’s
law require that the electric field be zero for all points on the surface? ii) If the
electric field is zero everywhere on the Gaussian surface, does Gauss’s law
require that the net charge inside the surface be zero?
c) Is Gauss’s law useful in calculating the electric field due to three equal
charges placed at the corners of an equilateral triangle? Explain.

You know that we can use Gauss’s law to calculate the electric fields due to
symmetric charge distributions in a much simpler way. In the next section we
briefly discuss some applications of Gauss’s law that show how it is a powerful
tool for determining electric fields of symmetric continuous charge
distributions.

1.5 APPLICATIONS OF GAUSS’S LAW


Recall that
Symmetry in physics essentially means that a system or an object
remains unchanged (or invariant) under some transformation. Thus,
Symmetric charge distributions are arrangements of charges that remain
unchanged (or invariant) or appear the same after a transformation, for
example, after translation along some axis, reflection or rotation about some
axis.
You know several examples of symmetric objects, e.g., a straight line, square,
plane, sphere, cylinder, etc. Due to the symmetries of charge distributions, the
calculations of electric flux and electric fields due to them become far easier.
We will consider one application each of Gauss’s law to charge distributions
having any one of the following symmetries:
1. Spherical symmetry
2. Cylindrical symmetry
24 3. Planar symmetry
Unit 1 Electrostatics in Free Space
Let us apply Gauss’s law for each of these symmetries.

1.5.1 Spherical Symmetry


Recall from your UG courses that

A charge distribution is said to possess spherical symmetry if it remains


invariant (the same)

 when it is rotated around any axis passing through its centre. It is said
to possess rotational symmetry about that axis;
 when it is reflected across any plane passing through its centre. This
is the reflection symmetry.
For such spherically symmetric charge distributions, we choose a spherical
Gaussian surface. For a point charge, the centre of the Gaussian surface lies
at the position of the charge. For a spherical charge distribution or a spherical
shell, the Gaussian surfaces are concentric with them. You must always
remember the following for any spherically symmetric charge distribution:

 The electric field due to the spherical charge distribution is directed


radially. It points outward from the centre of the sphere for positive
charge and inward for negative charge.

 The magnitude of the electric field at any point depends only on the
distance r of the point from the centre of the charge distribution.

Let us now apply Gauss’s law to determine the electric field due to a point
charge.

Example 1.3
Determine the electric field due to:
a) a positive point charge q, and
b) a spherical charge distribution having uniform volume charge density.

Solution : a) For a point charge: Qencl  q in Eq. (1.22a). To determine the


electric field due to q at point P situated at a distance r from it, we draw a
spherical Gaussian surface of radius r passing through the point with the P
Q
charge at the centre of the sphere (Fig. 1.13). Now, you have learnt that for r
spherical symmetry, the electric field points radially outwards for a positive q
charge, i.e., the direction of the electric field is normal to the sphere’s
 S
surface. The area vector dS for any surface area element of the sphere is

also normal to its surface. So, it is parallel to the electric field E and
 
E . dS  E dS. Then Gauss’s law becomes
Fig. 1.13: Spherical
  q Gaussian surface S for
 E .dS   E dS  0 determining electric field
S S due to a positive point
charge.
Due to spherical symmetry, the magnitude of the electric field due to the
charge would be the same for all points on the spherical surface and we 25
Block 1 Electrostatics
can take it to be constant for S. So, we can take E out of the integral and
write
q
 E dS  E  dS 
0
S S
The result for the So, the integral is just the area of the spherical surface, i.e., it
electric field due to a
charge distribution will is 4r 2. Thus,
be the same whether q
E 4r 2 
we use Gauss’s law or 0
Coulomb’s law to 
1 q 1 q
calculate it. The only or E  and E rˆ (1.26)
difference between the 4 0 r 2 4 0 r 2
two laws is this: It is Did you notice that Eq. (1.26) is the same as Eq. (1.6a) that was obtained
easier to use Coulomb’s
from Coulomb’s law? This means that Gauss’s law and Coulomb’s law
law for a charge
distribution having many give us the same result for the electric field due to a point charge. Gauss’s
discrete point charges. law is equally true for a distribution of charges (read the margin remark).
But it is far easier to use b) You know that a spherical charge distribution possesses spherical
Gauss’s law if the symmetry. Let the spherical charge distribution be spread over a sphere
charge distributions are
of radius R carrying total positive charge Q (Fig. 1.14). A uniform volume
continuous and
symmetric. charge density implies that  is constant. We draw a spherical Gaussian
surface S of radius r through the point P. Since the point P lies outside the
sphere, r  R and Qencl  Q. From Gauss’s law [Eq. (1.22a)], the electric
field due to this charge distribution at a point P outside it, at a distance r
from the centre of the sphere is:
  Q
 E . dS 
0
(1.27)
S
P
Due to spherical symmetry, the magnitude of the electric field is the same
R r
on all points on the Gaussian surface. So, we can take it to be constant for
Q this Gaussian surface. The direction of the electric field is radially outwards
  
S for the positive charge, i.e., in the same direction as dS. So, E and dS
are parallel and
 
E . dS  E dS (1.28a)
Fig. 1.14: Electric field
Since E (the magnitude of the electric field on the Gaussian surface) is
due to a spherical
charge distribution constant, we can pull it out of the surface integral. Therefore, Eq. (1.28a)
having uniform volume becomes
charge density and   Q
carrying total charge Q  
E . dS  E dS  E 4  r 2 
0
(1.28b)
at a point P outside the S S
sphere. 1 Q
or E  for r  R (1.28c)
4  0 r 2
The electric field is given by
 1 Q
E rˆ for r  R (1.29)
4  0 r 2
Notice that we have included the points lying on the surface of the
spherical charge distribution in the result because the Gaussian sphere of
radius R would enclose the entire charge.
26
Unit 1 Electrostatics in Free Space
Note that the electric field given by Eq. (1.29) is the same as that due to a
point charge [given by Eq. (1.26)]. It is as if the entire charge within the
spherical surface is concentrated at the centre of the sphere. Note that this
result is a consequence of spherical symmetry. So, a uniformly charged
sphere would exert the same force on a charge placed anywhere outside it as
an equivalent single charge would.

The electric field due to a uniformly charged sphere and the


electrostatic force exerted by it on a charge situated outside the
sphere are the same as the electric field and electrostatic force due
to a point charge (equal to the charge of the sphere) situated at its
centre.

You may like to determine the electric field at a point inside the spherical
charge distribution carrying net charge Q. Solve SAQ 4.

SAQ 4
Determine the electric field at a point inside the spherical charge distribution
of radius R carrying net charge Q.

Let us now apply Gauss’s law to charge distributions possessing cylindrical


symmetry.
1.5.2 Cylindrical Symmetry
A charge distribution (or any object) is said to possess cylindrical symmetry
if it remains unchanged (or is invariant) when it is

 moved along its axis (AB in Fig. 1.15a and CD in Fig. 1.15b), that is, the line A C

running through its core (translational symmetry);


 rotated around its axis (rotational symmetry);
R S
 rotated by 180° around any axis perpendicular to its axis, (PQ in Fig. 1.15a P Q
and RS in Fig. 1.15b), (180° rotational symmetry);
 reflected across any plane passing through its axis (reflection symmetry);
 reflected across any plane perpendicular to its axis (reflection symmetry). B D
(a) (b)
An infinite line or wire (like the axis of an infinite cylinder) also possesses
cylindrical symmetry (Fig. 1.15b).
Fig. 1.15: a) The axis AB
Let us now answer the second question and explain how Gauss’s law is (dotted line) of a cylinder;
useful for determining the electric field due to a cylindrically symmetric b) for a line or a wire, the
charge distribution. axis CD lies on the
line/wire itself.
While studying Sec. 1.5.1, you would have noted that due to the choice of the
spherical Gaussian surface enclosing the charge distribution, the calculations
became very simple for two reasons:
 the electric field was directed parallel to the area vector for a surface element
 
on the Gaussian surface so that E . dS  E dS; and
 the magnitude E of the electric field was the same at all points on the
Gaussian surface so that it could be treated as constant and taken out of the
surface integral. 27
Block 1 Electrostatics
The direction of the electric field at any point due to a cylindrical charge
++
E distribution is perpendicular to the cylinder’s axis, and the electric field is directed
++ radially outward from the axis for positively charged cylinder (Fig. 1.16). For a
+ + Electric field negative cylindrical charge distribution, it will be directed radially inward and
perpendicular
++ perpendicular to its axis.
to the
+ +
cylinder’s axis The magnitude of the electric field of a charge distribution having
++
cylindrical symmetry depends only on the perpendicular distance, say r, of the
Fig. 1.16: The direction
point from the cylinder’s axis. So, all points on the cylindrical surface of a given
 radius are equivalent as far as the magnitude of the electric field of any
of electric field E due
to a section of an cylindrical charge distribution is concerned: it could be a line charge, charged
infinite charged wire or charged solid/hollow cylinder. Then we can treat the magnitude of the
cylinder is electric field of such systems at a given cylindrical surface as constant and take it
perpendicular to its out of the surface integral. You will appreciate this point better in the next section.
axis.
Always remember the following for any charge distribution having cylindrical
symmetry:

 The electric field due to a charge distribution having cylindrical


symmetry is directed perpendicular to its axis of symmetry.
 The magnitude of the electric field at any point depends only on
its perpendicular distance from the axis of symmetry.

So, the Gaussian surface is cylindrical for a cylindrically symmetric charge


distribution. For a cylindrical Gaussian surface coaxial with the cylindrical charge
distribution (charged line or cylinder), the electric field is normal to the surface at
all points on it. Since for any area element centred at a point on the Gaussian

P surface, the area vector dS is directed normal to the surface (Fig. 1.17),

dS therefore, the electric field E at any point due to this charge distribution is

parallel to the area vector dS and hence,
 
E . dS  E dS (1.30)
Fig. 1.17: Area vector
 Also, the magnitude of the electric field at any point is the same everywhere on
dS for an element of
area centred at any the cylindrical Gaussian surface passing through that point. So we can treat it as
point P on a constant for that surface and take it out of the surface integral.
cylindrical Gaussian
With this understanding of cylindrical symmetry of charge distributions, let us
surface is normal to
the surface. apply Gauss’s law to one such system.

Example 1.4
Determine the electric field due to a uniform infinite line charge.

Solution : Consider an infinitely long wire carrying uniform linear charge


density . Let us apply Gauss’s law to determine the electric field at a distance
r from the wire. You know that an infinite line charge distribution has cylindrical
symmetry. The Gaussian surface is a surface of a right circular cylinder of
radius r and length L coaxial with the wire (Fig. 1.18).
Due to cylindrical symmetry, the magnitude of the electric field is the same for all
points on the cylindrical surface of radius r and can be treated as constant for that
particular surface. The direction of the electric field is normal to the surface at all
28
Unit 1 Electrostatics in Free Space

E 
E

S dS

dS
+ + + + + + + + + + + + + + + + + + + + + + + +

Fig. 1.18: Applying Gauss’s law to an infinite uniformly charged wire carrying
positive charge. The Gaussian surface is cylindrical having length L and
radius r. It encloses a section of the charged wire.

points as shown in Fig. 1.18. For positively charged wire, the electric field is
directed radially outwards from the wire’s axis. If the charge on the wire were 
negative, the electric field would point inwards towards the wire’s axis. Since E

and dS are parallel to each other for each area element on the curved part of the
cylinder’s surface, we can write:
 
E . dS  E dS (1.31a)
The electric flux at all points through both circular ends of the cylinder is zero
 
because E and dS are perpendicular to each other on these ends (Fig. 1.18).
 
Therefore, the product E . dS is finite only for the curved part of the cylindrical
surface. Thus, from Gauss’s law, we have
Qencl
 E . dS   E dS  E  dS  E 2rL 
0
(1.31b)
S S S

where we have taken E out of the integral as it is constant on this Gaussian


surface S. In Eq. (1.31b), we have also used the result that the area of the curved
surface of a cylinder of radius r and length L is 2rL . So from Eq. (1.31b), we
have Note that the electric
Qencl
E 2rL  field in Eq. (1.32)
0 does not depend on
Qencl the length of the
or E  (1.31c) cylindrical Gaussian
2 0 r L
surface.
For the uniform line charge density , the charge enclosed by the cylinder of
length L is given by In applying Gauss’s
L L law, the choice of
Qencl    d l     dl    L, since  is constant (1.31d) the Gaussian
0 0 surface is very
important for
Substituting Eq. (1.31d) in Eq. (1.31c), we get: simplifying
Qencl L calculations. This is
E  especially true for
2 0 r L 2 0 r L
symmetric charge
 distributions. You
or E (1.32) will appreciate this
2 0 r
point time and again
The electric field is directed perpendicular to the line charge or charged in this unit.
wire.
29
Block 1 Electrostatics
You should, however, note that Gauss’s law is always true, no matter what
the distribution of charges. But it is very useful for symmetric charge
distributions since its application makes the calculation much simpler.
You may like to know: Why do charge distributions have to be symmetric
for Gauss’s law to be applied to determine electric fields?
This is because the symmetry of the distribution helps us determine the
surfaces over which the magnitude of the electric field is constant (i.e., the
distance r is constant). Also, we know the direction of the electric field for a
given type of symmetry.
Then the trick is to choose the Gaussian surface to be the surface over
which the magnitude of the electric field is constant. Also, the direction
of the electric field should be parallel/perpendicular to the area vector at
all points on the surface.
Suppose, we choose some other shape for the Gaussian surface, then
 
Gauss’s law would still apply but E may not be in the same direction as dS
or perpendicular to it and its magnitude may not be constant over the surface.
Then we would not be able to take E out of the integral. That would make the
calculation difficult. So, symmetry is important for such applications of
Gauss’s law. You must have appreciated this point by now having studied
charge distributions possessing spherical and cylindrical symmetry. We end
this section with an SAQ for you.

SAQ 5
Determine the electric field at a point outside an infinitely long charged solid
cylinder of radius R, which has uniform volume charge density .

Let us now ask: What is the electric field of an infinite uniformly charged
cylinder at a point inside it? Go through Example 1.5 for the answer.
Example 1.5
An infinitely long uniformly charged cylinder of radius R has positive volume
P E charge density . Determine the electric field at a point inside the cylinder.
L
r Solution : We use Gauss’s law to obtain the electric field at a point P inside
the cylinder at a distance r from its axis. Since the charge distribution is
cylindrically symmetric, we draw a cylindrical Gaussian surface of length L
R
and radius r passing through P (Fig. 1.19). For any point inside the cylinder, r
 R and the Gaussian surface lies inside the cylinder. From symmetry
Fig. 1.19: Electric
field inside an
considerations that you have learnt in this section for cylindrical charge
infinite uniformly distributions, you know that the electric flux has contribution only from the
charged cylinder. The curved surface of the Gaussian cylinder and not its ends. Hence, from Gauss’s
Gaussian surface is a law, we have
cylindrical surface of Qencl
length L and radius  E . dS  E 2 r L  0 for r  R (i)
r  R. S
The charge enclosed by this Gaussian surface is:

Qencl    dV
30 V
Unit 1 Electrostatics in Free Space
where the volume is just the volume of the cylinder of length L and radius r.
Therefore,
Qencl   r 2 L

Qencl  r 2 L
and from Eq. (i), E 2 r L  
0 0

r
 E  for r  R
20
 r
and E  rˆ for r  R (1.33)
20

where rˆ is the unit vector in the radial direction pointing outward from the
cylinder’s axis. So, inside the cylindrical charge distribution, the electric field
increases linearly with an increase in the distance from the axis.

For an infinite uniformly charged cylinder, always remember the following:

 The electric field of an infinite uniformly charged cylinder at points


outside it decreases with an increase in distance from its axis.
 The electric field inside an infinite uniformly charged cylinder
increases linearly with an increase in distance from its axis.

Inthe next section, we will apply Gauss’s law to a charge distribution having
planar symmetry. Examples of such charge distributions are uniform
two-dimensional sheets of charge, thin plate carrying charge or uniform slabs
of charge as well as combinations of such sheets or slabs like the ones used
in parallel plate capacitors.
1.5.3 Planar Symmetry
In this section, we apply Gauss’s law to an infinite uniformly charged plane
sheet carrying a constant surface charge density . A large plastic sheet
uniformly charged on one side is an example of a non-conducting sheet of
charge. An aluminium foil is an example of a conducting sheet.
Let us first answer this question: What kind of symmetry does an infinite
sheet (planar charge distribution) possess? It remains the same if it is
 translated parallel to itself,
 rotated about any axis perpendicular to its plane, and
 reflected about any axis lying in its plane or perpendicular to its plane.
It follows from the symmetry considerations for a sheet of charge that the
electric field due to it is everywhere perpendicular to the plane of the sheet. It
is directed outward from the sheet, if positively charged and inward, if
negatively charged.
Let us now determine the electric field due to the infinite uniformly charged
sheet at a distance r from it. Let its surface charge density be . Here we
assume that the thickness of the sheet is much less than r. Now to use
Gauss’s law meaningfully, we need to choose a Gaussian surface that exploits
the fact that the electric field is directed normal to the charged sheet. What is 31
Block 1 Electrostatics
that Gaussian surface? We choose a closed cylindrical Gaussian surface
perpendicular to the sheet with each end of the cylinder located at an equal
distance (r) from the sheet. So, the length of the Gaussian cylindrical surface
is 2r (see Fig. 1.20a). Such a Gaussian surface is also called the Gaussian
‘pillbox’. In Fig. 1.20b, we show the side view of the sheet and the pillbox. Let
the area of cross-section of the Gaussian pillbox (i.e., the area of its ends) be
S.
+
+ + +
+ + 
+
+ + + dS
+ 
+ + + + dS+ + + +
+ + + + + + 
+ 
+ +
+ + + dS dS
+ + + + +  +
+ + + + dS  2r 
+ + + +
S+ r + +r + +  E E
+ + S + + +
+ + + + + E
+ +
+ + +  +
+ + + 
+ + d S + + dS
+ + + + +
+ +
+ +
+
+
+
(a) (b)

Fig. 1.20: a) A sheet of positive charge and the Gaussian pillbox for which the
 
electric field E and area vector dS are parallel at the ends and
perpendicular to each other on the curved part of the surface; b) the
sheet in its side view showing the electric field vectors and area vectors
for the pillbox.

Since the charge is positive, the electric field is directed away from the sheet
and is perpendicular to the sheet. This means that for the curved part of the
cylindrical Gaussian surface, the electric field vector is perpendicular to the
area vector at all points (see Fig. 1.20b). Thus,
 
E . dS  0 for all points on the curved part of the cylindrical surface
The electric field vectors point in an outward direction from the two ends of the
Gaussian pillbox, i.e., in the same direction as the area vectors for the ends.
So, the contribution to the electric flux is only from the ends of the Gaussian
pillbox and
 
E . dS  E dS for all points on one end of the cylindrical surface
Since there are two ends on the Gaussian pillbox, we need to consider the
surfaces of both ends while applying Gauss’s law and divide the surface
integral into three parts corresponding to the two ends and the curved part.
Then Gauss’s law gives us
      Q
 E .dS  
E .dS  
E . d S  0  E S  E S  encl
0
S Curved Both ends
part
(1.34a)
Q
or E  encl (1.34b)
20 S

Now, we need to express the charge on the sheet enclosed by the Gaussian
32 cylinder in terms of the uniform surface charge density . This is just the
Unit 1 Electrostatics in Free Space
charge enclosed by the area of the sheet equal to the cylinder’s cross-section,
i.e., the area S. Since  is uniform (i.e., constant), it is equal to the ratio of the
charge on a given surface to its area. Therefore, for the charge Qencl
enclosed by the area S, it is
Q
  encl  Qencl   S (1.35)
S
Substituting the value of Qencl from Eq. (1.35) in Eq. (1.34b), we get


E (1.36)
20

where the direction of the electric field is perpendicular to the sheet.


Eq. (1.36) holds for both non-conducting and conducting sheets of
charge provided the layer of charge on the sheet is very thin (or its
thickness is very small compared to the distance at which the electric
field is being calculated). It also holds for very large sheets of charge at
points far from the edges of the sheet and at distances much larger than the
thickness of the sheet or the layer of charge on the sheet. Eq. (1.36) tells us
that

The electric field due to an infinite (or very large) uniformly


charged sheet has the same value at all points lying outside it
and points in a direction perpendicular to the sheet.

Let us apply Gauss’s law to two infinite or large sheets of charge in Example 1.6.

Example 1.6
1 + 2
Two thin infinite non-conducting charged sheets are kept parallel to each 
other as shown in Fig. 1.21. The surface charge density of the negatively  +
charged left sheet is 1 and that of the right sheet carrying a positive charge  +
is  2 . Determine the net electric field in the region (1) to the left of the sheets,  +
(2) between the sheets and (3) to the right of the sheets.  +
(1)  (2) + (3)
Solution : We apply Gauss’s law to both sheets using the result obtained for
an infinite uniformly charged sheet. We use the fact that the charges are fixed
Fig. 1.21: Diagram
and obtain the electric field due to each sheet as if it were isolated. Then we for Example 1.6.
apply the principle of superposition to obtain the net electric field.

Remember that from Eq. (1.36), the magnitude of the electric field at any point
does not depend on the distance of the point from the sheet. It depends only
on the surface charge density. The directions of the electric fields depend on
the sign of the charge carried by them. The magnitudes of the electric field
due to the negatively and positively charged sheets having surface charge
densities 1 and  2 , respectively, are given by
1 2
E  and E 
20 20

Fig. 1.22a shows the directions of the electric fields in each region. Note that
the electric field due to the positively charged sheet points away from it in
each of the three regions. The electric field due to the negatively charged
33
Block 1 Electrostatics
sheet points towards it in each region. Let us denote the unit vector to the right
of the sheets by iˆ (Fig. 1.22b).
1 2 1
 +
E(  )  + 2
E(  )

E(  ) iˆ
+  +
 +  +
E(  )  E(  ) + E(  ) E1  E2 E3
+
 +  +
(1)  (2) + (3) (1)  (2) + (3)

(a) (b)

Fig. 1.22: Diagram for solution of Example 1.6.

Then the resultant electric field in each of these regions (Fig. 1.22b) is given by:
1
a) Region (1): E1  E  E  (E )(  iˆ)  (E )iˆ  (1  2 ) iˆ
20

1
b) Region (2): E2  E  E  (E  E )(  iˆ)   (1  2 ) iˆ
20

1
c) Region (3): E3  E  E  (E ) iˆ  (E )(  iˆ)  (2  1) iˆ
20

Such calculations are important in determining the electric fields of parallel plate
capacitors which, as you know are very useful in our daily lives. You may now
like to attempt an SAQ.

SAQ 6
Suppose in Example 1.6, the surface charge density of the negatively charged
sheet is 1  9.0  109 Cm2 and that of the positively charged sheet is
2  6.0  109 Cm2 . Determine the electric fields in the three regions.

With this application of Gauss’s law, we end the discussion on planar symmetry
and summarise the contents of this unit.

1.6 SUMMARY
In this unit, we have discussed the following concepts:
 Electrostatic force field, electric field and superposition principle
for a system of discrete charges and continuous charge distributions at
rest in free space.
 Electric potential for a system of discrete charges and continuous
charge distributions at rest in free space.
 Electric flux through a surface and Gauss’s law which relates the net
electric flux through any imaginary closed surface S of arbitrary
shape (called the Gaussian surface) to the net charge (Qencl )
enclosed by the surface.
 Applications of Gauss’s law for the determination of electric field due
to a point charge, distribution of discrete charges and continuous
charge distributions enclosed by arbitrary surfaces. Especially,
34
Unit 1 Electrostatics in Free Space
applications of Gauss’s law to determine of electric field due to
symmetric charge distributions having
 spherical symmetry such as a uniformly charged sphere;
 cylindrical symmetry such as an infinite, wire and uniformly
charged infinite cylinder; and
 planar symmetry such as a uniformly charged thin sheet.
1.7 TERMINAL QUESTIONS
1. Four charges  2q,  2q,  2q and  2q are placed at the vertices of a
rectangle of sides 3.0 m and 4.0 m. What is the net electric field due to the
charges at the point of intersection of the diagonals given that
q  3.0 109 C?
2. A uniform electric field of 3  103 NC 1 is in the positive x-direction. A
positive point charge 2 C is released from rest at the origin. Calculate the
potential difference V (5 m) – V (0).
3. Derive the expression for the electric potential due to an infinite uniformly B
charged wire. A

4. Derive the expressions for the electric potentials at points outside and inside
due to a) uniformly charged spherical shell and b) uniformly charged sphere. C
 q
5. Show that the line integral of the electric field E over a closed path is equal
D
to zero.
6. A charge q is placed at a corner of a cube as shown in Fig. 1.23. Fig. 1.23: Diagram
Determine the flux of the electric field of the charge through the right face for TQ 6.
(ABCD) of the cube? (Hint: Solving this problem requires a clever choice of
the Gaussian surface.) Solid
copper
7. A non-conducting thin spherical shell of radius R carries total positive wire
charge Q that is distributed uniformly over its surface. Determine the
electric field due to this shell at points lying outside and inside it.
Braided
8. A coaxial cable consists of a thin inner solid copper wire and an outer wire
sheath of braided copper wire (see Fig. 1.24). The linear charge density Plastic sheath
covering
of the inner wire is  and that of the outer wire is . Determine the electric
fields at a point (a) in the region inside the inner wire, (b) in the region Fig. 1.24: Diagram for
between the wires and (c) in the region outside the coaxial cable. TQ 8.

9. Two identical infinite non-conducting sheets having equal positive surface


charge densities  are kept parallel to each other as shown in Fig. 1.25. I
Determine the electric field at a point in (a) region I above the sheets,
(b) region II between the sheets and (c) region III below the sheets. II
10. A Gaussian surface of cylindrical shape (of radius 1.0 m and height 20 m)
encloses a few positive charges. Assuming that the electric field due to III
these charges is normal to the Gaussian surface and has magnitude Fig. 1.25: Diagram for
900 NC 1, calculate the volume charge density of the charge distribution. TQ 9.

1.8 SOLUTIONS AND ANSWERS


Self-Assessment Questions
1. a) From Eq. (1.3), the electrostatic force on charge q1 due to charge q 2
 1 q1q2
is given by Coulomb’s law as F21  rˆ21 where rˆ21 is the
4 0 r21 2 35
Block 1 Electrostatics
unit vector along the line joining the particles and directed from q 2 to
 1
q1 and r21  r . Also  8.99 109 N m2 C2 .
4 0

Substituting the values of q1, q2 and r for both cases, we get


 (8.0  C) (8.0  C)
i) F21  8.99 109 N m2 C 2  rˆ21  360 N rˆ21
(0.04 m) 2
 (15 m C) ( 10 m C)
ii) F21  8.99 109 N m2 C 2  rˆ21
(3.0 m) 2
 1.5  105 N rˆ21

b) We use the principle of superposition given by Eq. (1.4b) for a system


of three charges. Since rˆ21  iˆ and rˆ31   jˆ, we have
 1  q1 q 2 q q 
F1   (  î )  1 3 (  ĵ ) (i)
4 0  (r21 ) 2 (r31 ) 2 

where iˆ and ĵ are unit vectors along the x and y-axes (Fig.1.2).
Substituting all numerical values (with the sign of the charges) in
Eq. (i), we get

 ( 2.0  10 6 C)  (9.0  10 6 C) ˆ 
 ( i ) 
  (0.3 m) 2 
F1  (8.99  10 9 N m 2 C  2 )  
 ( 2.0  10  6 C)  (16.0  10  6 C) 
 (  ˆj )
 (0.4 m) 2 

 (ii)
or  ˆ ˆ
F1  1.8 i  1.8 j N 
The magnitude of the force is (1.8)2  (1.8)2 N  2.5 N

The direction of the force is given by the angle  it makes with the
q q  1.8 
C r P positive x-axis:   tan1  1 
  tan (1)  45 .
 1.8 
d
r 2. a) From Eq. (1.9), we determine the electric field due to each charge at
Fig. 1.26: An electric the point P and then use Eq. (1.10). From Eq. (1.9), the electric fields
dipole made up of equal due to both charges at the point P are, respectively,
and opposite charges, ( q ) rˆ  ( q ) rˆ
 q, separated by Eq  and E q 
40 (r  d )2 4 0 (r  d )2
distance 2d. The
vector 2d along the axis Here rˆ is the unit vector pointing from the charge  q to the charge
of the dipole is drawn  q along the line joining them and d is the distance of the midpoint
from the negative to the
from each charge (see Fig. 1.26). From Eq. (1.10), the resultant or net
positive charge. The
electric field at the point P due to the two charges is:
point P lies on the
   q rˆ  4rd 
dipole axis at a distance E  E  q  E q   2 
r from the midpoint C. 4 0  (r  d ) 
2 2

If we assume that the point P lies far away from the dipole so that
r  d, we can neglect the term d 2 in comparison to r 2 in the
36
Unit 1 Electrostatics in Free Space

denominator of the expression for E . Under this assumption, the net
electric field at P is
 
1 q rˆ( 4rd ) 1 2p
E   (i)
4 0 r4 4 0 r 3
 
where p  2qdrˆ (  2qd ) is a vector quantity called dipole moment.

b) Again we use Eq. (1.9) to determine the electric field due to each
charge at point P and then apply Eq. (1.10). The distance of the point P
from both the charges  q and  q is (d 2  r 2 (Fig. (1.27) and
therefore, the magnitudes of the electric fields at P due to these E q
charges are equal and, respectively, given by:

P
1 q 1 q 
E q  and E q 
4 0 d 2  r 2 4 0 d 2  r 2 E q
r
From Fig. 1.27a, you can see that the direction of the field is away from
the charge  q and towards the charge  q. To obtain the expression q
  q
for the resultant field at P, we take the vector sum of the two electric d
fields using the parallelogram law of vector addition. From 2d
Fig. 1.27a, note that the angle between the two electric field vectors is (a)
2. So, we obtain the magnitude and direction of the resultant electric
field as follows: E q

1 2q
E  E2q  E2q  2EqEq cos 2  cos  E P
4 0 d  r 2
2 

1 2qd d
or E since cos   E q
4 0 (d  r 2 )3 / 2
2
d2  r 2 (b)
The directionof the resultant electric field is given by the angle  it
makes with E  q (Fig. 1.9b): Fig. 1.27: Diagram for
answer to SAQ 2b.
 Eq sin 2 
  tan1   tan tan   
1
 Eq  Eq cos 2 

So, angle  is equal to the angle . Note that E is anti-parallel to

p. So, we can express E at point P as
 
p
E
4 0 (r 2  d 2 )3 / 2

If the point P is located far away from the dipole so that r  d, we can
express the electric field due to the electric dipole at the point as
 
p
E
4 0r 3
3. a) From Eq. (1.12a), the value of the charge on the particle is given by

qencl   0  E  (8.85 10 12 C2N1 m 2 )  500 Nm2 C 1


 4.42 109 C

The electric flux through the surface would not change since the net
charge enclosed by it remains the same. 37
Block 1 Electrostatics
b) i) When the Gaussian surface encloses zero net charge, Gauss’s law
 
yields E . dS  0. However, this does not mean that the electric field is
  
zero for all points on the surface. E . dS can be zero even when E and

dS are perpendicular to each other. (ii) If the electric field is zero
everywhere on the Gaussian surface, Gauss’s law requires that there
should be no net charge inside the surface, i.e., the net charge should be
zero.
c) Gauss’s law is not useful in calculating the electric field due to three equal
charges placed at the corners of an equilateral triangle because it is not
possible to find a closed surface of appropriate symmetry over which the
electric field can be taken to be constant and its direction can be taken to
be either parallel or normal to the surface to evaluate the surface integral.
4. We have to determine the electric field at a point inside a spherical charge
distribution carrying net charge Q , i.e., at points for which r  R (see
P
R Fig. 1.28).
r
S For this, we draw a spherical Gaussian surface of radius r  R. We apply
Q Eq. (1.22a) in which Q has to be replaced by the charge (q) enclosed by
the Gaussian sphere of radius r. Since the volume charge density is
uniform for the charged sphere of radius R (i.e.,  is constant) and the
4 3
Fig. 1.28: Determining volume of the spherical charge distribution is R ,  is given by:
3
the electric field of a Q
uniformly charged   (i)
4 3
sphere of radius R R
carrying net charge Q
3
at a point P inside the 4 3
Therefore, the charge enclosed by the Gaussian sphere of volume r
sphere. 3
is the product of its volume with the volume charge density:
4 3 Q  4 3  r3
q   r  3 r   Q (ii)
3 4 3   R3
R
3
S R  
Using Eq. (ii) for q and the result E . dS  E  dS  E 4  r 2 from
S S

L  Eq. (1.28b) in Eq. (1.22a), we have


E
q Q r3
r P E 4 r 2  
0 0 R 3
Q r
or E  for r  R (iii)
4  0 R 3
The electric field at a point inside the uniformly charged sphere is given by
 Q r
Fig. 1.29: Electric field at E rˆ for r  R (iv)
a point P lying outside a 4  0 R 3
uniformly charged 5. We use Gauss’s law to obtain the electric field for the uniformly charged
infinite cylinder. The infinite cylinder at a point P lying outside it at a distance r from its axis.
cylindrical Gaussian
Fig. 1.29 shows a section of the infinite cylinder by a solid line. For a point
surface is of length L
and radius r  R. P outside the cylinder, the Gaussian surface is cylindrical, of length L and
radius r, passing through P. For the curved part of the cylindrical Gaussian
38 surface, the direction of the electric field is normal to the surface at all
Unit 1 Electrostatics in Free Space
points. Also, the electric field is directed radially outwards from the
 
positively charged cylinder’s axis. Therefore, E and dS are parallel to
each other for each area element on the curved part of the Gaussian
 
surface and E . dS  E dS. The electric flux through both circular ends of
 
the cylindrical Gaussian surface is zero because E and dS are
perpendicular to each other at all points on these ends. Therefore, from
Gauss’s law, we have
  Q
  
E . dS  E dS  E dS dS  E 2rL  encl
0
(i)
S S S

Here, since E is the same on all points of the Gaussian surface S, we have
taken it to be constant for the surface and have taken it out of the integral.
In Eq. (i), we have also used the result that the total surface area of a
cylinder of radius r and length L is 2rL. So, from Eq. (i), we have
Qencl
E  for r  R (ii)
2 0 r L

We now have to determine Qencl in Eq. (ii), which is the net charge
enclosed by the cylindrical Gaussian surface, given that  is constant. It is
just the charge on the cylinder of length L and radius R (because the
charge distribution of the infinite cylinder is zero beyond its radius R). By
definition, it is given by the following volume integral:
Qencl    dV (iii)
V

Since  is uniform (constant), we can take it out of the integral and write
Qencl    dV   R 2 L (iv)
V

where the volume integral is just the volume of the cylinder of length L and
radius R. Therefore,
 R 2 L
E for r  R (v)
2 0 r L
  R2
E rˆ for r  R (vi)
20 r

where rˆ is the unit vector in the radial direction pointing outward from the
cylinder’s axis. Notice from Eq. (vi) that the electric field of a cylindrical
charge distribution at points lying outside it decreases as the distance from
the axis increases.
6. As explained in Example 1.6, for 1  9.0  109 Cm2 and
2  6.0  109 Cm2, the magnitudes and directions of the electric fields in
the three regions are given by
1 (9.0  6.0) 109 Cm2 ˆ
Region (1): E1  (1  2 ) iˆ  i
20 2  8.85 1012 C2N1 m2

 1.7 102 NC1 iˆ 39


Block 1 Electrostatics
1 (9.0  6.0) 10 Cm2 ˆ
9
Region (2): E2   (1  2 ) iˆ   i
20 2  8.85 1012 C2N1 m2

  8.5 102 NC1 iˆ


1 (6.0  9.0) 109 Cm2 ˆ
Region (3): E3  (2  1) iˆ  i
20 2  8.85 1012 C2N1 m2
  1.7 102 NC1 iˆ

Terminal Questions
1. Refer to Fig. 1.30 showing the four charges A, B, C, D, viz.  2q,  2q,
 2q and  2q placed at the vertices of a rectangle of sides AB  3.0 m
and BC  4.0 m . The net electric field due to the charges at the point of
y
intersection of the diagonals is the vector sum of the electric fields of the
 2q  2q
D   C respective charges at that point. Let us choose the x- and y-axes as shown
E2 E1 in Fig. 1.30. The length of the diagonal of the rectangle is
 (3.0)2  ( 4.0)2 m  5.0 m . Note from Fig. 1.30 that the electric fields
P 4.0 m
due to the charges placed at the vertices A and C point in the same
 B direction since the charges are unlike. So is the case for the charges
A x
 2q 3.0 m  2q placed at the vertices B and D. The magnitudes of the electric fields due
to all four charges are the same since the magnitudes of the charges are
Fig. 1.30: Diagram for equal and their distances from the point P are equal. Thus, the magnitude
the answer of TQ 1. of the electric field due to each charge is given by
1 2q 6.0  109 C
E   8.99  109 N m2 C 2  8.6 N C1
4 0 r 2 (2.5 m) 2
 
The net electric field is the resultant of the electric fields E1 and E2 shown
in Fig. 1.30 with their tails at the point P. Note that their magnitudes are:
E1  E2  2E

Note also from Fig. 1.30 that the x-components of these electric fields are
equal and opposite so they cancel out. Their y-components are equal in
magnitude and in the same direction and are given by:
BC 4
E1y  E2y  E1 sin   2E  2  8.6 N C1  13.8 N C1
AC 5
So, the magnitude of the net electric field is:
E  E1y  E2y  13.8 N C1  13.8 N C1  28 N C1

upto 2 significant digits. It is directed along the y-axis.


2. From Eq. (1.6b), we have:
B 5m
V (5 m )  V (0)    E.dl    Edl  (3  103 NC1)  (5m )
A 0
3
 15  10 V
3. The electric field at a point near an infinitely long charged wire (or a line
charge) is given by:

E rˆ (i)
40 20r
Unit 1 Electrostatics in Free Space
where  is the charge per unit length on the wire or the linear charge
density, r, the perpendicular distance of the point from the wire,  0 , the
permittivity of free space, and rˆ, the unit vector along the direction of
increasing r from the line charge (Fig. 1.31). The potential of this wire at a
point ‘a’ situated at a perpendicular distance of ra from the wire is:
ra
V    E.dl (ii)

We evaluate the line integral in Eq. (ii) by first moving a unit positive
charge from a finite distance r b instead of infinity, to point a at distance ra
and then let r b go to infinity. Here r b is the distance of point b from the
wire (see Fig. 1.31). On solving this integral, we get the potential difference
between points a and b:
ra rb rb
Va  Vb    E.dl   E.dl   E.dr (iii)
rb ra ra
because for the path a to b, dl is parallel to dr . Inserting the expression
for E from Eq. (i), we get
rb
 ˆ
r.dr
Va  Vb 
2 0  r
ra
Since rˆ and dr are in the same direction, we have
rb
 dr Note that an infinite line
Va  Vb 
2  0  r charge contains infinite
ra amount of charge. Thus,

ln r rrab    ln ra   ln rb    ln ra 
we cannot calculate V at
 a point for such a
2 0 2 0 2 0 2 0  rb 
continuous charge
(iv) distribution by taking total
We can take a point at infinity with zero potential as our reference point charge into consideration.
That method will not work
and calculate the potential at a given point with respect to infinity. If we
as it will give infinite
take point b at infinity, i.e., rb   and take the potential Vb equal to zero, potential everywhere.
then the RHS of Eq. (iv) tells us that the potential Va at point a will be That is why we have used
infinite. This is expected also because the infinitely long line charge having the relation between V
uniform charge distribution means an infinite amount of charge. Therefore, and E and the
the sum of finite contributions from each part or element of an infinite line expression of E for an
charge leads to an infinite potential. Thus, to have a physically meaningful infinite line charge to
expression for potential at a point finite distance away from the line charge, obtain a physically
meaningful expression for
we cannot take infinity with zero potential as our reference point. However,
V.
this does not cause any problem because we are usually interested in
potential difference between two points rather than its absolute value at a
given point. Thus, Eq. (iv) which gives the potential difference between
points a and b (Fig. 1.31) with both ra and r b having finite values meets
our requirement.
4. a) Fig. 1.32 shows a uniformly charged spherical shell of radius R.
To obtain an expression for the potential at an external point P, we first
identify a suitable element of charged shell. The charged surface of the
shell can be considered as a collection of a large number of thin rings
such as the ring AB. The orientation of these rings is so selected that 41
Block 1 Electrostatics

Fig. 1.32: A uniformly charged spherical shell of radius R and point P is an


external point.
the axis of the rings is along OP, the line joining the centre O of the
shell with the point P. Let the ring AB be contained between the
directions  and  + d with respect to the axis OP. Let it be of
infinitesimal width so that every point on it is at the same distance, say
r, from P. The angular width of the ring is d, its width is Rd and its
radius is R sin. The circumference of the ring is 2R sin  and hence,
its area is given by
dA  (2R sin )R d  2R 2 sin d (i)

If the total charge on the shell is Q, then charge per unit area,
  Q /( 4R 2 ). and from Eq. (i), we can write the charge on the ring
as:
Q Q
Qring   (2R 2 sin ) d  sin d (ii)
(a) 4R 2 2
We shall now determine the electric potential at point P due to the ring
AB. The ring is made up of a large number of point charges each
having charge equal to, say Q. So, the electric potential for one such
1 Q
point charge is . So, the electric potential due to the ring will
4 0 r 
be
 Q  1 Qring
  4 0  Q  4 0
1 1
dVring  
(b) r  4 0r 
 r
Fig. 1.33: Diagrams for So, on using Eq. (ii), we get
calculating electric
 1 Q 
potential of charged dVring     sin d  (iii)
spherical shell.  4  r 
0  2 
As we mentioned above, the shell can be imagined to be made of rings
like AB having a common axis OP. Since electric potential is a scalar
quantity, we shall integrate Eq. (iii) to get the electric potential V of the
shell. Note that on the RHS of Eq. (iii), we have two variables  and r.
It will be convenient if we can express it in terms of a single variable.
For this, we shall consider the relation between r, r and R. As you
know from your school and UG courses in Mathematics and Physics,
we can use the triangle law and write:

42 r  2  r 2  R 2  2rR cos 
Unit 1 Electrostatics in Free Space
On differentiating with respect to , we get
dr  dr  sin d
2r   2rR sin  or  (iv)
d rR r
Substituting Eq. (iv) in Eq. (iii), we get
1  Q   dr  
dVring     (v)
4 0  2   rR 

To obtain the electric potential due to the entire shell, we need to


integrate Eq. (v) over appropriate limits of integration to include the
contribution of every ring of the shell:
r2
1 Q

V  dVring 
4 0 2rR
dr   (vi)
r1

where r1 and r2 are, respectively, the minimum and maximum values
of r  . To write the values of r1 and r2 in terms of r and R, we consider
the two cases – point P outside the shell and point P inside the shell –
separately .
Point P outside the shell: In this case, as shown in Fig. 1.33a, the
values of r1 and r2 are:

r1  r  R and r2  r  R

So, Eq. (vi) becomes


(r R )
1 Q 1  Q  Q
V
4 0 2rR  dr   4 0  2rR  [2R ]  4 0r (vii)
( r R )

Eq. (vii) gives the electric potential due to a uniformly charged spherical
shell at a point outside the shell.
Point P inside the shell: Refer to Fig. 1.33b which depicts the point P
inside the shell. From the figure, we have that for r < R
r1  r  R and r2  R  r

We substitute r1 and r2 as limits of integration in Eq. (vi), and get:

(R r )
1 Q 1  Q  Q
V 
4 0 2rR  dr   4 0  2rR  [2r ]  4 0 R (viii)
( R r )

From Eq. (viii), which gives electric potential at an internal point P, we


note that the electric potential is independent of r, the distance of point
P from the centre O of the shell. This means that the electric potential
at every point inside the shell is same and its value is equal to its value
at the surface. From Eq. (viii) and Fig. 1.33, you can see that the
electric field inside the uniformly charged spherical shell is zero.
c) Let  be the volume charge density (charge per unit volume) of a
uniformly charged non-conducting sphere. Let the radius of the sphere
be R (see Fig. 1.34).
43
Block 1 Electrostatics

Fig. 1.34: A uniformly charged non-conducting sphere of radius R with point P1


outside the sphere and point P2 inside the sphere.

Electric potential at a point inside the sphere


Let point P2 be an internal point at a distance r from the centre O such
that r < R (see Fig. 1.34). If we divide the sphere into a large number of
thin concentric shells with centre O, then for shells with radii  r , point P2
is outside and for shells which have radii between r and R, point P2 is
inside. For shells with radii less than or equal to r, potential V1 at P2 can
be written as if point P2 is an external point and hence it is given by:
Q1 4 r 3 r 2
V1     (ii)
4 0 r 3 4 0 r 3 0
To evaluate the contribution to electric potential by the shells for which P2
is inside the sphere, let us consider a shell of radius x and thickness dx as
shown in Fig. 1.34. For this shell, the total charge Q 2 is equal to volume
times charge density, i.e. Q2  4x 2dx. This charge contributes a
constant electric potential dV2 at any internal point and is given by:
4 x 2 dx  x dx
dV2   (iii)
4  0 x 0
For adding the contributions from all such shells for which P2 is an internal
point, we integrate Eq. (iii) for x varying from r to R.
This gives the electric potential V2 at P2 due to shells for which point P2
is internal as:
R R
   R2  r 2 

V2  dV2 
0 
x dx  
0  2 
 (iv)
r r

Thus, we get the electric potential V of the non-conducting sphere at an


internal point P2 by adding Eqs. (ii) and (iv), as:
 2   R2  r 2 
V  V1  V2  r   
30 0  2 

  3R 2  r 2  4R 3  3R 2  r 2  Q (3R 2  r 2 )
      
3 0  2  3  40  2R
3 
 8   0R 3

44 (v)
Unit 1 Electrostatics in Free Space

where Q [ (4 / 3) R 3] is the total charge on the uniformly charged


non-conducting sphere.
5. Let us consider a closed path starting from and ending at a as shown in
Fig. 1.35. Let b be some point on this closed path. A unit positive charge
can be moved between points a and b through two paths: L and L. If Va
and Vb are potentials at a and b, respectively, we can write
b
  E.dl  Vb  Va (i) E
a
along
L Fig. 1.35: Diagram for
answer to TQ 5.
b
also   E.dl  Vb  Va (ii)
a
along
L

Now, by changing the limits of integration, we can write Eq. (ii) as:
b a
  E.dl   E.dl  Va  Vb (iii)
a b
along along
L L

Adding Eqs. (i) and (ii) and making use of Eq. (iii), we can write
b b b a
  E.dl   E.dl    E.dl   E.dl  Vb  Va  Va  Vb  0
a a a b
along along along along
L L L L

That is, along a closed path, the line integral of the electric field is equal to zero.
Alternative method: We can also use the fact that the line integral of electric field is
path-independent. Thus, we can write
b b
 E.dl   E.dl
a a
along along
L L
or
b b b a
 E.dl   E.dl  0   E.dl   E.dl  0
a a a b
along along along along
L L L L

Note that ( L  L) implies a closed path between points a and b in


Fig. 1.35.
6. The electric flux through the shaded right face (ABCD) of the cube having
area, say S , is
S    E .dS
S

To determine  S  , the trick is to choose an appropriate Gaussian surface


that encloses the charge q. We can put together 8 cubes of the same size 45
Block 1 Electrostatics
as the original cube in the problem to construct the Gaussian surface as
shown in Fig. 1.36.
It includes the right face ABCD of the original cube and encloses the
charge q. Note that the area of the Gaussian surface is 24 times the area
of the right face ABCD. So, now we can apply Gauss’s law to this problem.
  Q
From Gauss’s law, we have  E . dS  encl where S is the surface area
0
S
of the Gaussian surface enclosing the charge.

B
Gaussian surface
A

q C
D

Fig. 1.36: Diagram for answer to TQ 6.

Since the area of the Gaussian surface is 24 times the area S  of ABCD,
we have
    q   q
 
E . dS  24  E . dS 
0
or 
E . dS 
24  0
S S S

Thus, the electric flux through the right face (ABCD) of the cube is
q
E S  
240
S1 P
dS 7. See Fig. 1.37. A non-conducting thin spherical shell of radius R carries
R total positive charge Q that is distributed uniformly over its surface. To
Q
determine the electric field due to this shell at a point P lying outside it, we
draw a spherical Gaussian surface S1 through the point and concentric
with the spherical shell. Due to the spherical symmetry of the charged
spherical shell, its electric field has the same magnitude at every point on
any spherical Gaussian surface and is directed radially. We apply Gauss’s
law [Eq. (1.22a)] with Qencl  Q to the sphericalsurface S1 and note that

Fig. 1.37: A thin the  field E is in the same direction as dS for S1 so that
 electric
uniformly charged E and dS are parallel. Therefore,
spherical shell of  
radius R carrying a net E . dS  E dS (i)
charge Q. The
and since E (the magnitude of the electric field on the Gaussian surface) is
cross-section of the
Gaussian surface S1 is constant, we can pull it out of the surface integral. Therefore, Eq. (1.22a)
shown for a point lying becomes
outside the shell. It is Q
concentric with the  E .dS  E  dS  E 4r 2 
0
(ii)
shell. S S

1 Q
or E  for r  R (iii)
46 4  0 r 2
Unit 1 Electrostatics in Free Space
The electric field at any point lying outside the spherical shell of radius R is
given by
 1 Q
E  rˆ (spherical shell, for r  R) (iv)
4  0 r 2

Note that the electric field given by Eq. (iv) is the same as that due to a
point charge. For the electric field at a point lying outside the spherical
shell, it is as if the entire charge Q of the spherical shell were replaced by Q
R
a single equal charge placed at the centre of the shell. P
For a point lying inside the shell, we draw a spherical Gaussian surface S2
S2 concentric with the spherical shell, lying in the empty interior of the shell
(see Fig. 1.38). Since this Gaussian surface encloses no net charge, from
Gauss’s law, the electric field is zero at all points inside the shell:
 
E  0 (spherical shell, for r  R) (v)
Fig. 1.38: The cross-
So, when a charge is enclosed by a uniformly charged spherical shell so section of a Gaussian
that the charge lies inside the shell, no electrostatic force is exerted on the surface S2 enclosing
charge by the shell. the empty interior of
8. a) The electric field at a point inside the the thin uniformly
  inner copper wire (region I) is
zero since it is a conductor: E  0 . charged spherical shell
of radius R carrying a
b) Refer to Fig. 1.39. We take the Gaussian surface to be a coaxial net charge Q.
cylindrical surface of radius r and length L lying in the region II between
the wires. Note that the net charge enclosed by the Gaussian surface
is Qencl    L, where   is the linear charge density of the inner
Region
wire. From Gauss’s law, we have: +
+ +
I
+ +
  Q L
E . dS   
E dS  E dS  E 2r L  encl  
0 0
+
+

+
+
S S S + + Region
+ +
So, we have II
Region
  III
E   rˆ
2 0r
Fig. 1.39: Diagram for
where rˆ is the unit vector perpendicular to the cylindrical axis pointing the solution of TQ 8.
away from the axis. So, the electric field in region II is directed radially
inward.
c) For the point that lies outside the cable, the electric field is zero. This is ĵ E1 E2
because the two wires have equal and opposite linear charge densities I
Sheet 1
and the net charge
  enclosed by a Gaussian surface outside both wires
will be zero: E  0. E2
  II
E1
9. See Fig. 1.40. Let E1 be the electric field due to sheet 1 and E2 , the Sheet 2

electric field due to sheet 2 at some point in each of the three regions. The III
magnitudes of the electric fields due to the sheets will be equal since their
E1 E
surface charge densities are equal. Let us denote the magnitudes by E. 2

Then from Eq. (1.36), E 
20 Fig.1.40: Diagram for
answer of TQ 7.
Since both sheets are charged positively, the electric fields due to them
would be directed away from them in each region. The electric fields due 47
Block 1 Electrostatics
to the sheets in the three regions are shown in Fig. 1.40. Now we can
determine the net electric field at any given point in each region as follows:
a) Region I above the sheets: The electric fields due to the sheets are in
the same direction, say, ĵ , as both sheets are positively charged.
Therefore, the net electric field at a point in region I is
    ˆ  ˆ
E  E1  E2  2  j  j
20 0

b) Region II between the sheets: The electric field due to sheet 1 is


directed opposite to the electric field due to sheet 2. Therefore, the net
electric field at a point in region II is
c) Region III below the sheets: The electric fields are again in the same
direction, but opposite to ĵ . Therefore, the net electric field at a point in
region III is
     ˆ
E  E1  E2  2  (  ˆj )   j
20 0

10. We are given the electric field and the radius and height of the cylindrical
Gaussian surface and we have to determine the volume charge density of
the charge distribution enclosed by it. Since the surface area of the
cylinder is 2 r h, the electric flux through the Gaussian surface is

Q
E  E S  E  (2r h)  encl or Qencl  2 0r hE
0

The volume charge density  of the charge distribution is the net charge
enclosed per unit volume.
Qencl 2 0 r h E 2 0 E
   
V r 2 h r

2  8.85 10 12 C2N1 m 2  900 NC 1



1 .0 m

 1.6 108 C m3

48
Unit 2 Laplace’s and Poisson’s Equations

UNIT 2
LAPLACE’S AND
POISSON’S EQUATIONS
Structure
2.1 Introduction 2.4 Boundary Value Problems in
Expected Learning Outcomes Electrostatics
2.2 Laplace’s Equation Laplace’s Equation
2.3 Poisson’s Equation Poisson’s Equation
2.5 Summary
2.6 Terminal Questions
2.7 Solutions and Answers
2.1 INTRODUCTION
You have been introduced to Laplace’s and Poisson’s equations in the course
entitled Mathematical Methods in Physics (MPH-001). You have learnt about
their applications in many areas of physics. In this unit, we will discuss how to
solve both these equations (Secs. 2.2 and 2.3) with a special focus on
boundary value problems in electrostatics (Sec. 2.4). It will help you if you
keep Blocks 1 and 4 of the course MPH-001 handy so that you can refer to
various sections of the course referred to in this unit. You should also revise
vector differential and vector integral calculus from the undergraduate physics
and mathematics courses.
As you have learnt in Unit 1 of this course, the central problem of electrostatics
is to determine electric fields and electric potentials due to charges and
electrostatic forces on a charge or distribution of charges placed in an electric
field. In this unit, you will learn how Laplace’s and Poisson’s equations can be
solved to determine electric potentials and electric fields.
In the next unit, you will learn about the method of images for solving
Laplace’s equation and hence determining electric potentials and electric
fields.

Expected Learning Outcomes

After studying this unit, you should be able to:


 obtain the general solution of Laplace’s equation for problems in
electrostatics;
 obtain the general solution of Poisson’s equation for problems in
electrostatics; and
 solve Laplace’s and Poisson’s equations for given boundary value
problems in electrostatics. 49
Block 1 Electrostatics
2.2 LAPLACE’S EQUATION
In this section, we will obtain general solutions of Laplace’s equation in
one-, two- and three-dimensions in Cartesian, cylindrical and spherical polar
coordinates. We will refer to specific sections of Units 1 and 2 of the course
MPH-001. So, you should keep them handy.

We first write Laplace’s equation in one- two- and three-dimensions in


Cartesian coordinates:

d 2V
 0 (2.1a)
dx 2

d 2V d 2V
 0 (2.1b)
dx 2 dy 2

d 2V d 2V d 2V
2V     0 (2.1c)
dx 2 dy 2 dz2

where V is the electrostatic potential due to a given charge distribution


(discrete or continuous). The notation  or  is also used for electric potential.

Let us first consider the case when V depends only on one variable. As you
know, the general solution of Eq. (2.1a) is a straight line:
V ( x )  Ax  B (2.2)

where A and B are constants that are calculated for given boundary
conditions. For example, the potential between two conducting sheets or
plates of charge oriented perpendicular to the x-axis is a function of only the
distance between the sheets/conducting plates (recall the expression for
potential difference between capacitor plates).

In spherical polar coordinates, when V depends only on r [see Eq. (1.27),


Unit 1, MPH-001], Laplace’s equation (2.1a) is:

1   2 V 
 r r   0 (2.3a)
r 2 r  
which has the general solution:

A
V (r )   B (2.3b)
r
In cylindrical coordinates, when V depends only on the coordinate  [see
Eq. (1.23), Unit 1, MPH-001], Laplace’s equation (2.1a) is:

 2V 1 V
 0 (2.4a)
2  

which has the general solution:


V ()  A ln   B (2.4b)

You should verify Eqs. (2.2, 2.3b and 2.4b). Solve SAQ 1.
50
Unit 2 Laplace’s and Poisson’s Equations
SAQ 1

Verify Eqs. (2.2, 2.3b and 2.4b).

We have solved Laplace’s equation in two- and three-dimensions and


obtained its general solution in Sec. 1.3, Unit 1 of MPH-001 in Cartesian,
cylindrical and spherical polar coordinates for a variety of physical problems
(you may like to revise the section). You have learnt how to solve the
two-dimensional Laplace’s equation (2.1b) in Cartesian coordinates in
Example 1.1 of Unit 1, MPH-001. We will not solve it here. You may like to
work through Example 1.1 of Unit 1, MPH-001 and show that the general
solution of Eq. (2.1b) is of the form:

V  x, y    ( An cosh n y  Bn sinh n y ) (Cn sin n x  Dn cos n x )
n 1
(2.5)

where  n is a constant determined by the boundary conditions. Note that we


have retained the cosine term in the general solution which is absent in the
general solution of Example 1.1, Unit 1, MPH-001 because of the boundary
conditions there. You should work the steps until Eq. (2.5) for practice. The
two-dimensional Laplace’s equation (2.1b) in cylindrical coordinates is:

 2V 1 V 1  2V
  0 (2.6)
2   2 2

Let us obtain the general solution of Eq. (2.6) using the method of separation
of variables that you have learnt in Sec. 1.2 of Unit 1, MPH-001. Substituting
V (, )  R () P () and separating variables, we obtain the following ODEs:

P   n 2P  0 (2.7a)

and 2R  R  n 2R  0 (2.7b)

where n 2 is the separation constant. The solution of Eq. (2.7a) is of the form:
P ()  A cos n  B sin n (2.8a)

and using Frobenius method that you have learnt in Unit 1 of MPH-001, you
can show that the solution of Eq. (2.7b) is of the form:

R()  C n  D n (2.8b)

You should solve Eq. (2.7b) and obtain (Eq. 2.8b). Attempt SAQ 2.

SAQ 2

Solve Eq. (2.7b) and obtain Eq. (2.8b). Show that for n  0, the solution of
Eq. (2.7b) is: R()  C ln   D.

So, the linearly independent solutions of the two-dimensional Laplace’s


equation in cylindrical coordinates are: 51
Block 1 Electrostatics
1 ln 

n cos n  n cos n (2.9)

n sin n  n sin n

These solutions of Laplace’s equation are called cylindrical harmonics.


Thus, the general solution of Laplace’s equation in two-dimensions in
cylindrical coordinates is a linear combination of these solutions:

V (, )    An cos n  Bn sin n Cn n 
 Dn n (2.10)
n 1

The constants in the general solution [Eq. (2.10)] are obtained by imposing
boundary conditions for a given problem.

In Sec. 1.3.2, Unit 1, MPH-001, you have learnt about the three-dimensional
Laplace’s equation in cylindrical coordinates (, , z). For a potential V, it is
given as:

 2V 1 V 1  2V  2V
2V (, , z)     0 (2.11)
2   2 2 z2

It has the general solution [see Eq. (1.26) of Unit 1, MPH-001]:

 A ez ei J ()  B ez ei Y () 


    

  C ez e i J ()  D ez e i Y () 
     
V (, , z )  
  E e z ei J ()  F e z ei Y () 
       
 z i  
  Ge e J ()  He ze i Y () 
(2.12)
Let us now consider the solutions of two- and three-dimensional Laplace’s
equation in spherical polar coordinates.

The two-dimensional Laplace’s equation in spherical polar coordinates is


given by [refer to Eq. (1.27) of Unit 1, MPH-001; we put the term containing 
in it as zero for the two-dimensional case since the potential does not depend
on ] :
1   2 V  1   V 
r  sin  0
2 r     
(2.13)
r r  r sin   
2

From your study of Sec. 1.3.3 of Unit 1, MPH-001, you should be able to
determine the general solution of Eq. (2.13). Try SAQ 3.

SAQ 3

Obtain the general solution of Eq. (2.13).

On solving SAQ 3, you have obtained the general solution of Eq. (2.13) as:

52
V ( r , )  
l 0
 Cn r n  Dn r n 1  Pn (cos ) (2.14)
Unit 2 Laplace’s and Poisson’s Equations
The linearly independent solutions of the two-dimensional Laplace’s equation
in spherical polar coordinates are given as:

Vn (r , )  r n Pn (cos ) or Vn (r , )  r  n 1 Pn (cos  )

These are known as zonal harmonics.

Finally, the three-dimensional Laplace’s equation in spherical polar


coordinates [Eq. (1.27), Unit 1, MPH-001] is given as:

1   2 V  1   V  1  2V
 r  sin   0
r 2 r  r  r 2 sin      r 2 sin2  2
(2.15a)

with general solution [Eqs. (1.31a and b), Unit 1, MPH-001] given as:

 [Dl r l  Fl r l 1]Plm (cos )e im 
l
V (r , , )    (2.15b)
l  0m  0


 [Dl r l  Fl r l 1]Ylm (, ) 
l
V (r , , )    (2.15c)
l  0m  0

You should work through the relevant portion of Unit 1, MPH-001 to arrive at
Eqs. (2.15b and c). You will appreciate the importance of these methods when
we apply boundary conditions for specific charge distributions and obtain
particular solutions in Sec. 2.4.

Before you study Poisson’s equation, we would like to state two important
properties of the solutions of Laplace’s equation that you have already
encountered. We state them as the following theorems:
Theorem 1: If 1, 2 ,. .., n are linearly independent solutions of Laplace’s
equation, then their linear combination:
  C1 1  C2 2  ...  Cn n (2.16)

where C1, C2 , . . . Cn are arbitrary constants, is also a solution. Recall that you
have learnt this property in the course MPH-001 and also in your UG courses.

Theorem 2: Uniqueness Theorem  The solution  of Laplace’s equation


that satisfies either of the following boundary conditions is unique:

i) The value of  is specified on the surface S which bounds the volume V


(known as the Dirichlet condition),

or

ii) The normal derivative of  is specified on the surface S which bounds


the volume V (known as the Neumann condition).

Alternatively, we say that two solutions of Laplace’s equation that satisfy


the same boundary conditions differ at most by an additive constant.

Let us prove this theorem.


Proof: Let 1 be one solution of Laplace’s equation that satisfies the given
boundary conditions. Suppose  2 is another solution that satisfies the same 53
Block 1 Electrostatics
boundary conditions. We will prove that 1 and  2 are identical. Let us
suppose that
  1  2 (2.17)
If we can show that   0 or 1   2 throughout the volume V and on the
surface S of the boundary, then we can prove the uniqueness theorem. Let us
do that. We use the following vector identity [refer to Eq. (4.20c), Sec. 4.3.3 of
Unit 4, BPHE-104]:

 . ( )   .     2  ()2 (  2  0) (2.18a)

Integrating Eq. (2.18a) over volume V, we get:

 () d    .( ) d


2 (2.18b)
V V

Using the divergence theorem, we can write Eq. (2.18b) as:



 () d   ( ). dS
2 (2.18c)
V S

Since the solutions are equal at the surface S of the boundary, 1   2 or


  0 and the RHS of Eq. (2.18c) is zero. Thus, Eq. (2.18c) becomes:

 () d  0
2 (2.18d)
V

Since the integrand in Eq. (2.18d) is positive, Eq. (2.18d) will be satisfied (i.e.,
the integral will be zero) only if the integrand is zero everywhere. Thus, we
have:
  0 (2.18e)

  
or  0,  0 and 0 (2.18f)
x y z

which yields
  constant (2.18g)
However, since   0 on the boundary, and is constant, therefore   0
everywhere, and hence, we get:
1   2 (2.18h)

Thus, the uniqueness of the solution of Laplace’s equation (Dirichlet condition)


is proved. Further, if the boundary condition is the Neumann condition, viz.

that the normal derivative of  is specified on the surface S, meaning 0
n
on the surface S, then Eq. (2.18f) implies that  is equal to an arbitrary additive
constant throughout V. This means that the two solutions 1 and  2 differ at
most by an additive constant, or 1   2 within an additive constant, and the
uniqueness of the solution is established.
You may ask: What is the significance of the uniqueness theorem? It is
significant for the reason that once a solution of Laplace’s equation is obtained
for a set of boundary conditions, then that solution is unique. We need not
54 make any effort to determine other possible solutions.
Unit 2 Laplace’s and Poisson’s Equations
So far, you have learnt how to solve one-, two- and three-dimensional
Laplace’s equations in the Cartesian, cylindrical and spherical polar
coordinates. Let us now discuss Poisson’s equation to which you have been
introduced in Unit 2 of MPH-001.

2.3 POISSON’S EQUATION


In Unit 2 of MPH-001, you have learnt that Laplace’s equation is a special
case of Poisson’s equation [Eq. (2.2a), Unit 2, MPH-001], which is given by:
 From Gauss’s law:
2V   (2.19)
0 
. E 
0
This equation follows from the differential form of Gauss’s law that you have 
 Since E is conservative,
learnt in Unit 1:  . E  [Eq. (1.25)]. You can deduce it yourself or read the we can express it as:
0
E   V
margin remark.
Substituting this
We can solve Poisson’s equation for the potential V if we know how the charge 
expression of E and
density  varies in space. In fact, you have learnt how to solve Poisson’s 2
using  .    in
equation for the gravitational potential in Sec. 2.2 of Unit 2, MPH-001 and also
Gauss’s law, we get:
its general solution. Let us use the general form of Poisson’s equation and its 
solution given by Eqs. (2.1 and 2.5) from Unit 2 of MPH-001 and then apply 2V  
0
them to electrostatics. These are:

 2f ( x, y , z )  u( x, y , z ) (2.20a)

1 u( x , y , z)
f ( x, y , z )  
4  ( x  x  ) 2  ( y  y ) 2  ( z  z  ) 2
dx  dy dz

(2.20b)
 ( x, y , z )
We substitute f ( x, y , z )  V ( x, y , z ) and u( x, y , z )   in Eqs. (2.20a)
0
and 2.20b) to write:
 ( x, y , z )
2V ( x, y , z )   (2.21a)
0

1 ( x , y , z)
and V ( x, y , z )  
0  ( x  x )2  ( y  y )2  ( z  z)2
dx  dy dz (2.21b)

or in compact notation, we can write Eq. (2.21b) as:


1  ( r )
V 
0  r  r
d  (2.21c)

We can show that Eq. (2.21c) is indeed the solution of Eq. (2.21a) as follows.

Applying the Laplacian operator  2 on both sides of Eq. (2.21c), we get:


1 2  (r )
2V  
0
  r  r
d  (2.21d)

1 1
2V     (r )  d 
2
or (2.21e)
0 r  r 55
Block 1 Electrostatics
We now use the Green’s functions to solve Poisson’s equation. For this we
 
suppose that we have a solution [ G (r , r ) ] of Poisson’s equation when the
right-hand side is the Dirac delta function (refer to Sec. 13.3 of Unit 13,
MPH-001):
   
 2 G (r , r )   (r  r )   ( x  x )  ( y  y )  ( z  z)
(2.22a)
The Dirac delta function has the following property (see Sec. 13.3.1, Unit 13,
MPH-001):

  (r )  (r  r )d   (r ) (2.22b)

or   ( x, y , z)  (r  r )d   ( x, y , z ) (2.22c)

if the volume of integration includes the point (x, y, z) (and the integral is zero
otherwise). Now using Eq. (2.22a) in Eq. (2.22c), we can write:
1
2 V ( x, y , z )     G (r , r )  (r ) d 
2
(2.22d)
0

1  (r )
   (r  r )  (r ) d   
0 0

which is Poisson’s equation. Therefore, from Eq. (2.22d), you can see that a
solution of Eq. (2.21a) is given by:
1
V ( x, y, z)  
0  G (r , r )  (r ) d  (2.23)

Now comparing Eq. (2.23) with Eq. (2.21c), we can write:


  1
G ( r , r )    (2.24)
r  r

Note that Eqs. (2.21c and 2.24) give solutions that are zero at infinity. From
Eqs. (2.24 and 2.22a), we also get the result that:
1  
2     ( r  r ) (2.25)
r  r

SAQ 4

Solve Poisson’s equation to determine the electric potential due to the charge
e  r .
q
distribution having charge density
4 0 r

2.4 BOUNDARY VALUE PROBLEMS IN


ELECTROSTATICS
Most of the times, in electrostatics, electric fields and electric potentials are
known and then we need to determine the charge densities (of charge
56 distributions) that give rise to those fields and potentials. Many a times, we are
Unit 2 Laplace’s and Poisson’s Equations
given the charge densities, (which could also be zero as in Laplace’s
equation) and then we need to determine electric potentials and electric fields.
In all such cases, we need to apply boundary conditions for the problem and
determine the potentials or charge densities, as the case may be, by solving
Laplace’s and Poisson’s equations. In this section, we consider a few
examples of boundary value problems for solving both equations in different
geometries. Let us first solve Laplace’s equation for given boundary
conditions.

2.4.1 Laplace’s Equation


Let us solve Laplace’s equation for the familiar example of parallel plate
0   0
capacitor, coaxial cable and spherical capacitor for given boundary conditions.
 +
Example 2.1
 +

Determine the electric potential in a parallel plate capacitor having infinitely  +


long plates (Fig. 2.1).
 +
Solution : Let us take the x-axis to be along the separation of the two parallel
plates. Let one of these plates be at x  0 and the other at x  d . Suppose  +
the first plate is grounded, which means that it is at zero potential. Let the right  +
x
plate be at constant potential V0 . Note that there is no charge in the region 0 d
between the two plates. Therefore, the electric potential in the region between Fig. 2.1: A parallel
the two plates of the capacitor is obtained by solving Laplace’s equation. plate capacitor.

Let us now state the boundary conditions:

Since the potentials are specified at the two plates, the Dirichlet boundary
condition holds, namely, the left plate is at zero potential and the right plate at
potential V0 :

At x  0, V 0

At x  d , V  V0 (i)

Since the plates are of infinite length, we can neglect the variation of potential in
the y- and z-directions. We consider potential variation only in the x-direction.
Laplace’s equation is, therefore, reduced to a one-dimensional equation. Thus,
we have:
d 2V dV
2V  0   C  V  Cx  D (ii)
2 dx
dx

When we apply the boundary conditions (i), we get:

At x  0, V 0  D 0

V0
At x  d , V  V0  C 
d

Therefore, the solution is:


V0
V  x (iii)
d 57
Block 1 Electrostatics
The next step is to calculate the surface charge density on the plates. From
Eq. (iii), we can write the electric field as:
dV ˆ V
E  i   0 iˆ (iv)
From Gauss’s law: dx d
q S
 E . dS   and from Gauss’s law, the surface charge density is [read the margin remark and
S 0 0
or
refer to part (ii) of Example 1.16 of Unit 1]:
S   0 E . nˆ (v)
ˆ
 E . ndS 
S 0
where E . nˆ is the normal component of the electric field. The surface charge
where S is the
Gaussian surface, a pill
density on the plates is given by:
box (refer to Sec.1.5.3 V
  0 0 (vi)
and Example 1.16 of d
Unit 1 of this course).

Since from Eq. (iv), E is From the surface charge density, we get the total charge by multiplying it with the
constant, we can take area of the plates. Although the plates are of infinite length, the area will also be
its normal component infinite, but since we have determined the surface charge density, we can always
E . nˆ out of the integral
multiply it with the surface area A of the finite plates. Then we get the magnitude
and write:
of the total amount of charge on each plate as:
(E . nˆ )  dS  (E . nˆ )S V
S Q  0 A 0 (vii)
d
S

0 which gives the expression of capacitance as:
or Q  A
C  0 (viii)
  0 (E . nˆ ) V0 d

This is a well known expression right from school physics.

Let us now solve a boundary value problem involving the two-dimensional


Laplace’s equation in cylindrical coordinates. The simplest example is that of a
cylinder or a coaxial cable, which is used all around us.

Example 2.2
 + + 
Solve Laplace’s equation for an infinite coaxial cable of inner radius a and
 + + 
outer radius b given that the outer cable is grounded and the inner cable is at
 + + 
potential V0 (Fig. 2.2).
 + + 
 + +  Solution : This is a problem having cylindrical symmetry and so we use
 + +  cylindrical coordinate system and Laplace’s equation in cylindrical coordinates
  [Eq. (2.11)]. Now, recall that for cylindrical symmetry, the electric field and
electric potential will depend only on , the perpendicular distance from the axes
Fig. 2.2: BVP for coaxial of the inner and outer cylinders. So, then we need to solve only Eq. (2.4a):
cable.
 2V 1 V
 0 (i)
 2  

for the boundary conditions:


At   b, Vb  0

58 At   a, Va  V0
Unit 2 Laplace’s and Poisson’s Equations
The general solution of Eq. (i) is:
V  A ln   B (ii)

Applying the boundary conditions:


V (b)  0  A ln b  B  B   A ln b

a
V (a)  V0  A ln a  B  A ln a  A ln b  A ln  
b
V0
A
ln(a / b)

V0 V0 ln( / b)
V  ln   ln b  V0
ln(a / b) ln(a / b) ln(a / b)

Let us now determine the electric field, charge density and capacitance for the
coaxial cable.

E   V

Since V depends only on , gradient depends on  and therefore, we get:


V V0  V0 1
E  ˆ  [ln()  ln(b)] ˆ  ˆ
 ln(a / b)  ln(a / b) 

Charge density is:

  0 E . nˆ

For the inside cylindrical surface, the normal component of E is outward, which is
along ̂ and the normal component for the outside cylindrical surface is opposite
to ̂. Therefore, the charge density on the inside cylindrical surface for   a is:

0 V0

a ln(a / b)

Since the coaxial cables are of infinite length, we determine charge per unit
length, which is:
2a 0 V0 2 0 V0
Q   (2a)  
a ln(a / b) ln(a / b)

Therefore, the capacitance of the coaxial cable is:


Q 2 0
C 
V0 ln(a / b)

Finally, we solve the three-dimensional Laplace’s equation in spherical polar


coordinates for a given boundary value problem for a spherical capacitor.

Example 2.3
A spherical capacitor is made up of two hollow concentric metal spheres of
inner radius a and outer radius b, which is earthed. A charge + Q is placed on
the inner sphere and an equal and opposite amount of charge appears on the 59
Block 1 Electrostatics
inner side of the outer sphere. Solve Laplace’s equation for the system and
obtain the expression for its capacitance given that the inner metallic sphere is
maintained at a constant potential.

Solution : We show the cross-section of the spherical capacitor in Fig. 2.3. This
Q is a problem having spherical symmetry and so we use Laplace’s equation in
Q spherical polar coordinates. Due to spherical symmetry, the potential does not
b depend on  and  and Laplace’s equation becomes:
a
1   2 V 
 r r   0
r 2 r  
with the boundary conditions:
Fig. 2.3: BVP for
At r  b, Vb  0
spherical capacitor.
At r  a, Va  V0

Integrating Laplace’s equation, we get:


V
r2 A
r
where A is the constant of integration. Integrating once again, we get:
A
V  B
r
Let us apply the boundary conditions to the potential:
A A
V (b )  0   B  B
b b
A A A  1 1 V ab
V (a)  V0   B   A    A 0
a a b b a a b

Thus,
V ab 1 V0ab
V  0 
b a r b(a  b )

V0ab  1 1 
V   
(b  a )  r
or
b
V V ab 1
Now the electric field is: E   V    0 rˆ
r (b  a ) r 2

and the surface charge density at the inner sphere is:


V0ab 1
  0 E . nˆ  0
( b  a ) a2

This gives the total surface charge on the inner sphere as:
V0ab 1 V ab
Q  0  4a2  40 0
( b  a ) a2 (b  a)

Hence, the capacitance of this arrangement is:


Q 40ab
C 
V0 (b  a )
60
Unit 2 Laplace’s and Poisson’s Equations
So far, we have dealt with simple boundary value problems in which Laplace’s
equation was reduced to one-dimensional form due to the specific geometries.
Let us now take up a different example involving two-dimensional Laplace’s
equation (refer to Terminal Question 5, Unit 1, MPH-001).

Example 2.4

Solve Laplace’s equation in plane polar coordinates for electric potential on a


circular metallic disc of radius L with the following boundary conditions:

1   V  1  2V 
r   0, 0  r  L,       (i)
r r  r  r 2  2 

V (L, )  sin 3,     

Solution : We have to solve Laplace’s equation using plane-polar coordinates


( r , ).

Note that there are two special features of this problem:


The points    and    coincide. Therefore, the value of V and its
angular derivative w.r.t.  should match there:
V
V (r ,  )  V (r , ), r ,    V r , , 0  r  L
 
2
 V
The point r  0 is singular: the coefficient of in Eq. (i) is 1, while the
r 2
coefficients of other terms are 1/r and 1/ r 2 . We must, therefore, enforce a
condition of boundedness:
V (r , ) tends to a finite value, i.e., it is bounded, as r  0.

Keeping these special features in mind, we can solve the problem using the
method of separation of variables. We will not be repeating the solution here.
You may refer to TQ 5, Unit 1, MPH-001.
The general solution for V (r , ) is:

V (r , )  A0   r n An cos n  Bn sin n
n 1

The boundary condition on r  L yields:



A0   Ln An cos n  Bn sin n  sin 3, 
n 1

From this boundary condition, you can see that An  0, for all n. And the only
term that survives in the sine series is the term for which n  3, that is, the
term containing B3 . Thus,

L2B3  1  B3  1/ L2

Therefore, the unique solution is:

r2
V ( r , )  sin3 61
L2
Block 1 Electrostatics
Notice that we have solved Laplace’s equation for Dirichlet condition in the first
three examples and for Dirichlet and Neumann condition in Example 2.4. You
may like to solve an SAQ for practice before studying the next section.

SAQ 5
Starting from the general solution of Laplace’s equation in Cartesian
coordinates [see Eq. (1.15e), Unit 1, MPH-001], determine the potential in the
region between two infinite parallel plates at y  0 and y  2 for the
following boundary conditions:
V ( y  0)  cos 4 x cosh5z
and V ( y  2)  2cos 6cos 4 x cosh5z

2.4.2 Poisson’s Equation


Usually, it is difficult to solve Poisson’s equation analytically for given
potentials and numerical methods are used to obtain solutions. These
methods are beyond the scope of this course and you may learn about them
in the course on Computational Physics. We will take the example of solving
Poisson’s equation analytically for the p-n junction.

Example 2.5

Solve Poisson’s equation for a p-n junction of width d for which the volume
charge density is given by:

x x
  20 sech tanh , (i)
d d

where 0  eN A  eND (ii)

Here N A and ND are the number of acceptor and donor atoms in the p and n
You should note that
the symbol  is used in sides near the junction.
the text for volume
charge density as well Solution : Poisson’s equation for the p-n junction is:
as for a cylindrical 
 x x
coordinate. You need to 2 V     2 0 sech tanh (iii)
be clear about the 0 0 d d
context in which it is
being used. Since the charge density depends only on x, we need to solve the
one-dimensional Poisson’s equation:

d 2V  x x
  2 0 sech tanh (iv)
dx 2 0 d d

Integrating Eq. (iv) once, we get:

dV 20d x
 sech  A (v)
dx 0 d

40d 2
and V  tan1 (e x / d )  B (vi)
62 0
Unit 2 Laplace’s and Poisson’s Equations
Applying the condition that the potential at the centre of the junction is zero, we
get:

40d 2  4 d 2   d 2
0 B  B 0  0 (vii)
0 4 0 4 0

Thus, we get:

40d 2  d 2
V  tan1 (e x / d )  0
0 0

The corresponding electric field is:


dV 2 d x
Ex     0 sech
dx 0 d

With this example of solving Poisson’s equation for a given charge density, we
end the discussion on Laplace’s equation and Poisson’s equation, and
summarise the contents of this unit.

2.5 SUMMARY
In this unit, we have discussed the following concepts:

 General solution of Laplace’s equation for problems in electrostatics


in Cartesian, cylindrical and spherical polar coordinates;

 General solution of Poisson’s equation for problems in electrostatics


in Cartesian, cylindrical and spherical polar coordinates;

 Solutions of Laplace’s equation for Boundary Value Problems in


electrostatics and applications to systems having linear, cylindrical and
spherical symmetries; and

 Analytical solution of Poisson’s equation for specific Boundary Value


Problem in electrostatics.

2.6 TERMINAL QUESTIONS


1. Two semi-infinite conducting plates are kept at an angle of 0 to each
other as shown in Fig. 2.4. If the plate A is earthed and plate B is
maintained at a constant potential V0 , calculate V and E in the region
between the plates.
Insulating
gap

0
A B

V0

Fig. 2.4: Figure for Terminal Question 1. 63


Block 1 Electrostatics
2. The potential difference between two conducting spherical shells having
radii R1 (inner shell) and R2 (outer shell) is constant such that the outer
shell is at zero potential. Determine the electric potential and the electric
field in the region between the shells.
3. Solve Poisson’s equation to obtain the electric potential and electric field
due to a line charge along the x-axis having constant charge per unit
length for the following boundary conditions:
V ( x )  V0 at x  0 and V ( x )  0 at x  L.

 x
4. The charge density distribution in a wire is given by:  ( x )  0 . Show
L
that for the boundary conditions:
V ( x )
 0 at x  0 and V ( x )  0 at x  L
x

the electric potential and electric field due to the wire are given by:

V (x) 
0 3
60L
    x2
L  x 3 and E ( x )  0 iˆ
20L

2.7 SOLUTIONS AND ANSWERS


Self-Assessment Questions
d 2V
1. To solve  0, we integrate it twice. Thus, we get:
dx 2
dV
 A and V  Ax  B which is Eq. (2.2).
dx
1   2 V 
To solve  r r   0, we integrate it twice and get:
r 2 r  

V A A
r2
r
 A and V   r 2 dr B  V 
r
B

which is Eq. (2.3b).

 2V 1 V 1   V 
To solve   0, we rewrite it as:    0, and
2       
integrate it twice to get:

V 1
  A  V  A  B  A ln   B
 

which is Eq. (2.4b).

2. We use the Frobenius method to solve the ODE  R  R  n R  0 . We


2 2

expand R in the following series about   0, and take its first and second
order derivatives with respect to  :

R ()   amm k
m 0
64
Unit 2 Laplace’s and Poisson’s Equations

R ()   am (m  k )m k 1
m 0


R()   am (m  k ) (m  k  1)m k 2
m 0

Substituting R and its derivatives in the given ODE, we get:

 am (m  k )(m  k  1)  (m  k )  n2   0

k
m 0

k
The indicial equation is the coefficient of  for m  0 :


a0 k (k  1)  k  n 2  0   k 2

 k  k  n2  0

or k 2  n2  0  k   n

So, the solution is of the form R()  Cn  Dn , which is Eq. (2.8b). For
n  0, the solution of the ODE has been obtained in SAQ 1. It is:
R ()  C ln   D.

1   2 V  1   V 
 r    sin   0, we first separate the
 
3. To solve
r 2 r  r  r 2 sin   
PDE into three ODEs using the method of separation of variables. We get
the following three ODEs:

d 2
 m 2  0 (i)
2
d
d 2R dR
r2  2r  n2 R  0 (ii)
2 dr
dr
d 2 cos  d  2 m2 
  n   0 (iii)
d2 sin  d  sin2  
The solution of Eq. (i) is given by:

()  Ameim  Bm cos   Cm sin  (iv)

For solving Eq. (ii), we use the Frobenius method and obtain the solution
for n 2  l (l  1) as follows:

Rl (r )  Dl r l  Fl r l 1 (v)
We expand R in the following series about r  0, and take its first and
second order derivatives with respect to r:

R (r )   am r m  k
m 0


R (r )   am (m  k )r m  k 1
m 0


R (r )   am (m  k ) (m  k  1)r m  k  2
m 0 65
Block 1 Electrostatics
Substituting R and its derivatives in the given ODE, we get:


r k  am (m  k )(m  k  1)  2(m  k )  n 2  0
m 0

The indicial equation is the coefficient of r k for m  0 :


a0 k (k  1)  2k  n 2  0   k 2

 k  2k  n 2  0

 1  1  4n 2
or k 2  k  n2  0  k
2
Now we substitute n 2  l (l  1), so that

 1  1  4l 2  4l  1  (2l  1)
k 
2 2
Thus, we get 2 roots:
k1  l , k 2   l  1

So, the solution is of the form Rl (r )  Dl r l  Fl r l 1

where Dl and Fl are constants of integration. As you have learnt in


Sec. 1.3.3 of Unit 1, Eq. (iii) may be recast as the ODE for Associated
Legendre polynomials with the change in variable x  cos  and with
m  0, 1,..., l .

The general solution is:



 [Dl r l  Fl r l 1]Plm (cos )[Bm cos   Cm sin ] 
l
V (r , , )   
l  0m  0
(vi)
which can also be written as:

 [Dl r l  Fl r l 1]Plm (cos )e im 
l
V (r , , )    (vii)
l  0m  0

For m  0, the general solution is:


 l
V (r , , )    (Dl r l  Fl r l 1)Pl (cos )
l  0m  0

er , depends only on the variable r, we


q
4. Since the charge density,
40r
need to use only the radial part of the Laplacian in Poisson’s equation, which
becomes:
1   2 V  
er
q
 r   
2 r   2
r  r  0 4 0r

We now have to integrate the above equation twice to determine V(r). Hence,
we have:

rer dr  A  C rer dr  A
dV q
r2
dr
 
2
4 0
 
66
Unit 2 Laplace’s and Poisson’s Equations
q
where C   Integrating by parts, we get:
4 02

rer e  r
 rer dr   
 2

dV  rer er 
Therefore, r 2  C   A
dr   2 
 

1  rer er 
 dr  B
whence V (r )   C  r 2  

2 

 rer er 
or V (r )   C    dr  B
   2 2
 r 
Integrating the second integral by parts, we get:

1 er 1  er er 


 dr     dr 
2 r2 2  r r 

 rer er rer 
Thus, V (r )  C   
dr    dr   B
  2r  
 

C e  r q e  r
or V (r )   B B
2 r 402 2 r

1   2 V 
You can verify the above expression for V() by evaluating r  and
r 2 r  r 

er .
q
verifying that  
4 0r

5. The general solution [Eq. (1.15e), Unit 1, MPH-001] of three-dimensional


Laplace’s equation in Cartesian coordinates is given by:

V  x, y , z    ( An cos k1n x  Bn sin k1n x ) (Cn sin k 2n y  Dn cos k 2n y )
n 1
 (Pn cosh k3n z  Qn sinh k3n z )

where k12  k 22  k32 . We have to determine the potential in the region


between the two infinite parallel plates at y  0 and y  2 for the
following boundary conditions:
V ( y  0)  cos 4 x cosh5z
and V ( y  2)  2cos 6cos 4 x cosh5z
Since on both plates, the dependence on x and z is given as cos 4x cosh 5z,
the boundary conditions imply that we look for a general solution of the form:
V  x, y, z   A cos 4x cos3y cosh5z  B cos 4x sin3y cosh5z

Applying the first boundary condition, we get:


V  x, 0, z   A cos 4x cosh5z  cos 4x cosh5z  A  1 67
Block 1 Electrostatics
Applying the second boundary condition, we get:
V  x, 2, z   cos 4x cos 6cosh5z  B cos 4x sin6cosh5z  2cos 6cos 4x cosh5z

which gives B  cot 6

Thus, V  x, y, z   cos 4x cos3y cosh5z  cot 6cos 4x sin3y cosh5z

Terminal Questions
1. See Fig. 2.5.
z

Insulating
gap

0
A B
y

x
V0

Fig. 2.5: Diagram for answer of Terminal Question 1.

Since the potential depends only on , Laplace's equation in cylindrical


coordinates for the system reduces to:

1 d 2V
0 (i)
2 d2

Now, because of the insulating gap, we need not include the point   0 in
our calculation of the potential. So, we multiply Eq. (i) by 2 and write:

d 2V
0 (ii)
d2

You know that the solution of Eq. (ii) is:


V ()  C   D

where C and D are constants. Next we apply the boundary conditions to


obtain the values of C and D:
When   0, V  0 because the plate A is earthed. Hence,

D0

At plate B:
  0 , V  V0

Therefore,
V
C 0
0

V0
and V ()   (iii)
68 0
Unit 2 Laplace’s and Poisson’s Equations

The electric field E in the region between the plates is given by:
  1 dV ˆ 1 V0 ˆ
E   V     
 d  0

2. Refer to Fig. 2.6. Due to spherical symmetry of the system, the potential R2
depends only on r and Laplace’s equation is reduced to: R1

1 d  2 dV 
r 0 (ii)
r 2 dr  dr 
As you know, the solution of Eq. (i) is: Fig. 2.6: Diagram for
answer to Terminal
C
V (r )   D (ii) Question 2.
r
Now we apply the boundary conditions:
When r  R1, V  V0 because the inner shell is at a constant potential.
When r  R2, V  0

C
Thus, we have: D 
R2

C C
and V0   
R1 R2
V0
Therefore, C 
 1 1
  
 R2 R1 

V0 1 1 V0 V0 1 1 
and V (r )       
 1 1 r R 2  1 1  1 1  r R 
         2
 2
R R 1  2
R R 1  1
R R 2
Electric field in the region between the shells is given by:
  dV V0 1
E   V   rˆ  rˆ
dr  1 1  r2
  
 R1 R2 

3. Poisson’s equation along the x-axis for a line charge having constant
charge per unit length is:

d 2V 
 0
dx 2 0

and its solution after integration is:


0 2
V (x)   x  Cx  D
20
For V ( x )  V0 at x  0, we get: D  V0

1  0 2   L V
For V ( x )  0 at x  L, we get: C   L  V0   0  0
L  20  20 L

0 2  0L V0 
Therefore, V ( x )   x     x  V0
20  2 0 L  69
Block 1 Electrostatics
The electric field is:
  dV ˆ  0   L V    x  L   V0  iˆ
E   V   i  x   0  0  iˆ   0  
dx  0  2 0 L   0  2 L 

4. Once again we solve the one-dimensional Poisson equation:

d 2V ( x )  x
  0
dx 2 0  0L
0 3
Its solution is: V ( x )   x  Cx  D
60L

Applying the boundary conditions:


V ( x )
 0 at x  0, we get: C  0
x

 L2
and for V ( x )  0 at x  L, we get: D   0
60

0   x2 ˆ
Thus, V (x)  L3  x 3  and E( x )   dV  0
2 0 L
i
6 0L dx

70
Unit 3 Special Techniques

UNIT 3
SPECIAL TECHNIQUES
Structure

3.1 Introduction 3.4 Multipole Expansion


Expected Learning Outcomes 3.5 Summary
3.2 The Method of Images 3.6 Terminal Questions
3.3 Applications 3.7 Solutions and Answers

3.1 INTRODUCTION
In Units 1 and 2, you have learnt the basic techniques of obtaining the electric
potential and electric field due to a variety of charged systems comprising
discrete as well as continuous charge distributions. You can apply the Gauss’s
law to determine electric field due to symmetric charge distributions. You can
also solve Laplace’s and Poisson’s equations and determine the electric
potentials and electric fields of a variety of systems of charges.
In this unit, you will learn about two special techniques, namely, the method
of images along with its applications and the multipole expansion. This is
the final unit on electrostatics in free space/vacuum. In the next unit, you will
learn about electrostatics in material media.

Expected Learning Outcomes


After studying this unit, you should be able to:

 apply the method of images to solve problems in electrostatics for charge


distributions in free space/vacuum; and

 use multipole expansion technique to determine the electric potential due


to system charges.

3.2 THE METHOD OF IMAGES


The method of images is an “intuitive” method used to solve specific problems
in electrostatics for which it is difficult to determine analytic solutions of
Poisson’s or Laplace’s equations or the mathematics required is involved. It is
a special technique that enables us to arrive at solutions in specific cases. To
understand the method of images, consider a charge, charges or charge
distribution placed near a conductor. Ordinarily we would be required to solve 71
Block 1 Electrostatics
Poisson’s equation for such systems. But we can use the method of images to
solve such problems. Let us explain how we can do that.

When a conductor is placed in an external electric field [due to a collection of


charge(s)], induced charge(s) appear on its surface. So, we may view the
electric field in the region outside the conductor as a superposition of the given
electric field and the electric field due to the induced charge(s). Since the
boundary in such a system is the conductor, the potential at the surface of the
conductor is a constant, say, V0 (you know from UG physics that the surface
of a conductor is an equipotential surface). Then we have to solve Laplace’s
equation to determine the electric potential in our region of interest in which
the boundary conditions apply:

 2 V ( r )  0 (3.1)

such that V (r )  V0 on the surface S of the conductor (equipotential
surface).

If we can find a solution that satisfies Eq. (3.1) and the boundary conditions for
the problem, then the uniqueness theorem for Laplace’s equation tells us that
we have the unique and correct solution. Recall from Unit 2 that the
uniqueness theorem states that the solution of Laplace’s equation that
satisfies either the Dirichlet or the Neumann boundary conditions is unique.
So, as long as the solution satisfies i) Laplace’s equation and ii) the given
boundary conditions, we get the unique solution, which is the simplest solution
among all possible solutions.

Now outside the surface S, which is our region of interest, we may place any
distribution of charges that we wish. Since these charges are external to the
conductor’s surface, these are not sources inside S and Laplace’s equation
 2 V  0 will still be satisfied inside and on S. This is the trick that is used in
the method of images.

We place charges outside the surface of the conductor in such a manner that
Recall from V  V0 on the surface of the conductor. In this manner, we obtain the solution
school/UG physics
that earthed or without having to solve Laplace’s equation. The fictitious charges that we
grounded means place outside the conductor’s surface are called image charges. That is why
that the potential is this special technique is called the method of images (this terminology is
the same as that of taken from optics). Let us take the example of a single charge placed near an
the surface of the infinite conducting plane that is earthed (grounded) to illustrate this method.
Earth. Therefore, it
is the same as at a Refer to Fig. 3.1, which shows a positive point charge Q held at a distance D
point very far away above an infinite earthed/grounded conducting plane (read the margin
from the conductor, remark).
at infinity.
Let us ask: What is the electric potential in the region above the plane?
1 Q
Note that the electric potential is not given by:
4 0 r

Why? This is because the charge Q will induce an equal and opposite charge
( Q) on the surface of the conductor. So, the total potential at any point r will

72
Unit 3 Special Techniques
be due to both Q and  Q. The question is: How do we determine the potential
if the distribution of the induced charge is not known to us?

+Q

D
y

Fig. 3.1: Electric potential due to a charge near an earthed infinite conducting
plane.

You have learnt about Poisson’s equation in Unit 2. You can recognise that
the original problem is that of solving the Poisson’s equation in the region
z  0, for the single point charge Q situated at (0, 0, D). The boundary
conditions for the system are as follows:

1. Since the conducting plane is earthed,

V  0 when z  0 and

2. At points very far away from the charge, the electric potential must tend to
zero:

V  0 when x 2  y 2  z2  D2

So far, you have learnt how we can convert this problem to a problem of
solving Laplace’s equation and finding a unique solution, the only solution that
satisfies these boundary conditions. Can we guess this unique solution? The
method of images helps us do that.
This is what we do: Using the image of the charge + Q, we devise a new
system of charges as follows and solve the problem without having to solve
either Poisson’s equation or Laplace’s equation.
The new system consists only of two point charges, +Q at (0, 0, D) and ( Q)
at (0, 0,  D), and there is no conducting plane (Fig. 3.2).
z
r1 P

+Q
D
r2
y

x Q

Fig. 3.2: A system of two charges, +Q at (0, 0, D) and ( Q) at


(0, 0,  D), with no conducting plane replaces the original system.
73
Block 1 Electrostatics
Instead of solving Laplace’s equation for this problem, we suppose that an
“image charge” exists at a distance D “below” the plane. This is like the image
of an object located in front of a mirror. You know that the image of the object
is “behind” the mirror and virtual. In the same way, note that the image charge
is located at a virtual distance D “below” the plane, and the charge itself is
fictitious. We are just imagining the charge to be there.
Let the point P ( x, y , z) at which we wish to determine the electric potential be
at a distance r1 from the charge + Q and distance r2 from the image charge
 Q (see Fig. 3.2). You know from school and UG physics that the electric
potential for this system is:

Q 1 1
V    (3.2a)
4  0  r1 r2 

where

r1  x 2  y 2  ( z  D )2 (3.2b)

and r2  x 2  y 2  ( z  D )2 (3.2c)

Hence,

 
Q  1 1 
V 
4   0  x 2  y 2  ( z  D )2 2 2 2
x  y  ( z  D) 

(3.2d)

You can verify that Eq. (3.2d) satisfies the boundary conditions for the problem:

1. For z  0, V  0

2. When x 2  y 2  z2  D2 , then again V  0

Notice that the only charge in the region z  0 is the point charge +Q at
(0, 0, D). Also note that together with the boundary conditions, this represents the
original problem. So, the solution, that is, the potential of a point charge above an
infinite grounded conductor is given by Eq. (3.2d) for z  0.

Note that the role of uniqueness theorem is important in this calculation. Since
the solution satisfies Laplace’s equation in the region of interest and also the
boundary conditions for the system, it must be the correct and unique solution.
Without the uniqueness theorem, this solution would stand no chance! This is
because it has been obtained for a completely different charge distribution. But it
is acceptable because of the uniqueness theorem

To sum up, in this problem, we have replaced the original system of a charge
and an earthed conducting plane by the charge itself and an image charge; we
have not taken the conducting plane in the second system. The solution obtained
satisfies the boundary conditions. Although the solution is completely different for
the region below the plane, it does not matter to us since we are only interested
74
Unit 3 Special Techniques
in the region above the plane. The uniqueness theorem tells us that this is the
unique solution of the problem.

You can now determine the electric field, induced charge density and total
induced charge. Use cylindrical coordinates in your calculations in SAQ 1.

SAQ 1
Determine the electric field, induced charge density and total induced charge
for the charge placed near an earthed conducting plane as in Fig. 3.1.

Thus, you have found after solving SAQ 1 that the total induced charge has the
same magnitude as the real charge, though it is of opposite sign. Note that the You can watch the
lectures at the
image charges have the opposite sign. Thus, the x-y plane will always be at
following links to
zero potential. It has to be since the conductor is earthed! appreciate the
In the above description, instead of a single point charge near an earthed power of the method
conducting plane, we could have had any stationary charge distribution. It of images:
https://www.youtube
could be treated in the same way, by introducing its mirror image. The electric
.com/watch?v=eaXu
potential, electric field, induced surface charge density and total induced charge Ew3bnEQ;
can all be determined, once the potential is known (TQ 1).
https://www.youtube
To sum up, the method of images is essentially the technique of solving
.com/watch?v=Q8u
problems in electrostatics without having to solve Poisson’s or Laplace’s OpGvfkwU
equations. In this method, a given charge configuration near or above an infinite
perfect earthed conductor is replaced by the charge configuration itself, its mirror https://www.youtube
image and an equipotential surface instead of the conductor (in the above case .com/watch?v=jippP
surface was at zero potential). It involves using image charges to obtain the v6Gz14
electrostatic potential in the given region of space.
Always remember that the image charges have the opposite sign. Thus, the
surface near them will always be an equipotential surface. Also, all image
charges must lie outside the region of interest. Otherwise, the problem will
not remain the same.
You must understand that the method of images is not a general method in
the sense that we cannot apply it to every problem in electrostatics. We can
apply it only to a certain type of special problems. It is only useful in situations of
high symmetry for which we can actually find the size and position of the image
charges. In the next section we consider applications of this technique.

3.3 APPLICATIONS
In Sec. 3.2, you have studied an application of the method of images for a charge
located near an infinite grounded conducting plane. The technique works if there
are more than one infinite conducting planes but the calculation becomes a lot
more cumbersome. Let us now apply the method of images to a couple of
specific examples, that of two grounded, conducting planes perpendicular to
each other and a grounded conducting sphere.

Example 3.1
Consider two semi-infinite grounded, conducting planes that cover the y-z and
x-z planes and intersect each other along the z-axis (Fig. 3.3). In Fig. 3.3, the 75
Block 1 Electrostatics
z-axis points outwards of the plane of the paper, which is the x-y plane. A
positive charge Q is located at a point (a, b) in the region between the planes.
Use the method of images to calculate the electric potential due to the charge
 Q.

Solution : Do you see the similarity between this problem and the problem of
images formed by two plane mirrors intersecting at right angles in optics?
Using that knowledge, we can locate the image charges for this problem as
follows.
y

a Q

b
V 0
x

V 0

Fig. 3.3: A charge +Q is placed in the region between two semi-infinite conducting
planes.

Now refer to Fig. 3.4. The image charge due to the y  0 plane is formed at
(a,  b) and due to x  0 plane at ( a, b). Both these images have charge
Q. However, the image of these image charges is also formed at the point
( a,  b ). That has charge  Q.
y

Q a a Q

b b
x
b b
Q Q
a a

Fig. 3.4: Image charges for Example 3.1 with no conducting planes.

From UG physics courses, you know how to determine the electric


field/potential at an arbitrary point due to four charges placed at the corners of
a square/rectangle.

Note that the electric potential generated by the charge distribution of Fig. 3.4
is zero at every point on the y-z and x-z planes. Therefore, the electric
potential generated by this image charge distribution satisfies the same
boundary conditions as the electric potential of the original system, which are:

V  0 on the planes x  0 and y  0 (3.3)

The electric potential generated by the image charge distribution in the region
where x > 0 and y > 0 will be identical to the potential of the original system.
76
Unit 3 Special Techniques
The electric potential at a point P ( x, y , z ) is given by:

 Q Q 
  
2 2 2 2 2 2
 ( x  a)  ( y  b)  z ( x  a)  ( y  b)  z 
1  
V
4  0  
 Q Q 
  
 ( x  a )2  ( y  b )2  z 2 ( x  a )2  ( y  b )2  z 2 

(3.4)
You can verify that Eq. (3.4) satisfies the boundary condition (3.3) before
studying further. This is because the boundary condition implies that
V  0 at x   a, y  0 and V  0 at x  0, y   b.
Thus, Eq. (3.4) gives a unique solution for the electric potential. Notice how
simple it is to solve the problem using this technique! You may now like to
solve an SAQ. y

SAQ 2 V 0 Q
Suppose that the grounded conducting planes of Example 3.1 are at an angle
of 60 to each other (Fig. 3.5). Locate the image charges. 60
x
Example 3.2 V 0

A point charge Q is situated at a distance D from the centre of an earthed


conducting sphere of radius R (Fig. 3.6).
Fig. 3.5: Diagram for
SAQ 2.
+Q
R
D
D
V0

Fig. 3.6: A charge +Q at a distance D from the centre of an earthed conducting


sphere of radius R.

Solution : Let us first try to locate the image charge Q. From symmetry
considerations, the image charge should be on the line joining the charge Q and
the centre of the sphere. Let the image charge be located at z = d. Then the
electric potential due to the charges Q (at z  D ) and Q  (at z  d ) at some
point P ( x, y , z ) is given by:

1  Q Q 
V     (i)
4 0  r1 r2 
 
where the vectors r1 and r2 are shown in Fig. 3.7.
 
The magnitudes of r1 and r2 in terms of D, R and d are given by:


r1  r 2  D2  2rD cos 


and r2  r 2  d 2  2rd cos  (ii) 77
Block 1 Electrostatics
Now to determine Q  and d, we use the boundary condition that the potential is
zero at any point on the sphere. Then we have:

Q Q
V (R )   0
R 2  D2  2RD cos  R 2  d 2  2Rd cos 
(iii)
x

P (r )

  
r r2 r1
R
 DQ  +Q
z
O (0,0,d) (0,0,D)
D

Fig. 3.7: Locating the image charge for a charge +Q kept at a distance D from the
centre of an earthed conducting sphere of radius R.

or
1/ 2 1/ 2
Q   Q  
2 2
R d 
cos   cos  
2R 2d
1      1   
D  D D  R  R R 
   

You can verify that this equation is satisfied for all values of D and R only if

R R2
Q   Q and d  (iv)
D D

Eq. (iv) gives us the value of the image charge and its position.

Therefore, the potential is:

  R Q 
   
V (r , , ) 
1  Q
  D  
4  0  r 2  D 2  2rD cos  R 4 R 2 
 r2   2r cos  
 D2 D 

or
 
 
1  Q Q 
V (r , , )    
4 0  r  D  2rD cos 
2 2 2 
 rD  2
    R  2 rD cos  
 R 
(v)

Let us check if the potential is zero on the surface of the sphere for the values
obtained in Eq. (iii). Substituting r  R in Eq. (v), we get:

1 Q Q
V   0
4  0 R 2  D 2  2RD cos  D 2  R 2  2RD cos 
78
Unit 3 Special Techniques
So, the solution given by Eq. (v) satisfies the boundary condition for the problem.
From uniqueness theorem, this is the only solution.
Let us now determine the electric field at any point outside the sphere (inside the
conducting sphere, it is zero as you know from your UG physics). For the sphere,
it is given by:
 V
E  rˆ
r
 
 
 1  Q Q 
  rˆ   
r 4 0  r 2  D2  2rD cos  2 
 rD  2
    R  2 rD cos  
 R 

 
 rD2 
   2  D cos  
Q  r  D cos 
E   rˆ   R 
 
or
4  0  2 2 3/2
 
3/2 
 r  D  2rD cos 
2 2
 r D  R 2  2 rD cos   
  R2  
   

For the surface charge density, we take the normal component of the electric field at
r  R and write:

 
 rD2 
  2  D cos  
Q  r  D cos 
  0 Er     R 
4  2  2
 r  D  2rD cos 
3/2

r D
2 2
 R 2
 2 rD cos 


3/2 

  R2  
   r  R

 D2 
   D cos  
Q  R  D cos 
   R 
4  2  2
 R  D  2rD cos 
3/2 2
 2

D  R  2 rD cos 
3/2 
 
 

 
Q  D2  R 2 
  
 
Thus,
4R  2 2 3/2 
 R  D  2rD cos  

So far, we have applied the method of images to discrete charges placed near
conductors. We will now consider an application of the method of images to a
continuous charge distribution placed near a conducting surface.

Example 3.3

A charged infinite straight wire having uniform linear charge density  is situated
at a distance d above a grounded conducting plane (Fig. 3.8). Determine the
electric potential and the electric field in the region above the plane.

Solution : Let us suppose that the conducting plane is in the x-y plane and the
wire is parallel to the x-axis directly above the grounded plane (Fig. 3.8). 79
Block 1 Electrostatics
The boundary condition for this problem is that:
V ( z  0)  0
z

Fig. 3.8: Line charge above a grounded conducting plane.

We redraw Fig. 3.8 showing the image line charge of the given line charge
(Fig. 3.9). Note that we are viewing the line charge from its cross-section. You
have to visualize the x-axis perpendicular to the y-axis and the line charge
parallel to it.
z
P (y ,z)
r1


d r2
O
y
d


Fig. 3.9: Line charge above a grounded conducting plane and its image.

Now, you know from the results of Sec. 1.3.2 of Unit 1 and UG physics that the
potential difference due to this line charge at points O and P (y, z) is given by:
2
 r   r 
V (r )   ln   ln  (3.5)
2 0  d  4 0  d 

You will appreciate it in a moment why we have written Eq. (3.5) in this form. The
total potential difference due to the line charge (V1 ) and its image (V2 ) is the
sum:
2 2
 r   r 
V (r )  V1  V2   ln 1   ln 2  (3.6)
4 0  d  4 0  d 
2
 r 
or V (r )  ln 2 
4 0  r1 

  y 2  ( z  d )2 
or V  ln   (3.7)
4 0  y 2  ( z  d )2 

You can verify that the boundary condition [V ( z  0)  0] is satisfied by this


potential. So, the potential (potential difference) in the region above the plane
and the line charge is given by Eq. (3.7).
80
Unit 3 Special Techniques
SAQ 3
Suppose that there is a positive infinite line charge of charge density  instead
of a single charge in Example 3.1. Determine the electric potential.

As you would have realised by now, we can also calculate the electric field
and electrostatic force experienced by a charge using the method of images.
We end this unit with such an example of calculating the electrostatic force on
a charge in a system of two charges located above a conducting plane.

Example 3.4
Two point charges  2q and  q are placed at distances d and 2d above a
grounded conducting plane at z  0 (Fig. 3.10a). Obtain the electrostatic force
on the charge  q and the electric field.
z z

2d  q 2d  q

d  2q d  2q

y y

d  2q
x x
2d  q

(a) (b)

Fig. 3.10: a) Two charges above a grounded conducting plane; b) mirror images of
the charges.

Solution : We place the mirror charge  2q at z   d and  q at z   2d


(Fig. 3.10b). Then the electrostatic force on the charge  q is:

 q   2q 2q q  q2  2 1 
F     zˆ   2    zˆ
4 0  d2 (3d )2 ( 4d )2  4 0d 2  9 16 

1  265q 2 
or F    zˆ
40  144d 2 

The electric field due to the charge  q is:

1  265q 
E    zˆ
40  144d 2 

So, you must appreciate now that the method of images is a very powerful
technique for obtaining electric potentials, electric fields, electrostatic forces,
charge densities and the total charge for specific systems. We would like to
put in a word of caution here about using this method to solve potential
problems. We cannot obtain the electric potential in the region or on the 81
Block 1 Electrostatics
surface that surrounds the image charge. The electric potential is given by
the method of images only in the region where the actual physical
charge(s) reside. This is the region of interest for such problems and no
image charges should be present in this region. Similarly, the electric field is
given only in the region of interest.

3.4 MULTIPOLE EXPANSION


So far, you have learnt the methods of determining the electric fields and
electric potentials for a variety of charge distributions in both outside and
inside the regions where these are. For example, you have done direct
calculation Coulomb’s law, used Gauss’s law for symmetrical charge
distributions and solved Laplace’s and Poisson’s equations for given boundary
conditions. We have given select examples because even when charge
distributions are symmetrical, it could involve a lot of calculations. Sometimes,
we may not need a very precise information about the electric potential or
electric field. In such situations, we take recourse to the multipole expansion of
potentials, which we now discuss.

Let us consider a point very far away from the charge distribution. Then the
potential is to a good approximation,

1 Q
V (r )   , (3.8a)
40 r

where Q is the total charge. If the reference point for potential is at infinity,
then we also have:

1  d 
V (r ) 
40  r1
, (3.8b)

where  is the volume over which integration is being carried out [see
Fig. 3.11 for the variables defined in Eq. (3.8b)].

P
r1
d
r
r 

Fig. 3.11: Potential at a point P due to a charge distribution.

We now use the law of cosines:

r12  r 2  r 2  2rr  cos  (3.9a)

 r 
2
r 
 r 2  1     2 cos  
 r  r 
82  
Unit 3 Special Techniques
or r1  r (1  ) (3.9b)

 r  r 
where       2cos   (3.9c)
r r 

For a point P far away from the charge distribution,  << 1.

Let us now rewrite the term r1  r (1  ) as:

1 1
 (1  )1/2 (3.10a)
r1 r

and expand it binomially. Then

1 1 1 3 5 3 
  1    2    ...  (3.10b)
r1 r  2 8 16 

Substituting for  in Eq. (3.10b), we get:

1 1 1 rr  3 r r


2

2
 1    2 cos        2 cos  
r1 r  2 r  r  8 r   r 

5  r  3  r  
3 
     2 cos    ...
16  r   r  

We now collect the like powers of ( r  / r ) and write:

1 1 r r 2 3
 1  cos      cos 2   
1
r1 r  r   
r 2 2 

r 3 5 
    cos 3   cos    ...
3
(3.11a)
 r  2 2  

Do you recognise the coefficients of powers of ( r  / r )? These are Legendre


polynomials (see Unit 4, MPH-001) and so we have:

1 1 
2 3
r  r  r
 P0 (cos )  P1(cos )    P2 (cos )    P3 (cos )  ...
r1 r  r r  r  

(3.11b)

Thus, we can write:

1 1   r
n
  Pn (cos ) (3.12)
r1 r n 0  r 

Here  is the angle between r and r . Substituting Eq. (3.12) in Eq. (3.8b) and
since r is constant in the integral, we get:

 
1   1

40 n  0  r n  1 
  n 
n
V (r )  ( r  ) r  P (cos  ) d  (3.13)

   83
Block 1 Electrostatics
Writing Eq. (3.13) in its expanded form, we have:

1 1
40 r 
V (r )  [ (r ) d  Monopole

1

r2 
 r  cos (r ) d  Dipole

1 23 1
 r   2 cos   2  (r ) d 
2
 Quadrupole
r 3
  

 ... ] (3.14)

Eqs. (3.13a and b) give the multipole expansion of the electric potential in
powers of 1/r. Note that the first term corresponding to n  0 is the monopole
contribution to the potential, which varies as 1/r. The second term
corresponding to n  1 is the dipole contribution and it varies as 1 r 2 . The
third term corresponding to n  2 is the quadrupole contribution and it varies
as 1 r 3 , and so on. Eq. (3.13) is a precise expression of the potential. At large
distances or large values of r, the monopole term approximates the potential
quite well. Successive terms can be retained for greater precision.

With this discussion of multipole expansion, we end the unit and summarise its
contents.

3.5 SUMMARY
In this unit, we have discussed the following concepts:

 The special technique called the method of images to determine the


electric potential, electric field, electrostatic force, charge density and
total charge of specific systems of charges.

 The method of images cannot be applied to any arbitrary system of


charges. Rather, it applies to specific situations in which charges are
situated near a conducting equipotential surface and it is possible to
locate their mirror images. Several applications of the method of
images have been taken up to demonstrate this special and powerful
technique.

 The technique of multipole expansion of the electric potential in terms


up to the desired precision may be retained.

3.6 TERMINAL QUESTIONS


1. Suppose in Fig. 3.1, two charges  Q and  2Q are placed at a distance of
d and 2d, respectively, near the earthed infinite conducting plane.
Determine the electric potential using the method of images for the
system.
84
Unit 3 Special Techniques
2. A charge Q is placed in front of a conducting sphere of radius R which is
maintained at a constant potential. Determine the potential at points
outside the sphere.

3.7 SOLUTIONS AND ANSWERS


Self-Assessment Questions
1. The electric field is given by:

  V V
E   V   ˆ  kˆ
 z

Q    
  [2  ( z  D )2 ]3 / 2  [2  ( z  D )2 ]3 / 2  ˆ
4  0  

Q  zD zD ˆ
   k
4 0  [  ( z  D) ]
2 2 3 / 2 [  ( z  D ) ]
2 2 3 / 2

(i)

Remember that this expression for the electric field is valid only for z  0.
We can also determine the induced charge density and the charge induced
on the surface. Recall that   E .nˆ, and so we only need to evaluate the z
component of the electric field at z  0. Thus,

Q  D D 
induced  0 Ez ( z  0)   2  
4 2 3/2
 [  D ] [  D 2 ]3 / 2
2


Q  D 
  2  (ii)
2  [  D ]
2 3 / 2


We can determine the total induced charge by integrating the induced charge You can watch the
density in the entire x-y plane: lectures at the
following links to
  appreciate the
QD 2 d QD
Qinduced    induceddxdy  
2 
2 [  D ]2 3 / 2

[  D 2 ]1 / 2 0
2
 Q power of the method
of images:
0
https://www.youtube
.com/watch?v=eaXu
Thus, the total induced charge has the same magnitude as the real charge,
Ew3bnEQ;
though it is of opposite sign. Note that the image charges have the opposite
sign. Thus, the x-y plane will always be at zero potential. It has to be since https://www.youtube
the conductor is earthed! .com/watch?v=Q8u
OpGvfkwU
2. The image charges along with the charge  q forms a regular hexagon of side
d, as shown in Fig. 3.12: https://www.youtube
.com/watch?v=jippP
the image of  q at A is  q at B, v6Gz14

the image of  q at B is  q at C,

the image of  q at C is  q at D, 85
Block 1 Electrostatics
the image of  q at D is  q at E
and
the image of  q at E is  q at F.
z

q B

E A
q q
60

D q q
F

q C

Fig. 3.12: Locating image charges for a charge kept between two grounded
conducting planes at an angle of 60.

3. See Fig. 3.13a. Instead of the point charge  Q, we have an infinite line
charge of charge density  . The image charges will be line charges of
charge density   each at (a,  b) and (  a, b ).

Also, the image of these image line charges is a line charge of charge
density   at ( a,  b ). These are shown in Fig. 3.13b. As you know, the
electric potential due to a line charge parallel to z-axis is independent of
the z coordinate and depends only on the distance  of the point from the
line charge.
y y

a a a 
  

b b b
V 0
x x

V 0 b b
  
a a

(a) (b)

Fig. 3.13: a) An infinite line charge is placed in the region between two
semi-infinite conducting planes; b) Image line charges.

At a point P(x, y, z), it is given by:

  C2 
V ( x, y )  ln 2  (i)
4  0   

where C is a constant and 2  ( x  a)2  ( y  b)2 . Thus, we can write the


total electric potential due to the four line charges as:
86
Unit 3 Special Techniques

 C2 C2 
ln  ln 
 ( x  a )2  ( y  b )2 ( x  a )2  ( y  b )2 
  
V ( x, y ) 
4  0  
 C 2
C 2 
 ln  ln 
 ( x  a )2  ( y  b )2 ( x  a )2  ( y  b)2 

(ii)
You can verify that the electric potential given by Eq. (ii) satisfies the
boundary conditions:
V (0, y )  V ( x, 0)  0 (iii)

Also verify that the tangential component of the electric potential on the
V
y-plane is zero, i.e.,   0. The tangential component of the
x y  0
V
electric potential on the x-plane is also zero, i.e.,   0.
y x  0

Terminal Questions
1. See Fig. 3.14a, which shows two charges of magnitude Q and 2Q placed
at the distances d and 2d, respectively, from an earthed infinite conducting
plane. Fig. 3.14b shows the image charges.

So, following Example 3.1, the electric potential at a point P(x, y, z) is:

Q 2 1 1 2 
V      (i)
4  0  r1 r2 r3 r4 

where r1  x 2  y 2  ( z  2d )2 (ii)

r2  x 2  y 2  ( z  d )2 (iii)

r3  x 2  y 2  ( z  d )2 (iv)

r4  x 2  y 2  ( z  2d )2 (v)

z z

+2Q +2Q
d d
+Q +Q
d d
y y
d
Q
d
x x  2Q

(a) (b)

Fig. 3.14: Diagram for Terminal Question 1. 87


Block 1 Electrostatics
Hence,
 2 1 
 2  
2 2
 x  y  ( z  2d ) x 2  y 2  ( z  d )2 
Q  
V 
4  0  
 1 2 
  
 x 2  y 2  ( z  d )2 x 2  y 2  ( z  2d )2 

(vi)

2. You have seen in Example 3.2 that when the sphere is maintained at zero
R
potential, we can introduce an image charge Q   Q at a distance
D
R2
d  from the centre of the sphere.
D

If we want the sphere to be maintained at a constant potential, we can


imagine an additional image charge Q at the centre of the sphere which
Q
keeps the potential constant at . Equating this to the constant potential
4 0R
V0 , we take this charge at the centre to be equal to 4  0RV0 .

The potential at an arbitrary point located at a distance r from the centre is


4 0RV0
obtained by adding the term to the potential obtained in
r
Example 3.2 which is then given by:
 
 R 
  Q
1 Q 4 0RV0 
V ( r , )      D  
4  0  r  D  R2  r 
 r  2D 
 D 
 

88
Unit 4 Dielectric in Electric Field

UNIT 4
DIELECTRIC IN
ELECTRIC FIELD
Structure

4.1 Introduction 4.4 Electric Field of a Polarised


Expected Learning Outcomes Object
4.2 Dielectrics Physical Interpretation of Bound
4.3 Dielectric Material in Electric Field: Charge Density
Polarisation 4.5 Electrostatic Equations in
Induced Dipoles in Neutral Atoms and Dielectrics:
 Displacement Vector
Non-polar Molecules D and Gauss’s Law  
Alignment of Polar Molecules in 4.6 Boundary Conditions on E and D
Electric Fields 4.7 Summary
Polarisation of Dielectrics and 4.8 Terminal Questions

Polarisation Vector P 4.9 Solutions and Answers

4.1 INTRODUCTION
In the previous three units, you have studied electrostatics in free space. You
have learnt the concepts of electrostatic force, electric field and electric potential
when charges are placed in vacuum. You also learnt techniques for determining
potential due to charge distributions such as the method of images and
multipole expansion of potential. However, in real life, we mostly have situations
in which electric phenomena take place in matter. Matter, as you know, can be
in any form: solid, liquid or gas. Different kinds of matter behave differently in
the electric field. You know from undergraduate physics that we can broadly
classify most materials around us into two categories namely, conductors and
insulators, on the basis of their electrical properties. Insulators are also called
dielectrics. In this unit, you will study how dielectric materials behave in the
presence of electric fields and learn how Gauss’s law is modified in a dielectric
medium.

You may like to know: Why do we need to study about behaviour of


dielectrics in the presence of electric field? This is what we shall explain in
the beginning of this unit when we introduce dielectrics (Sec. 4.2). In Sec. 4.3,
we shall use a simple model of dielectric materials to explain what happens
when a dielectric is placed in an external electric field. You will learn that this
results in the phenomenon
 of polarisation of dielectrics. We shall
 define
electric polarisation P and introduce the displacement vector D to determine
the electric field in a dielectric material. In Sec. 4.4, we shall obtain the electric 89
Block 1 Electrostatics
field due to a polarised object and explain its physical meaning. In Sec. 4.5, we
discuss how Gauss’s law gets modified in a dielectric medium. Finally,
 in
Sec. 4.6, we obtain the conditions which the electric field vector E and electric
displacement vector D must satisfy at the boundary of two different media.

In Unit 5, you will study about microscopic properties of dielectrics in electric


field.

Expected Learning Outcomes


After studying this unit, you should be able to:
 explain the behaviour of dielectrics in an electric field;
 define electric polarisation and explain the mechanism of polarisation in polar
and non-polar dielectrics;

 define displacement vector D and deduce Gauss’s law in a dielectric
medium;
 
 relate D to the electric field E;
 
 obtain boundary conditions for E and D ; and
 define dielectric constant.

4.2 DIELECTRICS
Dielectrics (or insulators) are an important class of materials used in a variety
of applications such as in electrical insulation, capacitors, radio frequency
transmission lines, printed circuit boards, etc. The study of dielectrics helps us
understand how a proper dielectric is chosen for a capacitor, as well as many
optical phenomena such as reflection, refraction and double refraction in
quartz or calcite crystals. Natural rubber, cotton, wood are some examples of
good electrical insulators. Paper, mica, glass and a large number of plastics
are good dielectrics which are used in capacitors. You may have studied in
Fig. 4.1: Spherical
capacitors used by
your earlier physics courses that dielectrics are used in capacitors to
Faraday. Faraday showed increase their capacitance manifold. Why does this happen? Let us find
that when dielectric out why do we need to learn about dielectrics by answering this question.
material was placed
between the central brass The increase in capacitance due to the presence of dielectric was
ball and a concentric brass demonstrated by Faraday in 1837. He showed that when a slab of dielectric
shell, the capacitance of material (such as glass or mica) was introduced between a central ball and a
the spherical capacitor
concentric brass shell of a spherical capacitor (see Fig. 4.1), its capacitance
increased manifold. The
factor by which it
increased manifold (by a factor called the dielectric constant). The value of
increased was different for this factor is 1 for vacuum and greater than 1 for various dielectrics. The
different dielectric dielectric constants for a few materials are given in Table 4.1.
materials. (Source:
Collectionsonline.nmsi.ac.uk) The dielectric constant is one of the important macroscopic electrical
properties of a dielectric material and its value varies widely for different
dielectrics. For example, for water, it is 80.4 and for different types of glass, it
is around 6. So the capacitance increases according to the dielectric being
used in it. The choice of a dielectric in a capacitor depends on the application
for which it is to be designed.
90
Unit 4 Dielectric in Electric Field

To explain this phenomenon, let us consider a parallel plate capacitor with


some free charge q on its conducting plates (Fig. 4.2). Let us assume a
negative charge on the upper plate and a positive charge on the lower plate. If
A is the area of the plates and d the distance between them, you know from Table 4.1: Dielectric
undergraduate physics that the capacitance of the capacitor is given by: constants of some
common materials.
A
C  0 (4.1) Material Dielectric
d constant

and a charge q on the plates results in a potential difference given by: Air 1.0006

q Mica 59
V  (4.2)
C Glass 4.5  7.00

It is an experimental fact that if we put a dielectric slab between the plates of a Paper 2 2.3
capacitor, we find that the capacitance increases. As you can see from Water 80.4
Eq. (4.2), an increase in capacitance means that the potential difference
between the plates decreases. Let us try to understand this using the concepts
you have studied in Unit 1.

Free charges Conductor


                     
S

        
d 
E Induced charges
Dielectric
        

                     
Free charges Conductor

Fig. 4.2: A parallel plate capacitor with a dielectric material inserted between its
plates.

Refer to Fig. 4.2. Consider a Gaussian surface S (a rectangular box lying


partially inside the dielectric material and partially inside the conducting plate)
as shown in the figure. Recall from Unit 1 that Gauss’s law tells us that the
electric flux out of the surface is related to the enclosed charge qencl as:
  qencl
 E.dS  0
S

You know that potential difference or voltage is proportional to the electric


field. Therefore, when capacitance increases, the electric field must decrease
(because potential difference has decreased). From Gauss’s law, this, in turn,
implies that the charge should decrease. This can happen only if a positive
charge has appeared on the upper surface of the dielectric. This positive
charge, of course, has to be smaller than the negative charge placed on the
plate of the capacitor.
We can explain the increase in capacitance due to a dielectric material only if
we answer the question: How is a positive charge induced on one surface of a
dielectric material and a negative charge on the other surface when it is placed 91
Block 1 Electrostatics
in an electric field? This requires an understanding of the behaviour of
dielectric materials in an electric field, which we now discuss. In doing so, we
arrive at an understanding of polarisation of dielectrics in electric fields.

4.3 DIELECTRIC MATERIAL IN ELECTRIC


FIELD: POLARISATION
You have learnt in your undergraduate physics course the behaviour of
+ + + conductors which have free electrons to conduct electricity when they are
placed in electric field. You know that the electric field inside a conductor is
+ + + zero and the total charge of a conductor resides on its surface. The electric
field due to this charge is normal to the surface of a conductor. Also the
+ + + surface of a conductor is an equipotential surface.
What can you say about dielectric materials? We use the following simple
+ + +
model for describing dielectric materials (see Fig. 4.3):
+ + +  Like all matter, dielectrics are made up of a large number of atoms and
molecules.
+ + +
 Every atom consists of positively charged nucleus and negatively charged
electron cloud distributed around it.
Fig. 4.3: Model of a
dielectric made up of  The total positive charge is equal to the total negative charge so that the
atoms. The charges in atom/molecule is electrically neutral.
the dielectric are not
free to move around;  In contrast to conductors, the charges attached to atoms and molecules
they are bound to the are not free to move around in dielectric materials: at most they can move
atoms. The + sign within the atom/molecule. The charges are bound in the atoms and
depicts the positive molecules of a dielectric.
nucleus and the grey
sphere represents the  A molecule may be made of atoms of similar kind or of a different kind.
negatively charged
electron cloud. We will now use this model to understand the behaviour of dielectrics in
electric fields. Let us ask ourselves: What happens when we put a dielectric
material in an external electric field?

Since the dielectric is made up of atoms and molecules as described above, to


answer this question, we need to first understand: What happens to a neutral
atom or molecule of the dielectric when it is placed in an external electric field?
The behaviour of a dielectric in an electric field depends on whether it is made
up of neutral atoms/non-polar molecules or polar molecules.
Broadly there are two types of dielectrics: non-polar dielectrics made up of
neutral atoms or non-polar molecules and polar dielectrics made up of
polar molecules. You will learn about non-polar and polar molecules in the
next two sections.
There are two mechanisms by which an external electric field affects the
charge distribution in a dielectric:
1. By inducing dipoles in neutral atoms or non-polar molecules in a non-
polar dielectric.
2. By aligning the permanent dipoles of the polar molecules in a polar
92 dielectric.
Unit 4 Dielectric in Electric Field

We now explain both these mechanisms.

4.3.1 Induced Dipoles in Neutral Atoms and Non-polar +


Molecules
(a)
To find out what happens
 when a dielectric comprising neutral atoms is placed
in an electric field E, let us consider the following model of an atom
+
(Fig. 4.4a): 
d p 
E
 the positive nucleus is present at the centre of the atom,
 the negatively charged electrons are distributed in a spherical cloud about
the centre, (b)
 the centre of positive charges and the centre of negative charges coincide, Fig. 4.4: a) An atom with a
and positively charged nucleus
and a cloud of negatively
 the atom is electrically neutral and also has no electric dipole moment.
charged electrons such that
When the neutral atom is placed in an external electric field, the two regions of the centres of the positive
and negative charge
positive and negative charges within the atom are influenced by the electric
coincide; b) In the presence
field: the positive nucleus is displaced in the direction of the field and the of an electric field, the
negatively charged electrons in the atom are displaced in the opposite centres of positive and
direction. But the positive and negative charges also attract each other, and negative charge in the atom
no longer coincide. These
the atom is held together. If the electric field is very large, it ionizes the atom.
are separated and an
If the external electric field is not very large, equilibrium is soon established. induced dipole appears.

The two opposing forces, one due to the external electric field pushing the
positive nucleus and the electron cloud apart, and the other due to their mutual
electrostatic attraction pulling them closer, reach a balance. When this
happens, the centre of positive charge is shifted slightly in one direction and
the centre of negative charge is shifted in the opposite direction. This results in
a small separation of the centres of positive and negative charges and an
induced dipole appears (Fig. 4.4b).
Thus, a dipole moment is induced in the neutral atom in the presence of an

external electric field E . The dipole moment is in the same direction as the
electric field. Let us determine the expression of the induced dipole moment.
Suppose d is the distance between the centres of positive and negative
charges in the atom. Then the dipole moment of the atom is given by:
 
p  q d nˆ  q d (4.3)

where n̂ is a unit vector in the direction of d and points in the same direction
 
as E . Typically, the displacement d is proportional to the external electric field
unless the fields are very large. (Inthat case, it could
 even result in the
ionization of the atom.) Since d  E (as long as E is not too large) and
 
p  q d, the induced dipole moment is proportional to the electric field:
   
p  E and p   E (4.4)

The constant of proportionality  is called the atomic polarisability and its


unit is C 2mN 1 . Its value depends on the structure of the atom. Let us take up
an example to estimate the atomic polarisability of an atom using Eq. (4.4). 93
Block 1 Electrostatics
Example 4.1
a
+ Electron
cloud Consider an atom of radius a in the presence of an external electric field E
(Fig. 4.5). Calculate the separation between the positive nucleus and the
centre of the negatively charged electron cloud and its dipole moment.
Positive nucleus Calculate its atomic polarisability and estimate its value for the hydrogen atom
(a) of radius a0  10 10 m . Take the value of E to be 10 6 V m 1 .

Solution: You have learnt that in the presence of an external electric field,
the positive nucleus is pulled opposite to the centre of negative charge. For
b
keeping the calculation simple, we assume that at equilibrium, the negative
 +
p charge cloud keeps its spherical shape and is merely displaced by an amount
b with respect to the positive nucleus (see Fig. 4.5b).
 At equilibrium, the force
 on the nucleus due to the external electric field E is balanced by the attractive
E
force due to the negative charge
 cloud. The force on the nucleus due to the
(b)
external electric field is  qE, where q is the charge on the nucleus.
Fig. 4.5: a) A neutral
atom of radius a; b) A
Now, let us calculate the value of the electric field E e due to electron cloud at
dipole is induced in the new location of the nucleus. To do so, recall from undergraduate physics
the atom in the that the electric field due to a uniformly charged non-conducting sphere at an
presence of an internal point is given as:
electric field. We
assume that at 1 qb
Ee 
equilibrium, the 4  0 a 3
electron cloud is
spherical in shape. where b is the distance between the centre of the electron cloud and the
nucleus, q is the magnitude of the total charge of the electron cloud and a is
the radius of the uniformly charged spherical electron cloud.
Thus, at equilibrium when the force on the nucleus due to external field is
balanced by the attractive force between the nucleus and the electron cloud,
we can write the magnitude of the external electric field as
E  Ee

1 qb
or E
4 0 a3

4 0a3E
or b
q

From Eq. (4.3), magnitude of the dipole moment p  qb  4  0 a 3 E.

Therefore, from Eq. (4.4), the atomic polarisability is given by:


p
   4  0a 3
E
For the hydrogen atom, a  a0  1010m. Let us estimate the atomic
polarisability of the hydrogen atom based on this crude model. It is:
  4  8.85  1012  1030 C2 mN1  1.11 1040 C2 mN1

The separation b in the presence of a modest electric field E  106 Vm 1 is


4  8.85  10 12  10 30  10 6
b m  6.9  1015 m
94 1.6  10 19
Unit 4 Dielectric in Electric Field
Although this atomic model is very crude, the estimated value of the atomic
polarisability given by Eq. (4.4) based on this model is not too bad. It is accurate
to within a factor of four for simple atoms. Compare the value of  obtained from
this crude model with its experimental value for the hydrogen atom. In units of

1030 m3 , the experimental value of for the hydrogen atom is 0.667.
4 0

So far we have seen how an electric dipole moment is induced in a neutral atom
in the presence of an external electric field. What happens in the case of a
molecule? Let us find out.
In one type of molecules called non-polar molecules, the centres of positive
and negative charges always coincide. Such molecules have zero dipole
moment in the absence of external electric fields. The dielectrics made up of
such molecules are called non-polar dielectrics. Some examples of non-polar
molecules are air, hydrogen, oxygen (Fig. 4.6), benzene, carbon tetrachloride,
etc.
  
  
    
    
    
    
   
  

  
  
    
     Centres of positive charges and
     negative charges coincide
    
   
  

Fig. 4.6: In an oxygen molecule, the centres of positive and negative charges
coincide and it has zero dipole moment in the absence of an external
electric field.

Thus, the non-polar molecules do not possess any permanent dipole moment
(their dipole moment is zero in the absence of an external electric field).
Hence, you can immediately say that their behaviour in the presence of an
external electric field should be the same as that of a neutral atom.
So we find that when a neutral atom or non-polar molecule is placed in an
external electric field, it acquires (by induction) a tiny dipole moment in the
direction of the electric field. You may ask: Are there molecules in which the
centres of positive and negative charges do not coincide? What happens to
such molecules when they are placed in an electric field? We will discuss it
now.

4.3.2 Alignment of Polar Molecules in Electric Fields


The arrangement of atoms in some molecules is such that the centres of
positive and negative charges do not coincide. Such molecules, e.g., water
glass and hydrogen chloride, are electrically neutral but have electric dipole
moments even in the absence of external electric fields and are called polar
molecules. We say that polar molecules have permanent dipole moments.
Dielectrics made up of such molecules are called polar dielectrics. 95
Block 1 Electrostatics
Let us study the simple structure of the hydrogen chloride (HCl) molecule
H shown in Fig. 4.7. It is a diatomic molecule made up of dissimilar atoms H and
Cl. Originally, the H and Cl atoms are spherical. When the HCl molecule is

p formed from these atoms, the electron of the H atom shifts partially over to the
Cl Cl structure, leaving the positive hydrogen nucleus behind. Thus, there is an
excess of negative charge at the chlorine end and an excess of positive
Fig. 4.7: The hydrogen charge at the hydrogen end of the molecule. This separation of the centres of
chloride molecule in positive and negative charge gives rise to a permanent dipole moment in the
which the positive and
HCl molecule.
negative charge centres 
do not coincide and the When polar molecules are placed in an external electric field, the force F on
molecule has a

the positive charge will exactly cancel the force F on the negative charge.
permanent electric dipole
moment.
But, as shown in Fig. 4.8, these forces form a couple. Therefore, the electric
dipole associated with the molecule would experience a torque that will tend
to align it along the electric field. 
F

 q
E

q


F
Fig. 4.8: A polar molecule experiences a torque arising due to forces on the
separated positive and negative charges in the presence of external
electric field.

In Secs. 4.3.1 and 4.3.2, you have learnt how neutral atoms/non-polar
molecules and polar molecules behave when they are placed in an external
electric field. With this background knowledge, we are now in a position to
understand as to what happens when a dielectric material is placed in an
external electric field.
4.3.3 Polarisation
 of Dielectrics and Polarisation
Vector P
You now know that there are two types of dielectrics, non-polar dielectrics
made up of neutral atoms/non-polar molecules and polar dielectrics made up
of polar molecules. So, we need to actually understand: How do the polar and
non-polar dielectrics behave in the presence of an external electric field? On
the basis of what you have learnt in the previous sections, we can summarise
the answer as follows:
 Non-polar dielectrics: When a non-polar dielectric, which is made up of
neutral atoms or non-polar molecules is placed in an external electric field,
the atoms/non-polar molecules of the dielectric acquire a tiny dipole
moment in the direction of the electric field.
 Polar dielectrics: When a polar dielectric, which is made up of polar
molecules (having permanent dipole moments) is placed in an external
96 electric field, the permanent dipole moments of the polar molecules
Unit 4 Dielectric in Electric Field

experience a torque tending to align them in the direction of the electric


field.

Thus, both mechanisms (acquiring dipole moments in neutral atoms/non-polar


molecules and alignment of permanent dipoles in polar molecules) produce
atomic/molecular dipoles in the dielectric pointing along the direction of the
field. We say that the dielectric material is polarised. Note that the dielectric
as a whole remains electrically neutral and inside the dielectric slab there is no
excess charge in any volume element.

However, the atoms/molecules are constantly in random thermal motion and


collide with each other. Therefore, alignment of the electric dipoles is not
complete and increases as the electric field is increased or the temperature
decreases. The random motion of the molecules tends to destroy the
alignment of the dipoles at higher temperature, and particularly when the
electric field is removed. Thus, to sum up, qualitatively we can say that the
polarisation of a dielectric due to an applied external field results from
 Dipole moments that arise due to relative displacement of the centres of
negative and positive charges in neutral atoms/non-polar molecules in a
non-polar dielectric; or
 The alignment of permanent dipoles (in polar molecules) in polar dielectric
material. In a homogeneous
Let us now obtain a quantitative definition of the polarisation of a dielectric. dielectric, its properties
e.g., permittivity and
Let us consider a homogeneous and isotropic dielectric slab. This means that
susceptibility, are the
the properties of the dielectric are the same at all points and in all directions. same at all points in

Let the dielectric slab be placed in an external electric field E0 . The external the dielectric, i.e., they
electric field could be applied by any means, e.g., due to charges on the plates do not vary with
position. In an isotropic
of a parallel plate capacitor as shown in Fig. 4.9a. If the dielectric material is
dielectric, its properties
made up of neutral atoms/non-polar molecules, a dipole moment will arise in are the same in all
each atom or molecule of the dielectric. If the dielectric material is made up of directions.
polar molecules, each permanent dipole would experience a torque tending to
align it along the electric field. However, the direction of the dipole moments in
either case will be the same as that of the electric field.
        
  

         P 
      

       
   
         E E 


        
       
         


        
       
         

 
E0 E0
(a) (b)

Fig. 4.9: a) In a dielectric slab placed in an external electric field E0, the centres
of positive and negative charges are separated and it gets polarised;
b) the separation of charges produces surface charges on the slab
 
faces, which set up a field E  opposite to E0 . 97
Block 1 Electrostatics
The separation of the centres of positive and negative charges produces
surface charges on the faces of the dielectric slab as shown in Fig. 4.9b.  The
surface charges on the dielectric faces produce an electric field, say E  inthe
direction opposite to the external electric field. The resultant electricfield E 
inside the dielectric is given by the vector sum of the electric fields E0 and E  .

It is in the same direction as E0 but smaller in magnitude.

To describe this phenomenon mathematically, we define the polarisation P
as the total dipole moment per unit volume:

P  Dipole moment per unit volume
Thus, polarisation is simply the mean dipole moment averaged over a large
volume that contains a very large number of atoms/molecules. It is thus an
average macroscopic property of the dielectric, which is a large scale
manifestation of the electric dipole moments of the atoms and molecules the
dielectric is made up of. If there are N polarised molecules per unit volume in
the dielectric, we have
 
P  Np (4.5)

For an ideal, homogeneous and isotropic dielectric, polarisation P is
proportional to the electric field E in the dielectric and we can write
   
P E or P   0 E (4.6)
The constant of proportionality  in Eq. (4.6) is called the electric
susceptibility of the dielectric material/medium. It is a macroscopic property
of the material and depends on the microscopic structure of the medium. It is a
measure of the extent to which a dielectric is polarised by an external electric
field. The greater the susceptibility of the dielectric, the greater is the
polarisation of the material in response to the electric field, thereby reducing
the electric field inside the material.
The constant 0 appears in Eq. (4.6) so that  is dimensionless. Dielectric
materials that satisfy Eq. (4.6) are called linear dielectrics.
Eq. (4.6) is found to be experimentally true for many substances, provided that
the electric field is not too strong. Eq. (4.6) tells us that the susceptibility of a
dielectric provides a measure of the extent to which it can be polarised when it
is kept in an external electric field. The susceptibility  of a dielectric depends
on the microscopic structure of the material  and also on external factors such
as temperature. Note that in Eq. (4.6), E is the net electric field in the
dielectric. It is the resultant of the electric fields due to both free charges and
the polarisation of the dielectric. So, if we put a dielectric material in an
external electric field E0 , we cannot calculate P directly from Eq. (4.6).
This is because the external electric field will polarise the material; the
resulting polarisation will produce its own electric field. This contributes to the
net electric field, which gets modified. The modified electric field again
modifies polarisation and this process continues. Thus, in reality, the
phenomenon of polarisation of a dielectric is far more complex and we shall
not go into the details here. For the time being, we are interested in knowing:
What field does a polarised dielectric itself produce? This is what you will learn
in the next section. But you may like to work out a simple SAQ before studying
98 further.
Unit 4 Dielectric in Electric Field
SAQ 1

Determine the unit of P .

4.4 ELECTRIC FIELD OF A POLARISED


OBJECT
Consider a polarised dielectric object, which contains a large number of
atomic/molecular dipoles aligned in the direction of the applied electric field.
The dipole moment per unit volume of this material be given by polarisation

P . We are interested in determining the electric field produced by this object
at a given point.
To determine the field due to a polarised object, we divide it into a large
number of infinitesimal dipoles and integrate their electric fields to get the net
electric field of the object. This is a standard method in physics about which
you must have studied in your undergraduate physics courses. And you may
know that the problem is easier to solve for the electric potential since it is a
scalar. Then we can obtain the electric field from the expression for electric
potential.
So we consider a small volume element d of this material which has a dipole

moment P d . We first calculate the electric potential due to this dipole
element at the given point. The total electric potential is then obtained by
integrating over the entire material. You know from Unit 1 [Eq. (1.19e)] that for

a single dipole having dipole moment p, the electric potential V produced at a

point r from the dipole is given by:

1 rˆ . p
V (4.7)
4  0 r 2
Study Fig. 4.10. It shows a volume element d of the dielectric material. It is
 
situated at r  ( x , y , z) and has dipole moment P d. The electric potential dV
due to this volume element at the point P (x, y, z) is given by:
 
1 p . rˆ1 1 P . rˆ1  
dV   d ( p  P d) (4.8)
4 0 r12 4 0 r12
  
where r1  r  r  (4.9)
z


  
d  P r  r   r1
P ( x, y , z )

r

r

x
Fig. 4.10: Electric field due to a polarised dielectric.
99
Block 1 Electrostatics
Integrating over the entire volume of the dielectric material, we get the total
Note that for volume
electric potential as
integral in Eq. (4.10), we 
have used single integral  1 P . rˆ1
sign instead of triple
V (r )  
4 0 r  r 2
d (4.10)
integral sign. This has 
been done for the ease Now, you can show that
of writing the rˆ1
 1
mathematical     (4.11)
expressions. Both the  r1  r12
conventions - using where  is evaluated at ( x , y , z) . In fact, you can do this calculation and
double and triple integral
arrive at Eq. (4.11) yourself. Solve SAQ 2 before studying further.
signs for surface and
volume integrals,
respectively, and using SAQ 2
single integral sign with
Derive Eq. (4.11).
integrand indicating
whether it is a surface or
Using Eq. (4.11) in Eq. (4.10), we can write
a volume integral – are
used in physics text  1   1
books. You should be
V (r ) 
4  0 
P.   d
 r1 
(4.12)
familiar with both and 
mindful of the context.
We now make use of the following vector identity in Eq. (4.12):
  
. (fA)  f . A  A . f
You know that the

electric potential due to a where f and A are scalar and vector fields, respectively. We substitute
point charge q at a
distance r from it is 1  
f  and A  P in the vector identity. Then we get
 1 q r1
V (r ) 
4  0 r 
  1  P 1 
and the electric potential

P.      .   . P 
 r1   r1  r1
of a distribution of
charges is Substituting this result in Eq. (4.12), we can write the expression for the
 1 q electric potential as
V (r )   i
4  0 ri   
1   P 1 
For a continuous V 
4  0   
 .  d 
r1   r1
( . P ) d

(4.13)
distribution of charges:   
 1 dq
V (r )   We now apply the divergence theorem to the first term in Eq. (4.13) and
4  0 r rewrite the expression of the potential as
For a charge distribution 1 1   1 1 
having volume charge

V
4 0 r1 
P . dS 
4 0 r1 
( . P ) d (4.14)
density (r ) : S 

 1 (r ) The first term on the right hand side (RHS) in Eq. (4.14) is equivalent to the
V (r )   r d electric potential produced by a surface charge density b (see the last
4 0
For a surface charge equation in the margin remark) if we define b as
distribution having 
surface charge b  P . nˆ (4.15)

density (r ) : where n̂ is the unit vector normal to the surface. The second term in the RHS

 1 (r ) of Eq. (4.14) is equivalent to the electric potential produced by a volume
V (r )   r dS charge density b (see the equation in the margin remark) if we define b as
4  0

b    . P (4.16)
100
Unit 4 Dielectric in Electric Field
With these definitions, we can write Eq. (4.14) as

1 b 1 b
V 
4  0  r1
dS  
4  0  r1 d (4.17)
S 
Thus, the electric potential and hence the electric field of a polarised
 object is
the same as that produced by a volume charge density b   . P and a

surface charge density b  P . nˆ of bound charges in the dielectric. So we do
not need to calculate the contributions of all infinitesimal dipoles in a polarised
object to solve Eq. (4.10). Instead, we can determine the bound charges and
then calculate the electric fields they produce. This is the same as calculating
the electric field of any volume or surface charge density using Gauss’s law.
You may now like to understand the physical meaning of these bound charge
densities in a polarised dielectric.

4.4.1 Physical Interpretation of Bound Charge


Density
In the previous section, you have learnt that the electric field of a polarised
object is the same as that produced by a certain distribution of “bound
charges” having densities  b and  b . But we had just defined these
quantities to recast the integral of Eq. (4.12) in a certain form without
explaining the physical basis for these bound charge densities. This is what
we do now. We will now demonstrate that the bound electric charge densities
 b and  b represent actual accumulation of charge.
Let us first consider surface charge density  b . Let N be the number of
molecules per unit volume in the dielectric. In the presence of an external
electric field E, the centres of positive and negative charges are separated by
a distance d. Let us assume for simplicity that the centres of negative charges
remain fixed and the centres of positive charges move to produce a dipole 

moment p per molecule. Now consider an element of surface area  d A in the
dielectric (Fig. 4.11). In the presence of an external electric field E, the
centres of positive chargeswould cross the element of surface area d A by
moving in the direction of E .

E

dA  
 

dA  nˆdA
 


 F 

 

d d
 


Fig. 4.11: dA is along the normal to the shaded surface. The circles represent
positive and negative charges in the molecule, which are separated by
a distance d in the direction of the electric field. 101
Block 1 Electrostatics
The number of centres of positive charges that will cross the element of
surface area d A will be the number of molecules contained in a parallelepiped
of volume
 
dV  dA . d (4.18)

Therefore, the charge in the volume dV is


      
dQ  N q dV  N qd . dA  N p . dA  P . dA  P . nˆ dA (4.19)
   
where p  qd is the dipole moment, P  Np is the polarisation and n̂ is the

unit vector normal to the surface. From Fig. 4.11, you can see that if d A is an
element of area on the surface of the dielectric, the charge dQ will accumulate
there in a layer of thickness d . nˆ . Since d is of the order of molecular size, we
can consider the charge to be present on the surface of the dielectric.
Therefore, the surface charge density, i.e., surface charge per unit area, is
given by
dQ 
b   P . nˆ (4.20)
dA
The effect of polarisation is, therefore, to give rise to a bound charge over the
surface of the material.
Next, consider the case when the polarisation is non-uniform, that is, it is
different at different points in the dielectric. This means that the dielectric is not
a linear dielectric. It is anisotropic (polarisation is not the same in all directions)
and non-homogeneous (polarisation varies with position).
In this case we get an accumulation of bound charge within the material along
with the bound charge on its surface. Let us calculate the bound charge
density in this case. Since polarisation is non-uniform, a net charge Q flows
out of the
 volume dV of the parallelepiped through the element of surface
area d A . We can obtain its value by integrating dQ given by Eq. (4.19) over
the entire surface:
 

Q  P . dA (4.21)

The net bound charge that remains inside a given volume is equal and
opposite to the charge that flows out of it. Therefore,
 
 Q   P . dA  (4.22a)
S

As you know, we can express this net bound charge in terms of the bound
volume charge density as follows:
 
Q  b dV  
 P . dA  bdV  (4.22b)
V S V

You know from Gauss’s divergence theorem that


  

P . dA  . P dV (4.23)
S V

Substituting Eq. (4.22b) in Eq. (4.23), we get



. P dV   bdV  (4.24)
102 V V
Unit 4 Dielectric in Electric Field
Since this result is true for all volume elements, we have

b    . P (4.25)

Eq. (4.25) tells us that if the polarisation of the dielectric is non-uniform,


its divergence results in the net pile-up of bound charges in the material.
The volume charge density is associated with this bound charge. (The volume

charge density is zero for isotropic dielectrics since . P  0 for them.) These
are perfectly real charge densities which we have called here (surface and
volume) bound or polarisation charge density. You may now like to calculate
these charge densities for a concrete situation.

SAQ 3

a) A dielectric block is polarised such that P  2.5  107 (2x iˆ  ˆj  kˆ ) C m2 .
Calculate the bound volume charge density for the block.

b) Consider a polarised
 rectangular block of a dielectric (Fig. 4.12) whose
polarisation P  2.0  10 6 kˆ C m 2 . Calculate the bound surface charge
density on the six faces of the block.

H E

C
D

G F

A y
B

Fig. 4.12: Diagram for SAQ 3b.

We now discuss how Gauss’s law is modified inside a dielectric.

4.5 ELECTROSTATIC EQUATION IN 


DIELECTRICS: DISPLACEMENT VECTOR D
AND GAUSS’S LAW
You have studied the fundamental equation of electrostatics, namely, Gauss’s
law in Unit 1. Recall from Unit 1 that the differential form of Gauss’s law is
given by:
 
. E  (4.26)
0

where  is the volume charge density of all electric charges and E is the total
electric field of all these charges. For modifying Gauss’s law for dielectric
materials, we find that it is convenient to separate the electric field of
Eq. (4.26) into two parts: 103
Block 1 Electrostatics
1. one that results from the bound polarisation charge density ( b ), and
2. the other that is due to everything else (which, for want of a better term,
we call free charge).
The free charge is any other charge in the material that is not the result of
polarisation; it could be due to electrons in a conductor or ions embedded in a
dielectric material or due to any other factor. Let us not at the moment worry
about the source of the free charge. Then we can express the total volume
charge density  within the dielectric as the sum of bound polarisation
charge density  b and the free charge density f :

  f  b

Eq. (4.26) or Gauss’s law then becomes



 f  b f  . P 
. E   ( b   . P ) (4.27)
0 0

  P  f
or .  E    (4.28a)
 0  0
 
or  
. 0E  P  f (4.28b)

We define a new vector D called the electric displacement as follows:
  
D  0 E  P (4.29)

 in Eq. (4.28b), we get Gauss’s law for a dielectric


Substituting Eq. (4.29)
medium in terms of D :

 . D  f (4.30a)

In the integral form, Gauss’s law for a dielectric medium is given by:
 
 D . dS  (qf )en (4.30b)

where (qf )en is the total free charge in the volume. This is a useful way of
expressing Gauss’s law for dielectric materials as it refers to only free charges
enclosed in the volume. So we can calculate D by the standard methods
using Gauss’s law for charge distributions having some kind of symmetry

(linear, planar, spherical or cylindrical). The equation for curl of E remains
unchanged:
 
E  0 (4.31)

From
 Eq. (4.6) of Sec. 4.3, you know that for linear dielectrics, the polarisation
P is proportional to the electric field and is given by:
 
P  0 E (4.32)

provided E is not too strong. You have learnt that  is called the electric
susceptibility of the dielectric material/medium. Using Eq. (4.32) in Eq.
(4.29), we can write D for linear dielectrics as
   
D  0 E  0  E  0 (1  ) E (4.33)
104
Unit 4 Dielectric in Electric Field
If we define a new parameter  given by
   0 (1   ) (4.34)

Then we can write the D field inside dielectrics given by Eq. (4.33) as
 
D  E (4.35)

The constant of proportionality  defined by Eq. (4.34) is called the


permittivity of the dielectric material. Eq. (4.35) tells us that
 the electric
displacement D is proportional to the total electric field E . If we divide
Eq. (4.34) by the factor 0 , we get a dimensionless quantity  r or K:

r   1   K (4.36)
0

The constant  r or K is called the relative permittivity or the dielectric


constant of the material/medium. Henceforth, we shall use the symbol K for
the dielectric constant in this course. Thus, we can write Eq. (4.35) as
 
D  0 K E (4.37)

The susceptibility and dielectric constant are important macroscopic properties


of dielectric materials. The dielectric constant is also a measure of the extent
to which a dielectric is polarised by an external electric field. If a material with
a large dielectric constant is placed in an electric field, the magnitude of the
electric field will be significantly reduced inside the dielectric. This property is
used for increasing the capacitance and is important in the design of
capacitors for various applications. You may like to pause and reflect over
what you have studied so far. You may also like to try an SAQ to calculate the
value of dielectric constant.

SAQ 4
Two parallel plates, which have cross-sectional area of 100 cm 2 , carry equal
and opposite charge of 1.0  107 C. The space between the plates is filled
with a dielectric material and the electric field within the dielectric is
3.3  105 Vm 1. What is the dielectric constant of the dielectric if the electric

field across the plates without the dielectric is given by E0  , where  is
0
the surface charge density of the plates?

Thus, the laws of electrostatics in vacuum given by

   
. E0  f and   E  0 (4.38a)
0

are modified as follows for linear dielectrics, (i.e., when polarisation is


proportional to the electric field):

    
. D  f or .(K E )  f and   E  0 (4.38b)
0 105
Block 1 Electrostatics
If K is same everywhere, i.e., it is a constant, then we can write
   
  E    (K E )    D  0 (4.38c)

Note that Eqs. (4.38b and c) for KE are of the same form as Eq. (4.38a) for
E 0 , the electric field in vacuum. We, therefore, have the solution
KE  E 0 (4.39a)

Eq. (4.39a) implies that in a dielectric medium with dielectric constant K, the
electric field is everywhere reduced by a factor K.
Further, you know from undergraduate physics that the potential difference
between any two points a and b is just the negative of the line integral of the
electric field:
b 


Vb  Va   E . dr (4.39b)
a

Therefore, the potential difference or voltage is reduced by the same factor K.


For a parallel plate capacitor, the charge placed on the capacitor plates is the
q
same. Hence, its capacitance C  is increased by a factor K. Thus, from
V
Eq. (4.1), capacitance in the presence of dielectric of dielectric constant K is
given by:
 KA
C 0 (4.39c)
d

SAQ 5

Consider a parallel plate capacitor made up of two rectangular plates of area


of cross-section 6.45  10 4 m2 and separated by a distance of 2.0  10 3 m .
A voltage of 100 V is applied across the plates. If a dielectric material of
dielectric constant 6.0 is introduced between the plates of the capacitor,
calculate the
a) capacitance of the capacitor;
b) charge stored on each plate of the capacitor;
c) displacement D; and
d) polarisation P.

O q2 You may now like to know: What is the force between two point charges
q1 r
placed in a dielectric? To answer this question, consider two charges q1 and
q 2 situated in a homogeneous dielectric like a liquid or gas. Let us take a
S Gaussian spherical surface S in this material centred around the charge q1
and of radius r, the distance between the two charges q1 and q 2 (see
Fig. 4.13: Two
charges q1 and q 2 Fig. 4.13).

situated in a Let us apply Gauss’s law to this surface. For a spherical surface, D is along
homogeneous the radius vector. Thus, it is parallel to n̂ , the unit vector normal to the surface
dielectric medium.
S and we have
  

106
  
D .dS  D . nˆ dS  D dS  qen
S S S
Unit 4 Dielectric in Electric Field
For the Gaussian sphere of radius r enclosing the charge q1, applying
Gauss’s law for constant D, we get
q1
4 r 2D  q1 or D
4 r 2

From Eq. (4.37), we can write this result as


q1
D  0 K E
4 r 2

q1  q1
or E and E  rˆ (4.40)
4 0r 2K 4 0r 2K

Here rˆ is the unit vector along the radius pointing from q1 to q 2 .

Thus, the force experienced by the charge q 2 is,


  q1 q2
F  q2E  rˆ (4.41)
4  0 K r 2

From Eq. (4.41), you can see that the force between any two charges in a
dielectric medium is reduced by the factor K.
We now take up an example to calculate the electric field in a dielectric.

Example 4.2

A metal sphere of radius a carries charge q. It is surrounded by a linear


dielectric material of dielectric constant K up to distance b. Calculate the
electric field in the three regions (i) r < a, (ii) a  r  b and (iii) r  b and the
electric potential at the centre of the sphere.
Solution : Refer to Fig. 4.14. Since the charge in a conductor resides on its
surface, we have from Gauss’s law: Fig. 4.14: Diagram for
Example 4.2
E0 for r a

For calculating the electric field in the region a  r  b where the dielectric In solving the integrals in
  Eq. (4.44), we have
medium is present we use Eq. (4.30b) given as D . dS  qen  used the result
S
1 1
  r 2 dr   r
D 4 r 2   q
q
  D rˆ for a  r  b (4.42)
4 r 2
Hence,
From Eq. (4.37), a b
q q
  
  4  r 2
dr  
4  r a
D b
D  0 K E or E
0 K 
q  1  1
 
4  r a b
 q q
Thus, E rˆ  rˆ for a  r  b (4.43a) Other integrals are
4 0 Kr 2 4 r 2 special cases.

In the region r  b where the dielectric material is not present, the electric field
is given by:
 q
E rˆ for r b (4.43b)
4 0r 2 107
Block 1 Electrostatics
The electric potential at the centre of the sphere is, therefore,
0  b a 0
q q
V   E . dl   4  0 r 2
dr   4  r 2 
dr  0 dr (4.44)
  b a

q  1 1 1 
Hence, V     (4.45)
4   0 b  a  b 

You may now like to solve a problem on your own.

SAQ 6
A large metal plate of area 1.0 m2 carries a charge 4.4  1010 C. Calculate
the electric field at a point near the plate.

As you know, many electrostatic problems require us to solve differential


equations. If a problem
 involves
 two different media, then the derivative of a
quantity (such as E and D ) at the discontinuity or the interface of the two
media becomes infinite and the problem cannot be solved. In such situations,
differential equations need to be solved separately for the two homogenous
media/regions and then connect the solutions with the help of set of equations
called boundary conditions.
 You will learn to determine the boundary
conditions that E and D must satisfy at the interface between two dielectric
media.
 
4.6 BOUNDARY CONDITIONS ON E AND D

Let us consider two dielectric media 1 and 2 joined to each other


 as shown
 in
Fig. 4.15. We are interested in knowing how the field vectors D and E will
change in passing through the interface of these two media.

We first take up the displacement vector D . From Eq. (4.30b), we know that
Gauss’s law for dielectric is given as
 
 D . dS  (qf )en

for any closed surface S.


Fig. 4.15: Determining boundary conditions on D at the interface of two
108 dielectric surfaces 1 and 2 in contact.
Unit 4 Dielectric in Electric Field
In order to apply Gauss’s law at the boundary/interface of dielectric media 1 and
2, consider a tiny, wafer thin, Gaussian pill box extending just a little on either
side of the boundary as shown in Fig. 4.15. We assume that the pill box encloses
an area A of the interface and there is a surface charge density  f of external
 
charge on the interface of the two media. If D1 and D 2 are the displacement
vectors in medium 1 and 2, respectively, we can write Gauss’s law for this pill
box, as
 
D.dS  D1.nˆ1A  D2.nˆ 2 A  contribution form curved surface
pill box
 
 (D1  D2 ).nˆ1A  0  (qf )en (4.46)

because, in the limit as the height of the pill box goes to zero, there is no flux
from its curved surface and its contribution is zero. Further, in terms of the
surface charge density  f , we can write

(qf )en  f  A

And hence Eq. (4.46) as


 
( D1  D2 ) . nˆ   f (4.47)

where n̂ is the unit vector normal to the surface. Note that D.nˆ is the component

of D along n̂, the unit vector normal to the interface of the dielectric surfaces.

Thus Eq. (4.47) gives the boundary condition for the components of D fields of
the two media (dielectrics) 1 and  2 normal to their interface. Eq. (4.47) tells us
that the normal component of D is discontinuous at the boundary of two
dielectric; the discontinuity is equal to the free surface charge density.
In the absence of any free charges at the interface, f  0 and Eq. (4.47)
becomes
   
( D1  D2 ) . nˆ  0 or (D1  D2 )normal  0 (4.48)

since n̂ is unit vector normal to the surface. Eq. (4.48) tells us that in the absence
of any free surface charges at the interface of two dielectric materials in contact,
the normal component of D is continuous at their interface.

Let us now obtain the boundary conditions on the electric field vector E . To do
so, we consider a very thin rectangular amperian loop ABCDA covering a
portion of the boundary between medium 1 and 2 (Fig. 4.16). Let
AB  CD  l be the length of the loop.


Fig. 4.16: Determining boundary conditions on E at the interface of two dielectric
media 1 and 2 in contact. 109
Block 1 Electrostatics
 
We know that the line integral of E.dl around any closed path is zero. Thus,
for the loop ABCDA we can write

     
 E. dI  E1.I  E 2 .I  contribution from sides AD and BC  0

(4.49)
In the limit as the thickness of the amperian loop goes to zero, the contribution
from sides AD and CD becomes zero. Therefore, Eq. (4.49) becomes
    
(E1  E2 ) . I  (E1  E2 ) . IIˆ  0

because I  I Iˆ . Thus, we have
 
( E1  E 2 ) . Iˆ  0 (4.50)

since I is constant. ˆ
 Note from Fig. 4.16 that
 I is the unit vector in the plane of
the surface and E. Iˆ is the component of E along the surface. This means that
 
E. Iˆ is the tangential component of E . Therefore, we can write Eq. (4.50) as
follows:
 
( E 1  E 2 ) tangential  0 (4.51)

Eq. (4.51) tells us that the tangential component of electric field E is
continuous at the boundary.
Solve the following SAQ to know the behavior of the normal component of

E at the boundary.

SAQ 7

Show that the normal component of E is discontinuous across the boundary of
a dielectric.

We now summarise what you have studied in this unit.

4.7 SUMMARY
 When an insulating material called a dielectric is placed in an external
electric field it gets polarised.

 Electric dipole moment per unit volume P is called polarisation.

At the atomic level, polarisation of the dielectric material or medium


takes place in two ways:

i) If the dielectric material is made up of neutral atoms/molecules in


which the centres of positive and negative charges coincide, the
neutral atom/molecule (called non-polar molecule) does not have
any electric dipole moment. The effect of external electric field on
neutral atoms/molecules is that the centres of positive and
negative charges in them are separated and the material develops
a net electric dipole moment.
110
Unit 4 Dielectric in Electric Field
ii) If the dielectric material is made up of polar molecules, then in the
absence of the electric field the permanent dipole moments move
randomly due to the thermal motion of the molecules. However, in
the presence of external electric field, the permanent dipole
moments tend to align along the direction of the electric field and
the dielectric material develops a net electric dipole moment.

 The electric dipole moment acquired by an atom/molecule is


proportional to the electric field and can be written as
 
p  E

where  is called the atomic/molecular polarisability.

 The electric field produced by a polarised dielectric is equivalent to the


electric field produced by a bound surface charge density  b  P . nˆ
 
and a bound volume charge density  b   . P.

 The electric potential due to a polarised dielectric at a distance r1 is


given as
1 b 1 b
V 
4  0  r1 dS  
4  0  r1 d
S 
 Gauss’s law of electrostatics gets modified in a dielectric
 medium and it
is convenient to introduce a displacement vector D for the medium
given by:
  
D  0 E  P
 
In terms of D, Gauss’s law states that the flux of D through a closed
surface is equal to the total free charge enclosed in the volume
bounded by the closed surface:
   

D . dS  (qf )enclosed or  . D  f
S

 For ideal, homogeneous and isotropic dielectrics, called linear


dielectrics
 
P  0 E

where  is called the electric


 susceptibility of the dielectric material.
The displacement vector D for linear dielectrics is
  
D   0 (1 ) E   E

where  is called the permittivity of the medium. We define a


dimensionless quantity K called the dielectric constant as
  
K and hence D   0 K E
0

D depends only on the free charges and can be obtained without any
reference whatsoever to the bound charges in a dielectric. 111
Block 1 Electrostatics
 In a dielectric, the electric field due to a distribution of free charges is
reduced by a factor K. This has the effect of increasing the capacitance
of a capacitor filled with a dielectric by a factor equal to the dielectric
constant of the material.
 
 The boundary conditions on the D and E fields at the boundary of two
dielectric media 1 and 2 are
 
(D1  D2 )normal  f
 
( E 1  E 2 ) tangential  0

where  f is the free surface charge density at the boundary. If  f  0,


that is, the boundary of the two dielectric media is charge free, we have
 
(D1  D2 )normal  0

4.8 TERMINAL QUESTIONS


1. Two parallel conducting plates of area of cross-section 2.0 m 2 are
separated by a distance of 1.0  102 m. The potential difference (V0)
between them in vacuum is 3000 V. When a dielectric sheet of thickness
1.0 cm is introduced between them, the voltage is found to decrease to
1000 V. Calculate

a) the dielectric constant K, the permittivity  of the dielectric and its


susceptibility ,
b) the electric field between the plates in vacuum,
c) the electric field in the dielectric, and
d) the electric field produced by the bound charges.

2. Consider two isotropic dielectric mediums A and B of permittivity 1 and


2 , respectively, separated by a charge free boundary as shown in

Fig. 4.17. The electric field E1 is incident at the boundary of the media at

an angle of incidence i and the electric field E 2 in medium B makes an
angle of refraction r . Assuming that at the interface of the two dielectrics,
 
the normal component of D and tangential component of E are
tan i 
continuous, show that  1
tan r 2

 f E1
                     
i
d1 A

E r dielectric
d2 B

E2
                     
 f
Fig. 4.17: Diagram for TQ 2.
112
Unit 4 Dielectric in Electric Field
3. Show that the polarisation (bound) charge density at the interface of two
charge free dielectrics of permittivity  1 and  2 is given by

    2 
 b  nˆ . ( P1  P2 )   0 1 E1 . nˆ
2

where n̂ is a unit
 vector normal to the surface. Assume
 that the normal
component of D and tangential component of E are continuous at the
interface of the two dielectrics.

4. A thin dielectric rod of cross-section A extends along the x-axis from x  0


to x  L. The polarisation of the rod is along its length and is given by
P  (ax 2  b) iˆ. Obtain the bound volume charge densities and the surface
charge densities at each end of the rod. Show explicitly that the total
bound charges vanish.

4.9 SOLUTIONS AND ANSWERS


Self-Assessment Questions
 
1. P is dipole moment per unit volume. Therefore, unit of P is:

Coulomb  metre
or Cm  2
3
(metre)

 1      1 
2. From the definition of the gradient,     iˆ  ˆj  kˆ  
 r1   x y  z   r1 

where from Eq. (4.9), r1  ( x  x )2  ( y  y )2  ( z  z)2 . Now

 
  1   1   ( x  x ) ,
  

x  r1  x    r13
 ( x  x )  ( y  y  )  ( z  z )
2 2 2

 
  1   1   ( y  y )
  
y   r1  y   ( x  x )2  ( y  y )2  ( z  z)2  r13
 

and

 
  1   1   ( z  z )
  
z  r1  z  ( x  x )2  ( y  y )2  ( z  z)2  r13
 

Therefore,

ˆ      1  ( x  x) iˆ  ( y  y ) ˆj  ( z  z) kˆ
i  jˆ  kˆ   
 x  y  z   r1  r13


r1 rˆ 
  1 ( r1  r1 rˆ1)
r13 r12 113
Block 1 Electrostatics

where we have used Eq. (4.9) for the expression of r1 . Hence, we get
Eq. (4.11):
  1 rˆ1
  
 r1  r12

3. a) From Eq. (4.25), the volume charge density  b is given by

      
b  .P   iˆ  ˆj  kˆ  . (2x iˆ  ˆj  kˆ )  2.5  107 C m3
 x y z 

  2  2.5  107 Cm3   5.0  107 C m3



b) Surface charge density is the component of P normal to the surface.
Now refer to Fig. 4.12. The faces BFEC and AGHD have normal along
ĵ and  ĵ , respectively. The charge densities on these surfaces are
 
zero since jˆ . kˆ  0 : P . jˆ  0 and  P . jˆ  0. The faces ABCD and
GFEH have normals along iˆ and  iˆ, respectively. The charge
densities on these surfaces too are zero because iˆ . kˆ  0.

Charge density on the face DCEH    P . kˆ  2.0  106 C m2

Charge density on the face ABFG     P . kˆ  2.0  106 C m2

4. The surface charge density on the plates is

q 1.0  107 C
   1.0  10 5 C m 2
A 100  10 4 m2

The electric field between the plates in the absence of any dielectric is

 1.0 105 C m2 1.0  107


E0    Vm 1
0 8.85  1012 C2N1m 2 8.85

In the presence of the dielectric, the field is reduced by a factor equal to


the dielectric constant. Therefore, from Eq. (4.39a),
E0
E 
K

E0 1.0  10 7
 K   3.4
E 8.85  3.3  10 5

5. a) From Eq. (4.39c), the capacitance C of a parallel plate capacitor filled


with a dielectric material of dielectric constant K is given by

 A 6.0  8.85  10 12  6.45  10 4


C  K 0  F  1.7  10 11 F
 3
d 2.0  10

b) The voltage applied is 100 V. Therefore,


Charge stored on each plate  CV

 1.7  10 11  100 C  1.7  10 9 C


114
Unit 4 Dielectric in Electric Field
   
S

c) Applying Gauss’s law for the dielectric  D . dS  (qf )en  to this case,

q
we get DA  q or D 
A

q 1.7  109
 D  C m 2  2.6  10 6 C m 2
A 6.45  10 4
V
d)  D  0E  P and E  , we get
d

 8.85  1012  100 


P  D  0E  2.6  10 6   Cm
2
3
 2.0  10 

or P  (2.6  10 6  4.4  107 ) C m2  2.2  106 C m2

6. Let  be the charge density on the surface of the plates. Now consider a
“Gaussian pill box” which extends to equal distances above and below the
E
plane of the positively charged plate (Fig. 4.18). Let us apply Gauss’s law
to this surface:
A
  (q )
 E . dS  f encl
0
(i) E
S
Fig. 4.18: Diagram
If A is the area of the lid of the pill box, then for this case for SAQ 6.

( q f ) encl  A (ii)

Only the top and bottom surfaces of the pill box contribute to the integral
 
since for other surfaces, E and dS are perpendicular to each other and
their scalar product is zero.

For both the top and bottom surfaces of the pill box, the electric field points

away from the plane (since the vector dS is normal to the surfaces). It is
upwards for the points above the plane and downwards for the points
below the plane.
Thus, we take the contributions of only the top and bottom surfaces of the
pill box to the electric field into account. Then using Eq. (ii), the value of the
integral of Eq. (i) is given by
  A 
 E . dS  2AE  0 or E 
20
S
q 4.4  10 10 C
Since A  1.0 m2 ,     4.4  10 10 C m  2
A 1 .0 m 2

 4.4  1010 C m2


 E    24.9 Vm1  25 Vm1
20 12 2 1 2
2  8.85  10 C N m

7. From Eq. (4.48), we note that the normal component of displacement


vector is continuous at the boundary of two charge free dielectrics:
 
(D1  D2 )normal  0 115
Block 1 Electrostatics
 
You know that for a dielectric, D   E. Thus, we can write
 
(1 E1   2 E2 )normal  0
 
(1( E1)n   2 ( E2 )n

( E1)n  2
or   1
( E2 )n 1

Thus, the normal component of E at the dielectric boundary is
discontinuous.

Terminal Questions
1. a) From Eq. (4.39a), the dielectric constant

E0 V 3000
K   0   3
E V 1000

From Eq. (4.36):   K  0  3  0 and 1    K  2

V 3000
b) E0  0  V m 1  3.0  105 V m 1
d 1.0  10  2

V 1000
c) E   V m 1  1.0  105 V m 1
 2
d 1.0  10

d) The electric field E is the resultant of the electric field E0 and the field
Eb set up by bound charges.

 E b  E 0  E  2.0  10 5 V m 1

2. As given in the problem, at the interface of the two dielectrics,


 the normal
component of D and the tangential component of E are continuous.
Therefore,

E1 sin i  E 2 sin r (i)

and D1 cos i  D2 cos r (ii)

But from Eq. (4.35), D   E and therefore,

D1  1E1 and D2  2E2

Hence, Eq. (ii) becomes 1E1 cos i   2E 2 cos r (iii)

Dividing Eq. (i) by Eq. (iii), we get

1 1 tan i 
tan i  tan r or  1
1 2 tan r  2

3. The surface charge density of a polarised medium is given by b  P . nˆ,
where n̂ is the unit vector
 normal
 to the face on which polarisation (bound)
charges appear. Let P1 and P2 be the polarisation vectors in the two
116 media.
Unit 4 Dielectric in Electric Field
At the interface, the net surface charge density b is given by:
 
b  nˆ.(P1  P2 )
     
Now D  0 E  P  P  D  0E
     
and P1  P2  (D1  D2 )  0 (E1  E2 )
   
 b  nˆ.(D1  D2 )  0nˆ.(E1  E2 ) (i)

As per the problem, the normal components of D are continuous at the
interface. Thus, we have
 
nˆ.(D1  D2 )  0
     
Therefore, from Eq. (i) b  0nˆ. (E1  E2 )  0nˆ. (E2  E1). But D   E
and therefore,
   
nˆ.(D1  D2 )  0  nˆ.(1 E1  2 E2 )  0
  
or nˆ .E2  1 nˆ .E1
2

      2 
 b  0  1  1 nˆ .E1  0  1  E1. nˆ

 2   2 

4. It is given that P  (ax 2  b) iˆ. See Fig. 4.19.

S1 S2
A x
O dx L

Fig. 4.19: Diagram for TQ 4.

The volume charge density is given by


      
b   . P   iˆ  ˆj  kˆ  .(ax 2  b) iˆ  2ax (i)
 x y z

Since nˆ  iˆ at the face at x  0, the surface charge density at x  0 is


given by
 
b x 0  P . nˆ   P . iˆ  (ax 2  b)  b (ii)
x 0 x 0

Since nˆ  iˆ at the face x  L, the surface charge density at x  L is


 
b x L  P . nˆ  P . iˆ  (ax 2  b)  (aL2  b) (iii)
x L x L

Since dV  Adx, using Eq. (i), we get the total bound volume charge as
L
QV   (2 a x ) A dx   a A L
2
b  b dV 
V 0 117
Block 1 Electrostatics
Using Eq. (ii), we get the bound surface charge on the surface S1 at x  0
as
S
Qb 1  b x  0 A   b A

Using Eq. (iii), we get the bound surface charge on the surface S2 at
x  L as
S
Qb 2  b x  L A  (aL2  b)A

Thus, the total bound charge on the rod is


S S
Qbtotal  QV
b  Qb  Qb
1 2   a AL2  bA  (aL2  b )A  0

as expected.

118
Unit 5 Dielectric Properties

UNIT 5
DIELECTRIC
PROPERTIES
Structure

5.1 Introduction 5.4 Electrostatic Energy in a


Expected Learning Outcomes Dielectric
5.2 Electric Fields in a Dielectric 5.5 Summary
Molecular or Local Field and Average 5.6 Terminal Questions
Macroscopic Field in a Dielectric 5.7 Solutions and Answers
Determination of Molecular (Local) Field
in a Spherical Cavity in the Dielectric
5.3 Relation between Polarisability
and Dielectric Constant: Clausius-
Mossotti Equation
Atomic/Molecular Polarisability: A Simple
Model

5.1 INTRODUCTION
In Unit 4, you have studied about the polarisation of a dielectric and the
macroscopic
 aspects of dielectric polarisation. You know that the polarisation
P of a dielectric is actually a large scale manifestation of the electric dipole
moments of its atoms/molecules and it is given by dipole moment per unit
volume of the dielectric. You have seen that in many cases, the polarisation of
a dielectric can be taken into account through the introduction of a dielectric
constant.

In the present unit, we shall explain the polarisation of the dielectric at the
atomic/molecular level and understand how the average macroscopic electric
field in a dielectric is related to the electric field that polarises atoms and
molecules in a dielectric.

In Sec. 5.2, you will study about polarisation at the microscopic level. The
macroscopic properties (such as dielectric constant) of dielectrics can be
related to their microscopic properties (such as atomic/molecular
polarisability). In Sec. 5.3, we derive the Clausius-Mossotti equation that
gives the relation between atomic/molecular polarisability and dielectric
constant for a dielectric. We also discuss a simple physical model of a
nonpolar dielectric which is consistent with the predictions of Clausius-
Mossotti equation. In Sec. 5.4, we study the electrostatic energy in a dielectric.
119
Block 1 Electrostatics
In the next unit which is the first unit of Block 2, you will study about magnetic
fields and magnetostatics.
Expected Learning Outcomes
After studying this unit, you should be able to:
 define the molecular (local) field and relate it with polarisation;
 determine the average macroscopic electric field within the dielectric and
relate it to the local field and polarisation;
 derive Clausius-Mossotti equation for a dielectric;
 explain a simple model of non-polar dielectric; and
 derive the expression for electrostatic energy of a dielectric.

5.2 ELECTRIC FIELDS IN A DIELECTRIC


You have studied in the previous unit about polarisation of a dielectric material
placed in an electric field. You know how a macroscopic parameter called
dielectric constant characterises the polarisation of dielectric. You would now
like to know the microscopic mechanism responsible for polarisation of a
dielectric kept in an electric field. The question we ask is: What happens at
the level of atoms and molecules of the dielectric when it is placed in an
electric field? To understand the mechanism, we investigate the relation
between the average macroscopic electric field in the dielectric and the local
or molecular electric field responsible for polarising a molecule of the

dielectric. We first define the local or molecular electric field (E m ) and then
establish its relationship with the average macroscopic electric field inside the

dielectric. We then determine E m in terms of the macroscopic electric field
and polarisation.
5.2.1 Molecular or Local Field and Average Macroscopic
Field in a Dielectric
Consider a uniformly polarised dielectric (Fig. 5.1). The electric field which is
responsible for polarising a molecule of the dielectric is called the local or the
molecular electric field.

Fig. 5.1: Site A where molecular electric field is to be calculated in a uniformly


polarised medium.
The molecular or local electric field at a given molecular position in a dielectric
is the electric field produced by all external sources and by all polarised
molecules in the dielectric with the exception of the molecule at the position
under consideration.
Let us consider the electric field at
 the molecular position, say A, in the
120 dielectric shown in Fig. 5.1. Let E m denote the molecular or local electric field
Unit 5 Dielectric Properties
 
at point A and pm be the dipole moment of the molecule induced by E m .
Then, from Eq. (4.4)
 of Unit 4, you know that the dipole moment at point A is
proportional to E m and we can write
   
pm  Em or pm   Em (5.1)

where  is the constant of proportionality and is called the atomic/molecular


polarisability of the dielectric. The question
 now is: How do we determine
the local (or molecular) electric field E m ? This is what we do now.

Let us suppose that a thin dielectric slab has been polarised by placing it in a
uniform electric field, for example, by placing it between the plates of a parallel
plate capacitor with opposite charges on its plates (Fig. 5.2a). We assume that
the polarisation produced in the dielectric material is uniform. Since the
polarisation is uniform, itis independent
 of position and its divergence will be
zero. Thus, we have . P  0 . Also, P is parallel to the electric field producing
it.

          E dp  
         

P  
      

            

         
    
      A 
     
      



           
       
         
            
E ex

(a) (b)
Fig. 5.2: a) A polarised dielectric; b) replacement of dielectric outside the cavity
by bound charges.

Next, let us cut out a small piece of the dielectric (say of radius 100 times the
size of the molecule), leaving a spherical cavity around the point A (Fig. 5.2b).
We treat the rest of the dielectric as a continuum from the macroscopic point
of view.
We then put all the molecules back in the cavity except the particular molecule
at the centre A of the cavity where we wish to compute the molecular electric
field. The molecules which have been put back in the cavity are not to be
treated as a continuum but as a collection of a reasonably large number
of individual dipoles. This procedure will work if the net result comes out to
be independent of the size of the cavity. We will find that under certain
conditions, this is indeed so.
Now, recall from Sec. 4.4 of Unit 4 that the electric field of a polarised
dielectric is the same as that of bound surface and volume charge densities.
Therefore, we can replace the dielectric material outside the cavity by a
system of bound charges at the surface of the dielectric slab  and on the
surface of the cavity. Thus, we can write the electric field E m at the centre of
the cavity as     
E m  Eex  Edp  Es  Ec (5.2)
 
where E ex is the applied external electric field. In this particular case, E ex is
the external electric field between the plates of the parallel plate capacitor; 121
Block 1 Electrostatics

E dp is the electric field produced by the bound charges on the outer
surface of the dielectric (this field is called depolarising field because
 it is
directed opposite to the direction to E ex , the polarising field); Es is the

electric field due to bound charges on the surface of the cavity; Ec is the
electric field due to all the dipoles inside the cavity.

The sum of the external electric field E ex and the depolarising electric field

E dp due to the bound charges (to which the polarised dielectric is equivalent)
on the outside surface of the dielectric is nothing but the average macroscopic
electric field E inside the dielectric:
  
E  E ex  E dp (5.3)

Therefore, we can write Eq. (5.2) as


   
E m  E  Es  Ec (5.4)

Eq. (5.4) relates the local or molecular electric field E m to the average
macroscopic electric fields in the dielectric material.
Now, to obtain an expression for electric field at the centre (A) of the spherical
cavity we will use spherical polar coordinates and take the z-axis along the
direction of polarisation of the dielectric
 to calculate the. This will lead us to the
expression for the molecular field E m .

5.2.2 Determination of Molecular (Local) Field in a


Spherical Cavity in the Dielectric
 
From Eq. (5.4), it is evident that we need to obtain the values of E, Es and
 
Ec to determine E m . Since you have already learnt in Unit 4 how to

determine
  due to a polarised dielectric,
E  you now need to learn to determine
Es and Ec . Let us first determine Es . For this, we consider a spherical cavity
of radius r centred at A as shown in Fig. 5.3 and use spherical polar
coordinate system.

 

   
dq 
 
  
  z
  
 r A 
   
 
 
 
P


E

Fig. 5.3: Molecular field at the centre A in a spherical cavity in the dielectric.

Let dq be the charge on infinitesimal surface element dS on the surface of the


cavity. You know that in spherical polar coordinates, dS  r 2 cos  sin  d d .

Further, note that the electric field Es arises due to the bound surface charge
density σ b on the surface (S) of the spherical cavity is given by

122 σ b  P.nˆ (5.5)
Unit 5 Dielectric Properties
where n̂ is the normal to the inner surface of the cavity. Since the outward
normal on the surface of the cavity is to be drawn from the dielectric medium
to the free space, n̂ points opposite to the radial direction (Fig. 5.3) and it is
directed towards A, the centre of the cavity. Thus, we can write nˆ   rˆ and
Eq. (5.5) reduces to
 
σb  P.nˆ   P. rˆ   P cos  (5.6)

Recall from Unit 1 that the expression for the electric field at a point due to a
 1 dq
charge dq is given as dE  rˆ . So, the magnitude of the electric
4 0 r 2
field dE s at the centre of the cavity due to an infinitesimal surface charge
(dq  b dS) placed at the surface of the cavity is given by:
 b dS P cos 
dEs    dS (5.7)
4  0 r 2 4  0 r 2

The electric field dE s is directed towards A along the vector joining dq to the
centre A of the cavity, i.e., along  rˆ.
 P cos  P cos 
 dEs   dS ( rˆ)  dS (rˆ) (5.8)
4  0 r 2 4  0 r 2
 
We can now determine the value of Es by integrating dE s over the entire
surface. This means that we have to calculate the double integral over  (from
0 to ) and  (from 0 to 2). Note that the electric field is radial. Hence, from

symmetry, only the component of the electric field along the direction of P will
contribute
 to the integral. The component of the electric field perpendicular to
P cancels out when integrated over the entire surface. Thus, the electric field
 
ES is along P and is given by:
 2 
  P cos  2

E s  dE s 
4  0 r2r cos  sin  d d
0 0

2
You can see that  d   2. Therefore, we get
0
 2   
 P P
Es 
4  0  
d cos 2  sin  d  (2)
4  0 
cos 2  sin  d
0 0 0

To evaluate the integral over , we substitute x  cos  in the above integral 2
 cos  sin  d 
so that dx   sin  d . Therefore, we get (see margin remark): 0
1
 1     x 2 ( dx )
 P P  2   P

1
Es  . x 2dx  .  or E s  (5.9)
2 0 2 0  3  3 0 1
1   x 2 (dx )
 1
Let us now calculate the value of Ec , the electric field due to electric dipoles 1
inside the cavity. Suppose a large number of dipoles are present in the cavity x3 2
 
and the dielectric material is a gas or a liquid. Then even though the dipoles 3 3
1
are oriented parallel to the electric field, they can be thought of as being
distributed randomly inside the cavity in an isotropic dielectric (this assumption
will not hold for anisotropic dielectrics). Thus, they would not contribute to the 123
Block 1 Electrostatics
net electric field at the molecular position A.In the case of a crystal, due to the
regular atomic positions of a cubic crystal, Ec is again  zero. Therefore, here
we shall confine ourselves to the situation for which Ec  0. Then the
molecular or the local electric field is given by

  P
Em  E  (5.10)
3 0
 
For linear dielectrics, we can express E m in terms of only E as

 E
E m  ( K  2) (5.11)
3

where K is the dielectric constant.

You may like to work out an SAQ to derive Eq. (5.11).

SAQ 1

  P 
Express the molecular electric field E m  E  in terms of E alone.
3 0

We now derive the Clausius-Mossotti equation, which relates the


atomic/molecular polarisability, a microscopic property of dielectric with the
dielectric constant of a dielectric, a macroscopic property of dielectric.

5.3 RELATION BETWEEN POLARISABILITY AND


DIELECTRIC CONSTANT: CLAUSIUS-
MOSSOTTI EQUATION
Recall from the discussion in Sec. 5.2 that the dipole moment of a molecule
 is

related to polarisability and the molecular electric field as: pm   Em .
Remember that the molecular electric field in a dielectric is due to everything
except the particular molecule
 under consideration. We have  established the
relationship between Em and the total macroscopic  field E in the dielectric in

Eq. (5.11). From the relation between pm and Em , we find that the molecular

polarisability  is the dipole moment pm of a molecule produced by a unit
polarising molecular electric field. In other words,

p
  m
Em
Further, if there
 are N molecules per unit volume in the dielectric, then
polarisation P  N pm . Thus, using Eq. (5.10), we can write

    P 
P  N pm  N  Em  N   E   (5.12)
 3 0 

 N   
or 1   P  N E
 30 

 N E
or P  (5.13)
N
1
3 0
124
Unit 5 Dielectric Properties
   
You know from Eqs. (4.29) and (4.35) of Unit 4 that D  0E  P   E, which
yields
 
P  (  0 ) E (5.14)
You also know from Unit 4 that the permittivity  of the medium is related to its
dielectric constant K by Eq. (4.36):   0 K. Therefore, Eq. (5.14) becomes
 
P  0 (K  1) E (5.15)
We can now obtain the relation between the microscopic quantity  and the
 N 
macroscopic quantity K using Eqs. (5.13) and (5.15). Substituting Eq. (5.15) in 0 (K  1)  1  N 
 3 0 
Eq. (5.13), we get
N or
0 (K  1)  N
N 0 (K  1)  0 (K  1)
1
3 0 3 0

We can simplify this expression further to obtain (see margin remark) N 

3 0 (K  1) or
  (5.16)  1 
N (K  2) N  1  (K  1) 
 3 
Eq. (5.16) relates the microscopic quantity atomic/molecular polarisability with
the macroscopic quantity, namely, the dielectric constant of the dielectric that   0 (K  1)
can be determined on macroscopic basis. This relation is called the Clausius-
or
Mossotti Equation. Let us now work through an example to understand how
N
the Clausius-Mossotti Equation can be applied in a real situation. ( K  2)   0 (K  1)
3
Example 5.1
A sphere of linear dielectric
 material of dielectric constant K is placed in a
uniform electric field E0. Determine the electric field inside the sphere and
polarisation as a function of E0.

Solution : Refer
 to Fig. 5.4. The external field polarises the sphere. The
polarisation P is the dipole moment per unit volume.


E0

Fig. 5.4
 
You know that P is proportional to the electric field E inside the dielectric
[Eq. (4.6)]:
 
P  0  E

where  is electrical susceptibility. Further, it can be shown (SAQ 2) that



P
polarisation produces an electric field  inside the sphere. The electric
30
field
 inside the sphere can be obtained as a superposition of the external field
E 0 and the electric field due to polarisation. Thus, we can write
 
  P  0  E
E  E0   E0 
3 0 30 125
Block 1 Electrostatics

 3 E0
 E  (5.17)
(3  )

From Eq. (4.36) of Unit 4, you know that r  1    K. Therefore,


 
 3 E0 3 E0
E  
(3  (K  1)) (K  2)
  3 0 (K  1) 
and the polarisation P  0  E  E0 (5.18)
(K  2)

You may like to prove the result that the polarisation


 in a sphere of linear
P
dielectric material produces an electric field  inside the sphere. Try the
30
following SAQ before studying further.

SAQ 2
Show that the electric field due
 to polarisation inside a polarised sphere is
P
uniform and is equal to  .
30

[Hint: Consider two charged spheres whose centres are separated and
superimpose their electric fields.]

So far we have confined our discussion to the behaviour of a dielectric in an


electrostatic field and obtained relations between its macroscopic and
microscopic properties. You may ask: Can we think of a physical model which
describes the behaviour of electrons and ions in a dielectric under the
influence of electric field and account for atomic/molecular polarisability? This
is what we shall discuss now.
5.3.1 Atomic/Molecular Polarisability: A Simple Model
In the atoms of a non-conducting material, electrons are bound to nucleus by
forces which are in general very complicated. We can obtain a simplified
picture of a dielectric by assuming that the electrons are bound to the nucleus
y by harmonic forces. Thus, we can picture the electrons in the material as
electron
being attached to the nucleus by springs of some spring constant k (Fig. 5.5).
This picture can be justified quite generally for arbitrary forces as long as the
k
displacements from the equilibrium position are small. Assuming one-
dimensional motion along the y-direction (see Fig. 5.5), we can write the
expression for the binding (or restoring) force F between the electron and the
x
nucleus as
nucleus
Fig. 5.5: A simple F   ky   m 02 y (5.19)
model of electrons
and ions in a where m is the mass of the electron and 0 (  k m ) is the natural frequency
dielectric.
of oscillation of electron about the nucleus.
Let us assume that in the unpolarised state, the centres of the nucleus and the
electron cloud of the atoms/molecules of a dielectric coincide. When such a
126
Unit 5 Dielectric Properties

dielectric is placed in a polarising molecular (or local) field E m , the positive
nucleus of its atom/molecule gets displaced with respect to the electron
 cloud.
At equilibrium, the electrostatic force ( qE m ) on an electron due to E m is
balanced by the restoring force [Eq. (5.19)]. So, we can write:
q
m 02 y  qE m  y Em (5.20)
m 02

The dipole created due to separation of nucleus and the electron due to the
applied field will have dipole moment:
  q2 
pm  qy  Em (5.21)
m 02
 
where we have used Eq. (5.20) for y . Further, we also know that pm   Em .
Thus, comparing this expression for dipole moment with Eq. (5.21), we get:
q2
  (5.22)
m 02

Note that we arrived at Eq. (5.22) by considering a single atom/molecule under


the influence of molecular/local field. Since we are dealing with a
homogeneous and isotropic non-polar dielectric, we can extend these results
on a macroscopic scale. Recall from Eq. (4.5) of Unit 4 that if there are N
atoms/molecules per unit volume, the polarisation P is given by:
 
P  Np (5.23)

where p is the dipole moment per atom/molecule. So, using Eq. (5.21), we
write Eq. (5.23) as
 N q2 
P E (5.24)
m 02

where E is the macroscopic electric field in the polarised dielectric.
 
Comparing Eq. (5.24) with the relation P   0 E , we get the expression for
susceptibility of the dielectric as
N q2
 (5.25)
 0 m 02

Further, the dielectric constant K is defined as


 N q2 
K    0  (1  )  1   (5.26)
 m  0 02 
 

using Eq. (5.25).


Now, to check the validity of this simple model of dielectric, let us obtain a
rough estimate of the dielectric constant K of hydrogen, a good example of
non-polar atom/molecule. The energy required to ionise the hydrogen atom is
13.60 eV. Equating this to  0 , we have

13.60  1.602  10 19


0  rads1  20.67  1015 rads 1 127

Block 1 Electrostatics
We can calculate the dielectric constant K (  1  ) as:
Nq 2
K  1 
m 0 02
(1.602  1019 )2  2.690  1025
 1  1.0002
9.109  1031  8.854  1012  (20.67  1015 )2
where we have taken N (  2.690  10 25 m -3 ) to be the number of atoms per
unit volume at STP. Comparing this value of K with its experimental value
1.00026, we can say that it is quite a good estimate.
So far in this section, you have learnt about a simple physical model of a
dielectric as an aggregate of a large number of simple harmonic oscillators.
You have learnt that for electrostatic fields, this model provides a reasonable
estimate of macroscopic properties of a dielectric such as the dielectric
constant. Note that this simple model is valid for non-polar dielectrics because
we have assumed that dipole moment arises only in the presence of polarising
field. For polar dielectrics, we need to take into consideration the fact that it
has permanent dipole moment even in the absence of polarising field.
Before proceeding further, you should solve an SAQ.

SAQ 3
a) A sample of diamond (nucleus of 6 protons surrounded by 6 electrons) has
a density of 3.5 gcm 3 and polarisation of 1.0  10 7 Cm 2 . Calculate: (i)
average dipole moment per atom, and (ii) average separation between the
centres of positive and negative charges.
b) Nitrogen gas at 1 atmospheric pressure has density   1.18 kgm 3 and
dielectric constant K  1.0006. Calculate the polarisability  and using
3
the relation   4 0R0 , estimate the molecular size.

You know from undergraduate physics that many problems in physics become
easier to solve using energy considerations. Like a mechanical system, the
energy of a system of charges comprises of kinetic and potential energies.
However, for electrostatic situations, the entire energy of the system exists as
potential energy. So, you would like to know: What is the electrostatic
energy stored in a dielectric placed in an electrostatic field? This is what
we shall discuss now.

5.4 ELECTROSTATIC ENERGY IN A DIELECTRIC


You have learnt in undergraduate physics that the electrostatic energy of an
arbitrary charge distribution is equal to the work done to assemble this
distribution of charges against the Coulomb interaction. The charge
distribution is assembled one by one so that when we bring a charge from
infinity to the point of interest, we need to do work against the potential of the
charge(s) already present at that point. And, this work done is stored as
electrostatic energy of the charge distribution. If the charge distribution is
continuous having charge density  and the potential is V then the work done
W is given as
1
128
W U  
2
V d (5.27)
Unit 5 Dielectric Properties
where U is the electrostatic energy. The work done [given by Eq. (5.27)] is
stored as the electrostatic energy of the continuous charge distribution. Now,
the question is: Can we use this expression if a dielectric is present at the
location where we are interested in calculating the electrostatic energy? We
cannot do that because we not only need to calculate the work done in bringing
the charge from infinity to the desired location but also need to take into
account the work done in producing polarisation of the dielectric.
Let us assume that we have a dielectric which has a charge distribution with
charge density  . For the moment, let us not worry about how this charge
distribution has been obtained. Further suppose that when a small amount of
charge is brought from infinity, it changes the charge density to    . This
change in charge density can be written in terms of change in electric
displacement because, we know from Eq. (4.38) of Unit 4 that
 
  .D     .( D)
Now, from Eq. (5.27), we can write the additional work  W done in changing
the charge density by   as

 
W  V d  [.( D )]V d (5.28)

If you compare Eqs. (5.28) and (5.27), you will note that the factor of (1 2) is
missing in Eq. (5.28). It is so because in Eq. (5.27), the factor of (1 2) is to
avoid double counting between any two charges while they are being
assembled. But, Eq. (5.28) gives the work done when a small amount of
charge is being brought from infinity to the dielectric and thereby increasing its
charge density by  . To proceed further, we simplify Eq. (5.28) by doing
integration by parts and write:
 
  
W  V [ .(  D )] d  [ V .(  D ) d ] d
  

 V [ .(  D )] d   E . (  D ) d (5.29)

because  V  E . Note that if we apply divergence theorem on the first integral
on the RHS of the above expression, we can write
  
 
V [ .(  D )] d  V (  D ). dS

If we consider the bounding surface S to be very large, that is r   , then


 
 (  D ). dS  0

because variation D tends to zero as r   .
Thus, Eq. (5.29) reduces to
 

W  E .(  D ) d

So, the total work done can



be written as
D  
 
W  d E .  D (5.30)
0

In writing Eq. (5.30), we have considered the total work done in moving
charges from infinity which has caused increase in the value of displacement 129
Block 1 Electrostatics

vector from zero to D . Eq. (5.30) holds for any material. To simplify this
relation, let us assume that the dielectric is linear. Then, we have D  E . So,
we can write
  1  
E . D  (E.D) (5.31)
2
1   1    
because ( E. D )   (  E 2 )   E . E  E . D
2 2
Substituting Eq. (5.31) in Eq. (5.30), we get

D  
1
W 
2  
d (E.D )
0

1  
or W U 
2 
(E.D) d (5.32)

Eq. (5.32) gives the electrostatic energy stored in an electrostatic system


containing a dielectric. You may ask: Where is the electrostatic energy stored
in the system? The energy is stored in the electric field in the system. Further,
you can see from Eq. (5.32) that the energy stored per unit volume, that is,
energy density u can be written as
1  
u (E.D) (5.33)
2
So, if we know the electric field and electric displacement in a region, we can
determine the electrostatic energy stored in the region.
Let us now summarise what you have learnt in this unit.

5.5 SUMMARY
 In a dielectric material, the induced dipole moment and the polarisation
are directly proportional to the molecular/local electric field:
   
p   Em and p  Em

 The local (or molecular) electric field inside a spherical cavity in a


dielectric is given by:

  P
Em  E 
3 0

 In terms of macroscopic electric field E , the local (or molecular)
electric field inside a spherical cavity in a dielectric is given by:

 E
E m  ( K  2)
3
 The relation between the microscopic quantity  (atomic / molecular
polarisability) and the macroscopic quantity K (the dielectric constant)
is given by the Clausius-Mossotti equation:
3 0 K  1

N K 2
130
Unit 5 Dielectric Properties
 A simple model of dielectric in which electrons are considered to be
attached to their respective nuclei by a spring of spring constant k
gives the susceptibility of the dielectric medium as
N q2

 0 m 02

where  0 is the frequency of the electron- nuclei system bound by


spring force. And, the dielectric constant is given as
 N q 2 
K    0  (1  )  1 
 m  0 02 

 The electrostatic energy of a charge distribution in a dielectric is given
as
1  
W U 
2 
(E.D) d

5.6 TERMINAL QUESTIONS


1. A dielectric sphere of radius R has a point charge  q embedded at the
centre. Calculate the bound surface and volume charge densities. Also
calculate the total bound charge on the surface. For a cavity of radius  at
the centre of the sphere, calculate the surface charge density and the total
charge on the surface of the cavity.

2. Consider a free charge density f on a conductor in contact with a


dielectric material of permittivity . Calculate b and show that
  0 
b   f and the total surface charge density t  0 f .
 

3. The dielectric constant of methane at 0 C and 1 atmospheric pressure is


1.00088. There are approximately 2.8  1019 molecules in 1 cm3 volume.
What is the molecular polarisability of methane in units of 0 ?

5.7 SOLUTIONS AND ANSWERS


Self-Assessment Questions (SAQs)

  P
1. From Eq. (5.10), we know that E m  E  . And, from Eq. (4.29) of
3 0
    
Unit 4, we have D   0 E  P   E   0 K E . So, we can write
 
P   0 ( K  1) E
 
  ( K  1) E E
Thus, Em  E   ( K  2)
3 3

2. See Fig. 5.6. A uniformly polarised sphere can be considered as two


spheres of positive and negative charges. Without polarisation, the
spheres overlap so that the centres of positive and negative charges
coincide (Fig. 5.6a). But when the material is polarised, the negative and 131
Block 1 Electrostatics
positive charges move away from each other. The spheres of positive and
negative charges do not overlap completely. Fig. 5.6b shows a situation in
which the negative charges move upward and the positive charges move
downward.

Fig. 5.6

The left over negative charge at the top “cap” and the left over positive
charge at the down “cap” (Fig. 5.6b) constitute the bound surface charges
and the separation of positive and negative charge centres constitutes a
dipole.
Let d be the distance between the centres of negative and positive charges
(Fig. 5.6c). If q be the total charge on the positive sphere, the dipole
moment is given by:
 
p  qd

where d is the displacement of the positive charge with respect to the
negative charge. Now consider a spherical Gaussian surface S of radius r
about the centre of positive charge, passing through the point A
(Fig 5.6c, which is a magnified version of Fig. 5.6b). Let  be the volume
charge density of the uniformly polarised sphere of radius R having charge
q. Then
q
 
4 3
R
3
Applying Gauss’s law to the Gaussian surface S, we have
q en 1  4 3 
E  ( 4 r 2 )    r 
0 0  3 
 r
or E 
3 0
Consider another surface S of radius (d  r) with centre at the negatively
charged sphere. Applying Gauss’s law, we have
1  4
E  4 (d  r ) 2    ( d  r ) 3  
132 0  3 
Unit 5 Dielectric Properties
 r d
or E 
3 0
The total electric field at A is given by the superposition of E and E  ,
which are in opposite directions.
 d
 E  E  E 
3 0

  d  
and E   ( E is directed opposite to the vector d )
3 0

Substituting the value of  we get



 1 q d
E  
3 4 3  0
R
3

 qd
 E   (i)
4  0 R 3
 
Now the dipole moment p  qd

and polarisation is the dipole moment per unit volume.


 
 p qd
Thus, P 
4 3 4 3
R R
3 3
But from (i),
 
qd   4  0R 3E
  4  0 R 3 
 P   3  0E
4 3
R
3
 1 
and E   P
3 0

3. a) You know from undergraduate physics that the number of molecules


per unit volume is given by
N A
N 
M
where  is the mass density, N A is Avogadro’s number and M is the
molecular weight. Substituting the values of  ,
N A (  6.002  10 23 mol -1 ) and M (  12  10 3 kg mol -1 ) for diamond,
we get
[( 3.5  10 3 kg) /(1.0  10 2 m) 3 ]  ( 6.022  10 23 mol 1 )
N 
(12  10 3 ) kg mol 1

3.5  10 3  6.022  10 23
 m 3  1.76  10 29 m 3
12  10 3
Remember that N is the number of molecules per unit volume. 133
Block 1 Electrostatics
i) From Eq. (4.5) of Unit 4, you know that the magnitude of polarisation
is given as P = Np. Thus, we can write the average dipole moment
per atom as
P 1.0  10 7 Cm 2
p    5.7  10 37 Cm
N 1.76  10 29 m 3

ii) Since diamond has 6 electrons, the total charge is 6e where e is


the electron charge and p = 6 e d, where d is the separation
between the centres of positive and negative charges. Therefore,
the average separation between the centres of positive and
negative charges is

p 5.7  10 37 Cm
d    5.9  10 19 m
6e 6  1.6  10 19 C

b) The number of molecules per unit volume is given by


 (6.022  10 23 mol 1 )  1.18 kg m 3
N  NA   0.51 10 26 m  3
M (14  10  3 ) kg mol 1

where we have taken the molecular weight of nitrogen equal to as


14  10 3 kgmol -1 . Using Clausius-Mossotti equation [Eq. (5.16)], we
can write the atomic polarisability as

3  0 K  1 3  8.85  10 12 0.0006 2


   C mN1
N K 2 0.51 10 26 3 . 0006

  1.04  1040 C2 mN1

Also, as per the problem


  4   0 R 03

Therefore, the
1/ 3
  
R0     9.8  1011 m
 4  0 
Terminal Questions
S
r 1. In order to calculate
 the bound
 surface and volume charge densities, we
q
first calculate D and then P as follows:
R
Consider a Gaussian spherical surface of radius r as shown in Fig. 5.7.
  
Fig. 5.7 
From Gauss’s law: D.dS  q, where q is the charge enclosed and D is
S
along rˆ , the normal to the sphere’s surface. Hence,
 q
D  4  r 2  q and D  rˆ
4 r 2

     D
Now D  0 E  P   E or E 

     
Thus, P  D  0E  D  0 D
134 
Unit 5 Dielectric Properties
   0    0 q
 P  D rˆ
  4 r 2
Therefore,  b at the outer surface of the Gaussian sphere is
   0 q
 b  P . rˆ  (i)
 4 r 2

The bound volume charge density is given by b   . P
Substituting the expression for divergence
 in spherical polar coordinates,
and since only the radial component of P is finite, we have
1 d 2 1 d  2   0 q 
b   (r Pr ) or b   r 0
r 2 dr r 2 dr   4 r 2 

Since the bound volume charge density is zero, the total bound charge on
the surface of the dielectric sphere (of radius R) is given by Eq. (i) above
for r  R. Thus, we get
  0
q b  4 R 2  b  q

Now consider a cavity of radius  at the centre of the sphere (Fig. 5.8). As

explained in Sec. 5.2.2, the normal to the surface of the cavity is along
 rˆ. Therefore, the surface charge density on the cavity is  b . Hence, for
r  , we can write [using Eq. (i) above]
Fig. 5.8
the total charge on the surface of the cavity   4 2b
    0   q 

  ( 4   2 ) 
    4   2 
  0
 q,

which is equal and opposite to the total positive charge on the surface of
the dielectric sphere.
2. Consider a cylindrical Gaussian surface S of area of cross-section A as
shown in Fig. 5.9.
 
E 0 A
Metal n̂ 
                  

Dielectric S n̂

D
Fig. 5.9
Applying Gauss’s law to this surface, we have
 

D.dS  qen  f A
S

D is normal to the unit vector normal to the curved surface of the cylinder
   
and D . dS is zero for it. D . dS is non-zero only for the circular cross-
section of thecylinder of area, say A, inside the dielectric because the
electric field E inside the metal (conductor) is zero.

 DA  f A and D  f nˆ (i) 135
Block 1 Electrostatics
where n̂ is the unit vector normal to the conducting surface.
   
Now, you know that D  0E  P   E from which
  (   0 ) 
P  (   0 ) E  D (ii)

The bound charge density on the surface of the dielectric is given by:
 
b   P . nˆ   P . nˆ (iii)

where nˆ  is the unit vector normal to the dielectric surface


 pointing
 in
the upward direction and n  n . Substituting the value of P and D from
ˆ ˆ
Eqs. (i) and (ii) in Eq. (iii), we get
  0   0
b   D   f
 
Thus, total charge density  t is
  0 
t  f   b  f  f  0 f
 
3. From Eq. (5.16), we can write the atomic polarisability in units of  0 as

3 K 1

N K 2
or
3 0.00088
 cm -3  3.1 10 23 cm 3
2.8  1019 3.00088

136

You might also like